Anda di halaman 1dari 140

Soal TO Mingguan

SOAL TRY – OUT UJIAN SARINGAN MASUK


POLITEKNIK KEUANGAN NEGARA STAN
TAHUN AKADEMIK 2016/2017

Perhatian!
1. Untuk semua soal, pilihlah satu jawaban yang paling tepat dari pilihan yang tersedia. Isikan jawaban anda pada lembar
jawaban yang disediakan sesuai dengan petunjuk pengisian.
2. Jawaban benar bernilai 4; jawaban salah bernilai -1, tidak menjawab bernilai 0 (nol)
3. Nilai mati berlaku pada setiap bagian soal. Anda memperoleh nilai mati jika pada salah satu dari dua bagian soal jawaban
benar yang Anda peroleh kurang dari 1/3 jumlah soal pada bagian tersebut.

BAGIAN PERTAMA
TES POTENSI AKADEMIK
(NOMOR 1 s.d. 120)

Untuk bagian ini, jawaban benar kurang dari 1/3 dari jumlah soal (kurang dari 40) berarti nilai mati.

KOSAKATA
1. SIMPTOM B. tidak sopan D. penghancuran
A. tanda C. buka-bukaan E. puing
B. akibat D. bebas
C. dampak E. terbuka 6. SAFARI
D. penyakit A. hiburan
E. trauma 4. TRIBUNE B. taman
A. berita utama C. perjalanan
2. YUSTISI B. surat kabar D. suaka
A. hakim C. pertandingan E. satwa
B. pengadilan D. panggung
C. peradilan E. penghargaan 7. EPIK
D. keadilan A. cerita bohong
E. pemutusan 5. RUIN B. cerita sedih
A. bangkai C. cerita dongeng
3. VULGAR B. roboh D. cerita kepahlawanan
A. porno C. pengeboran E. cerita bagus

SINONIM
8. HOTEL PRODEO B. sistem D. karib
A. hotel mewah C. cara E. dekat
B. hotel melati D. bohong
C. penjara E. pendekatan 13. ISBAT
D. rumah singgah A. penting
E. apartemen 11. DAUR B. utama
A. siklus C. kewajiban
9. KOMITMEN B. olah D. penetapan
A. teguh pendirian C. pakai E. umum
B. bertekad D. atur
C. niat E. pengulangan 14. ANALOG
D. percaya A. manual
E. kontrak 12. INTIM B. kuno
A. rahasia C. tombol
10. MODUS B. suami-istri D. sederhana
A. menipu C. tersembunyi E. identik

ANTONIM
15. DELUSI E. makian
A. khayal 16. CENDALA
B. de facto A. pintu 17. CENDEKIA
C. ilusi B. pendukung A. dungu
D. perasaan C. terpuji B. malas
D. intuisi D. hina C. gelap mata

Marketplace untuk Staners. Platform online yang menyediakan seluruh kebutuhan yang berkualitas untuk
(calon) mahasiswa, alumni, dan orang-orang yang mempunyai ketertarikan terhadap PKN STAN.
www.stanersstore.com @stanersstore stanersstore
Soal TO Mingguan

D. lamban 19. REMUK C. lunak


E. pintar A. rapi D. lembek
B. bagus E. padat
18. LESAP C. utuh
A. rembes D. cantik 21. GENCAR
B. jebol E. patah A. sunyi
C. hilang B. tenang
D. muncul 20. GEMPAL C. aktif
E. masuk A. kurus D. berpisah
B. padat E. jarang

ANALOGI
22. BERAS : NASI GORENG ::: C. bola : lingkaran
A. semen : pasir D. botol : kaca 25. BANTING : PECAH : PECAH
B. komputer : laptop E. lingkaran : jari-jari BELAH
C. kayu : gelondongan A. dorong : jatuh : rusak
D. kayu : api 24. KENYANG : MAKANAN : LAPAR ::: B. jatuh : retak : plastik
E. angin : kincir A. bersih : pembersih : sabun C. tembak : mati : makhluk hidup
B. pandai : ilmu : belajar D. hewan : panah : berburu
23. KOTAK : BALOK ::: C. senam : badan : sakit E. bongkar : bagus : mesin
A. gedung : kubus D. terang : lampu : gelap
B. drum : silinder E. kuat : minuman : haus

WACANA

Penderita diabetes membutuhkan waktu lebih lama untuk menyembuhkan luka di kaki. Butuh kesabaran ekstra dalam
merawatnya, sebab rasa frustrasi yang muncul ternyata bisa memperlama proses kesembuhan. Lamanya penyembuhan luka penderita
diabetes ini karena terkait kadar kadar kortisol atau hormon stres. Kadar kortisol akan meningkat jika penderita diabetes depresi atau
frustasi. Semakin depresi maka semakin tinggi kadar kortisolnya.
Penelitian di University of Nottingham mengungkapkan hubungan kadar kortisol dengan lamanya waktu yang dibutuhkan
untuk menyembuhkan luka (ulcer) di kaki. Penelitian tersebut dipublikasikan dalam jurnal Diabetologia edisi bulan Agustus. Dikutip dari
Healthday, Senin (9/8/2010), peneliti melibatkan 93 penderita diabetes yang mengalami luka di kaki. Pengamatan terhadap kondisi luka
dilakukan di awal penelitian, lalu diulangi secara periodik pada pekan ke-6, 12 dan 24.
Depresi yang ditunjukkan dengan peningkatan kadar kortisol menyebabkan proses penyembuhan luka di kaki berlangsung
lebih lama. Temuan ini mengisyaratkan pada penderita diabetes untuk mengurangi beban pikiran yang bisa menyebabkan stres,
terutama saat mengalami luka. Selain itu, sikap tidak sabaran juga menghambat penyembuhan luka.
Berdasarkan penelitian tersebut, para ahli kini tengah merancang metode perawatan luka khusus untuk penderita diabetes.
Salah satu tujuannya adalah untuk membantu mempercepat penyembuhan luka melalui pendekatan psikologis.

26. Yang berikut ini merupakan penyebab lamanya A. Tingginya kadar kortisol pada seseorang akan
penyembuhan luka di kaki pada penderita diabetes, membuatnya menjadi depresi.
KECUALI: B. Stress merupakan satu-satunya penyebab peningkatan
A. frustasi kadar kortisol yang menyebabkan diabetes.
B. beban pikiran yang banyak C. Penyakit diabetes dapat disembuhkan dengan
C. alergi pendekatan psikologis.
D. peningkatan kadar kortisol D. Mengurangi beban pikiran dapat membantu
E. perasaan tertekan mempercepat penyembuhan luka di kaki pada penderita
diabetes.
27. Manakah dari pernyataan berikut yang SESUAI dengan E. Depresi dapat ditunjukkan dengan pendekatan psikologis
wacana di atas?

KEMAMPUAN KUANTITATIF
28. Berapa banyak bilangan dua angka (dua digit) yang habis 29. Dari bilangan berikut ini, manakah merupakan daftar lengkap
dibagi 5 maupun 4? dan akurat semua bilangan yang dapat membagi habis 64?
A. Lima A. 1, 2, 4, 6, 8, 12, 16, 32, 64
B. Empat B. 1, 2, 4, 6, 8, 16, 32, 36, 64
C. Tiga C. 1, 2, 4, 8, 16, 32, 64
D. Tidak ada D. 1, 2, 4, 6, 8, 12, 16, 32, 36, 64
E. Tujuh E. 1, 2, 4, 5, 6, 8, 16, 32, 64

Marketplace untuk Staners. Platform online yang menyediakan seluruh kebutuhan yang berkualitas untuk
(calon) mahasiswa, alumni, dan orang-orang yang mempunyai ketertarikan terhadap PKN STAN.
www.stanersstore.com @stanersstore stanersstore
Soal TO Mingguan

30. Berat dari empat paket masing-masing 30, 40, 70 dan 120 kg. C. 4
Manakah dari total berat berikut (dalam kg) yang bukan D. 2
merupakan berat dari kombinasi yang dapat dibuat dari empat E. 1
buah paket tersebut?
A. 140 Untuk soal nomor 9 s/d 10 berdasarkan data berikut!
B. 220 Berikut adalah suku bunga Bank Bimbel Infinity:
C. 230 Deposito Bunga (tahun)
D. 240 1 bulan 5%
E. 270 3 bulan 5%
6 bulan 5%
31. Bobot minimum dari 12 telur jika 3 dari 10 telur tersebut 12 bulan 6%
berbobot 15 sampai 25 ons per butir dan yang lainnya berbobot 24 bulan 6%
20 sampai 30 ons perbutir adalah ...
Reguler 3%
A. 200 ons
B. 225 ons
C. 245 ons 36. Jika Bang Dion mendepositokan uangnya sebesar Rp
D. 345 ons 10.000.000,- selama 3 bulan, maka jumlah uang bang Dion
E. 375 ons setelah 3 bulan?
A. Rp 10.500.000,-
32. Jika B + E + N + I = 32. Maka, I + M + A + N = …? B. Rp 10.125.000,-
A. 37 C. Rp 500.000,-
B. 24 D. Rp 125.000,-
C. 16 E. Rp 105.000,-
D. 8
E. 22 37. Jika Kak MeeMoo mempunyai uang Rp 12.000.000,- dan 50%
didepositokan selama 6 bulan dan 50% lagi ditabungnya
33. Perusahaan Bimbingan Belajar terbesar di dunia “STAN –Pro selama 6 bulan. Maka, jumlah uang kak MeeMoo setelah 6
College” cabang Los Angeles mempunyai 20 karyawan, yaitu 4 bulan adalah ...
orang Manajer, 6 orang Supervisor dan 10 orang staf Kepala A. Rp 24.000,-
Bagian. Gaji Manajer adalah 4 kali gaji Supervisor, gaji B. Rp 6.015.000,-
Supervisor dua kali gaji staf Kepala Bagian. Maka, gaji Kepala C. Rp 12.024.000,-
Bagian jika total gaji seluruh karyawan Rp 162.000.000,- ? D. Rp 12.124.000,-
A. Rp 3.000.000,- E. Rp 15. 084.000,-
B. Rp 6.000.000,-
C. Rp 8.000.000,- 38. 70% dari x = 42 dan 2 y = 40 , maka hubungan x dan y adalah
D. Rp 24.000.000,- 3
...
E. Rp 26.000.000,-
A. x > y
B. x < y
34. Manganjur Marudut Sidabutar adalah seorang siswa “Bimbel
C. x = y
Infinity” paling ganteng dan berprestasi, berlari menuju rumah
D. x dan y tidak dapat ditentukan hubungannya
pacarnya yang berjarak 30 km dengan kecepatan 2x km/jam.
E. y = 2x
Jika, Bang Benny adalah pacarnya dan berlari dari rumahnya
menuju rumah Manganjur dengan kecepatan x km/jam. Jika,
39. Seorang pedagang memperoleh gaji minimal mingguan Rp
mereka berangkat pada waktu yang sama dan bertemu 20
250.000,- ditambah komisi sebesar 15% dari jumlah penjualan
menit kemudian, maka berapa kecepatan Manganjur berlari?
di atas Rp 4.000.000,- yang dicapai selama seminggu. Jika,
A. 15 km/jam
petugas tersebut mengharapkan memperoleh penghasilan Rp
B. 30 km/jam
650.000,- untuk satu minggu, berapakah jumlah minimum yang
C. 60 km/jam
harus dicapainya selama seminggu?
D. 120 km/jam
A. Rp 4.000.000,-
E. 150 km/jam
B. Rp 5.000.000,-
C. Rp 6.000.000,-
35. Benny ”Si Cantik Rupawan” dan Novilda ”Si Ganteng” akan
D. Rp 10.000.000,-
berjanji sehidup dan semati. Benny berjalan dari rumahnya ke
E. Rp 8.000.000,-
rumah Novilda pada pukul 03.00 wib yang berjarak 34 km. Satu
jam kemudian Novilda berlari dari rumahnya ke rumah Benny.
40. Tiga buah rumah dapat diselesaikan oleh 5 orang tukang
Kecepatan Benny VB km/jam, kecepatan Novilda VN km/jam
dan mereka bertemu 5 jam dari rumah Novilda. Jika, V B = (2)V , dalam waktu 36 hari. Tentukan banyak tukang yang diperlukan
untuk menyelesaikan 8 rumah dalam waktu 60 hari!
N

maka V N = ... ? A. 4
A. 16 B. 6
B. 8 C. 8

Marketplace untuk Staners. Platform online yang menyediakan seluruh kebutuhan yang berkualitas untuk
(calon) mahasiswa, alumni, dan orang-orang yang mempunyai ketertarikan terhadap PKN STAN.
www.stanersstore.com @stanersstore stanersstore
Soal TO Mingguan

D. 10 A. 1 : 1 : 2
E. 18 B. 1 : 5 : 2
C. 3 : 4 : 5
41. Sebuah sepeda motor dapat menempuh 15 km dengan 8 Liter D. 30 : 40 : 50
bensin. Maka, untuk menempuh sejumlah 25 km diperlukan ...
E. 30: 60: 90
Liter.
A. 7
48. Kak Zizah dapat mencuci pakaian dalam 20 menit, sedangkan
B. 40/3
Bang Uzar mengerjakan pekerjaan yang sama 10 menit lebih
C. 16
lama dari kak Zizah. Jika, mereka bekerja bersama-sama,
D. 20
berapa lama mereka menyelesaikan pekerjaan itu?
E. 27
A. 10 menit
B. 12 menit
42. Jarak 2 buah kota dapat ditempuh dalam waktu 3 jam dengan
C. 12 ¼ menit
mobil berkecepatan rata-rata 70 km/jam. Jika, waktu yang
D. Lebih dari 15 menit
tersedia hanya 2 jam, berapa kecepatan mobil yang harus
E. Lebih dari 30 menit
dipacu agar sampai tujuan tepat waktu?
A. 105 km/jam
49. Seorang anak yang tingginya 160 cm difoto dalam ukuran kecil
B. 90 km/jam
dengan skala 1 : 40, kemudian foto tersebut diperbesar dengan
C. 60 km/jam
skala 5 : 1, maka tinggi anak dalam foto yang terakhir adalah ...
D. 140/3 km/jam
A. 20 cm
E. 125 km/jam
B. 30 cm
C. 40 cm
43. Jika populasi suatu negara bertambah satu orang tiap 15 detik,
D. 50 cm
berapa banyak pertambahan populasinya selama 30 menit?
E. 70 cm
A. 120
B. 80
50. Keliling ∆BEF pada gambar berikut
D C

C. 60
adalah ...
D. 40 3

A. 5 cm
E. 55
B. 4 cm
F

C. 3 cm
1

44. (142 x117 ) + (173x142) + (142 x 210) = ...


D. 2 cm A B E

A. 70.000 E. 7 cm
B. 71.000
C. 72.000 51. Suatu kelas terdiri dari 84 siswa. 1/3 dari seluruh siswa itu
D. 73.000 menyukai olah raga berenang, 1/6nya menyukai berenang dan
E. 76.000 sepak bola, dan 36 orang tidak menyukai keduanya. Berapa
orang yang hanya menyukai sepak bola?
45. Pegawai Bimbel Infinity ingin memasang bendera di sepanjang A. 14
jalan S. M. Raja mulai dari ujung jalan ke ujung jalan lainnya. B. 20
Jika, panjang jalan adalah 1918 m, berapa banyak bendera C. 28
Bimbel Infinity yang dipasang dengan jarak 14 m? D. 34
A. 137 E. 52
B. 138
C. 139 52. Suatu kelas terdiri dari 65 siswa. 40 siswa menyukai Bang
D. 141 Heru. Jika jumlah siswa yang menyukai Bang Heru dan Bang
E. 180 Benny setengah dari siswa yang hanya menyukai Bang Benny
dan 5 orang tidak menyukai keduanya, maka banyak siswa
46. Dalam rangka ulang tahun ”Bimbel Infinity”, SPC menggelar yang menyukai Bang Benny adalah ...
Bazar ”Gebyar Prestasi Anak Bangsa”. Pada kesempatan ini A. 10
untuk program belajar Grand Class diberi diskon 70%, B. 15
sedangkan Regular Class diberi diskon 60%. Dengan demikian C. 20
jumlah uang yang dibayar siswa untuk Grand Class D. 40
dibandingkan Regular Class adalah ... E. 60
A. Lebih kecil
B. Lebih besar 53. Di dalam suatu kelas terdapat 40 siswa dan nilai rata-rata tes
C. Sama Matematika adalah a, guru Matematika menaikkan nilai tiap
D. Tidak dapat ditentukan siswa sebesar 5. Tentukanlah rata-rata nilai baru siswa
E. Dua kali lipat tersebut!
A. a + 40
47. Di antara berikut ini, manakah yang bukan merupakan sisi-sisi B. a + 200
dari segitiga siku-siku? C. 5a
Marketplace untuk Staners. Platform online yang menyediakan seluruh kebutuhan yang berkualitas untuk
(calon) mahasiswa, alumni, dan orang-orang yang mempunyai ketertarikan terhadap PKN STAN.
www.stanersstore.com @stanersstore stanersstore
Soal TO Mingguan

D. 8a E. xy - 1
E. a + 5
60. Untuk menjadi mahasiswa STAN, seseorang harus bimbingan
54. Dari suatu kelas yang terdiri dari 40 orang, diperoleh nilai rata- di Bimbel Infinity, lulus tes TPA dan lulus Bahasa Inggris.
rata kelas 7,00 untuk mata pelajaran Matematika. Jika, 5 nilai Dalam tes yang diikuti 40 peserta, 35 orang bimbingan di
siswa terendah dengan rata-rata 6 dan 10 nilai siswa tertinggi Bimbel Infinity, 30 orang lulus TPA dan 25 orang lulus Bahasa
dengan rata-rata 8,5 dikeluarkan, maka nilai rata-ratanya Inggris. Paling banyak berapa orang yang lulus STAN?
adalah: A. 5 orang
A. 8,2 B. 10 orang
B. 7,6 C. 15 orang
C. 7,1 D. 25 orang
D. 6,6 E. 30 orang
E. 8,8
61. Pada suatu segi banyak beraturan, besar sudut-sudutnya
55. Suatu kota terdiri dari 226 keluarga dengan rata-rata jumlah masing-masing 1440. Segi berapakah yang dimaksud?
anggota keluarga adalah 5 orang dan jumlah orang dewasa A. 8
seluruhnya 596. Peluang anak-anak terserang penyakit Z B. 9
adalah 0,5. Berapakah peluang banyak anak yang tidak C. 10
terserang penyakit Z? D. 11
A. 267 E. 15
B. 367
C. 467 62. Lina memiliki perangko 80% lebih banyak dari pada punya
D. 534 Fino. Banyaknya perangko Fino 3/5 dari perangko Rara. Jika,
E. 624 Fino memberikan 150 perangko ke Rara, maka Rara akan
memiliki perangko tiga kali dari sisa perangko Fino. Total
56. Sebuah kotak berisi 45 kelereng merah, 55 kelereng putih dan perangko tersebut adalah ...
x kelereng biru. Jika, diambil satu kelereng dari kantong A. 450
tersebut, nilai kemungkinan terambilnya kelereng biru adalah B. 750
7 . Maka, x adalah ... C. 810
25 D. 2010
A. 20 E. 3010
B. 25
C. 35 63. BF = FC dan DE = 6 EC. Berapakah perbandingan luas daerah
D. 45 yang tidak diarsir dengan luas yang A 7m B
E. 55 diarsir?
A. 9/2
57. Dua dadu dilempar. Tentukan, peluang mata dadu berjumlah B. 19/2 2n F
bilangan prima! C. 9/19
A. 13/36 D. 2/19
B. 10/36 E. 3/19 D 6m E C
C. 6/36
D. 3/36 64. Nilai 12 – 22 + 32 – 42 + … + 20012 – 20022 + 20032 adalah …
E. 9/36 A. 4005
B. 4008
58. Presiden dan Wakil Presiden beserta 6 menteri melakukan C. 200700
rapat di ”Bimbel Infinity” pada meja berbentuk melingkar, D. 2002006
banyak cara mereka duduk adalah ... E. 2505006
A. 450
B. 540 65. Banyak bilangan bulat antara 1000 dan 2005 yang habis dibagi
C. 4050 13 adalah ...
D. 5040 A. 78
E. 5540 B. 68
C. 58
59. Luas segitiga dinyatakan dalam (x dan D. 48
y) adalah ... (2,y) E. 38
A. xy
2 66. Nomor polisi mobil-mobil di negara ”Hidup Segan Mati Pun
B. xy − 1 Jadi” selalu berupa 4 angka. Selain itu, jumlah keempat angka
2
pada setiap nomor juga harus habis dibagi 5. Jumlah nomor
C. xy polisi terbesar dan nomor polisi terkecil adalah ...
(x,0)
D. 2 xy − 1 A. 40

Marketplace untuk Staners. Platform online yang menyediakan seluruh kebutuhan yang berkualitas untuk
(calon) mahasiswa, alumni, dan orang-orang yang mempunyai ketertarikan terhadap PKN STAN.
www.stanersstore.com @stanersstore stanersstore
Soal TO Mingguan

B. 35 C. 4
C. 30 D. 5
D. 25 E. 6
E. 20
73. Pada saat liburan akhir tahun, maskapai penerbangan ”Bimbel
67. Kereta api ”Wahana Bimbel Infinity” berjalan pada lintasan Infinity” menerapkan kebijakan 80% tiket dijual pada harga
lingkaran dengan diameter 3,5 m. Untuk menempuh satu batas atas dan sisanya dijual pada harga batas bawah.
putaran penuh diperlukan waktu 22 detik. Kecepatan kereta api Sementara, pada hari biasa 70% tiket dijual pada harga batas
... km/jam. bawah dan sisanya dijual pada harga batas atas. Jika, daya
A. 180 angkut pesawat 200 orang, harga tiket batas bawah x, dan y
B. 18 untuk batas atas, maka selisih pendapatan sekali penerbangan
C. 1,8 pada saat liburan akhir tahun dan hari biasa adalah ...
D. 0,18 A. 100x – 100y
E. 1,18 B. -100x + 100y
C. 100x + 100y
68. Sebuah akuarium dengan ukuran alas 40 cm x 80 cm dan D. x – y
tinggi 60 cm. Akuarium itu diisi setengahnya dengan air. Jika, E. 100x - y
dalam akuarium dimasukkan 6 buah hiasan yang sama persis,
tinggi air akan naik 3 cm. Tentukanlah, volume tiap hiasan! 74. Harga barang dipotong 50% untuk sebuah penjualan. Berapa
A. 19.200 cm3 persen harga barang harus dinaikkan agar sama dengan harga
B. 9.600 cm3 semula?
C. 3.200 cm3 A. 50%
D. 1.600 cm3 B. 75%
E. 2.600 cm3 C. 100%
D. 200%
69. Maya berdiri di dekat sebuah menara. Panjang bayangan Maya E. 275%
adalah 1,25 m dan bayangan menara 23, 3 m. Jika, tinggi
badan Maya 1,5 m, maka berapakah tinggi menara itu? 75. Jika tutup sebuah kotak memmpunyai luas 120 cm2, sisi depan
A. 2,796 m mempunyai luas 96 cm2 dan sisi samping mempunyai luas 80
B. 27,96 m cm2. Tinggi kotak tersebut adalah ... (dalam cm)
C. 62,79 m A. 10
D. 6,279 m B. 12
E. 4,286 m C. 16
D. 18
70. Nilai dari 1 − 1 1 − 1 1 − 1 1 − 1  ⋅ ⋅ ⋅ E. 8
    
 2  3  4  5
A. 1/5 76. Jika, ∠ABC adalah siku-siku, maka Y A

B. 2/5 adalah ...


C. 3/5
0 0
A. 25 Y X
65 0
D. 1 B. 55
B

E. 4/5 C. 155 B
D. 200
71. Benny ”Si Tampan” membuat teka-teki”. Jika a = 9 9! dan E. 250
b = 10 10! ”, maka hubungan a dan b adalah …
77. Seutas tali dipotong menjadi 10 bagian dengan panjang tiap
A. a > b
potongan membentuk barisan aritmatika. Jika panjang tali
B. a < b
terpanjang 23,5 dan terpendeknya 16,3 cm. Panjang tali mula-
C. a = b
mula adalah ... cm.
D. tidak dapat ditentukan
A. 100,8
E. a = 2b
B. 129
C. 159
72. Lakukanlah operasi kali, bagi, tambah atau kurang pada
D. 199
bilangan yang ada di kotak untuk menghasilkan bilangan yang
E. 399
di dalam lingkaran!
C
0
23
1 7 2 4 7 4
78. XAQ sejajar YBR. Perpanjangan
3
5
3
3
8
... garis lurus AB dan PQR Y
B

berpotongan di C. Bila besar sudut


R

CAP = 900, sudut CAQ = 500 dan


sudut ACP = 230. Hitunglah ∠YBA ! 50
0

A. 2
A
X
Q
B. 3 A. 1030
P
Marketplace untuk Staners. Platform online yang menyediakan seluruh kebutuhan yang berkualitas untuk
(calon) mahasiswa, alumni, dan orang-orang yang mempunyai ketertarikan terhadap PKN STAN.
www.stanersstore.com @stanersstore stanersstore
Soal TO Mingguan

B. 630 E. 50 tahun
C. 500
D. 230 85. Berapa umur nenek 10 tahun ke depan, apabila 3 tahun yang
E. 630 lalu umurnya 60 tahun?
A. 60 tahun
79. BCD sebuah garis lurus A B. 70 tahun
dan DE sejajar BA. Bila x E C. 63 tahun
BC = CA, sudut ADB = D. 73 tahun
100, sudut ACB = 400, E. 90 tahun
sudut BAC = x0 dan 400 100 y
sudut ADE = y0. B C D 86. Manakah pecahan di bawah ini yang lebih kecil dai 5/4?
Hitunglah nilai y! A. 56/34
A. 400 B. 16/7
B. 700 C. 35/26
C. 900 D. 18/28
D. 1000 E. 27/25
E. 1550
87. Sebuah bejana berbentuk silinder berisi air 1/3nya. Jika,
80. Jika 6 p = 3 dengan p dan q adalah bilangan asli, maka ... kemudian ditambah air sebanyak 3 liter lagi, bejana ini akan
2q − 1 berisi kira-kira ½ nya. Berapa literkah kapasitas bejana itu?
A. p < q A. 15 liter
B. p > q B. 18 liter
C. p = q C. 24 liter
D. tidak dapat ditentukan D. 27 liter
E. q = 2p E. 30 liter

81. Rata-rata nilai dari siswa wanita di suatu kelas adalah 73 dan KEMAMPUAN PENALARAN
siswa laki-laki adlaah 71. Rata-rata nilai kelas itu adalah 71,8. 88. 9, 9, 9, 6, 9, 3, ….., …..
Persentase selisih banyak siswa laki-laki dan wanita adalah ... A. 11, 11
% B. 13, 17
A. 50 C. 9, 0
B. 40 D. 19, 10
C. 30 E. 10, 11
D. 20
E. 10 89. 49, 64, 64, 49, 81, ….., …..,
A. 100 , 25
82. Rata-rata tinggi 20 siswa adalah 155 cm. Setelah diteliti B. 25, 16
ternyata terjadi kesalahan mencatat seharusnya 198 cm, tetapi C. 36, 81
tercatat 189 cm. Hitunglah rata-rata tinggi yang terkoreksi! D. 81, 36
A. 144,45 E. 36, 100
B. 145,45
C. 155,45 90. 31, 31, 33, 37, 35, 41, 37, ….., …..
D. 165,45 A. 39, 39
E. 175,45 B. 41, 41
C. 39, 41
83. Harga sebuah buku mula-mula Rp 10.000.000,-. Jika harga D. 41, 43
buku tersebut naik 20%, kemudian turun 20%, maka ... E. 43, 39
A. untung 4%
B. rugi 4% 91. 11, 13, 17, 19, 23, 29, ….., …..
C. untung 5% A. 33, 39
D. rugi 5% B. 31, 35
E. untung 7% C. 35, 39
D. 31, 37
84. Usia Herta 10 tahun lebih muda dari usia Ela. Perbandingan E. 31, 39
usia Herta dan Ela 10 tahun yang lalu adalah 5 : 6, sedangkan
perbandingan usia Herta dan Ela 10 taun yang akan datang 92. 3, 4, 8, 9, 18, 19, ….., …..
adlaah 8 : 9. Tentukanlah usia Ela sekarang! A. 37, 38
A. 30 tahun B. 38, 39
B. 35 tahun C. 39, 40
C. 40 tahun D. 6, 37
D. 45 tahun E. 33, 39

Marketplace untuk Staners. Platform online yang menyediakan seluruh kebutuhan yang berkualitas untuk
(calon) mahasiswa, alumni, dan orang-orang yang mempunyai ketertarikan terhadap PKN STAN.
www.stanersstore.com @stanersstore stanersstore
Soal TO Mingguan

A. Jika tulisanku dimuat di surat kabar, maka aku tidak


93. 60, 55, 56, 45, 48, 35, 32, 25, ….., ….. bersemangat
A. 12, 10 B. Jika tulisanku tidak dimuat di surat kabar, maka aku
B. 0, 15 bersemangat
C. 22, 15 C. Jika aku bersemangat, maka aku menulis lagi
D. 1, 15 D. Jika aku tidak bersemangat, maka tulisanku tidak dimuat di
E. 15, 22 surat kabar
E. Jika aku semangat, maka tulisanku tidak dmuat di surat
94. Semua karyawan tidak malas. Semua pelajar tidak berambut kabar
panjang. Maka:
A. Semua karyawan tidak berambut panjang Informasi berikut untuk soal No. 101 – 104
B. Semua pelajar tidak malas Pada suatu jamuan makan malam di istana negara, 8
C. Semua karyawan dan semua pelajar tidak malas dan tidak orang – Bapak SBY, Ibu Ani, Barrack Obama, Michelle, Bapak
berambut panjang MENLU, Bapak MENKOMINFO, Bapak KAPOLRI, dan Bapak
D. Sebagian pelajar malas Panglima TNI duduk mengelilingi satu meja bundar.
E. Tidak ada jawaban yang tepat. 1. Bapak SBY duduk berseberangan dengan Bapak KAPOLRI
2. Michelle duduk di antara Ibu Ani dan Obama
95. Tidak semua analisa terbukti benar, sementara analisa hasil 3. Bapak Panglima TNI dan Obama duduk tepat berhadapan
skripsi tidak diuji. Maka: 4. Ibu Ani duduk 2 kursi terpisah dari Bapak KAPOLRI
A. Sementara sarjana tidak menulis skripsi
B. sementara analisa skripsi terbukti benar 101. Bapak SBY duduk dipisahkan oleh 1 kursi dengan Bapak
C. sementara analisa skripsi tidak terbukti benar MENLU, Pernyataan di bawah ini yang benar adalah ….
D. sementara analisa hasil skripsinya benar A. Bapak MENLU duduk di antara Bapak SBY dan Ibu Ani
E. semua analisa hasil skripsi terbukti benar B. Bapak MENKOMINFO duduk dipisahkan 2 kursi dengan Ibu
Ani
96. Semua kambing pemakan rumput. Sebagian binatang adalah C. Bapak MENKOMINFO dan Bapak MENLU duduk tepat
kambing. Maka: berseberangan
A. Semua pemakan rumput adalah kambing D. Ibu Ani duduk di seberang Bapak MENKOMINFO
B. Sebagian binatang pemakan rumput
C. Sebagian kambing pemakan rumput 102. Bila Michelle duduk dipisahkan oleh 2 kursi dengan Bapak
D. A dan D benar Menlu, pernyataan di bawah ini yang benar adalah ….
E. A, B, dan C benar A. Michelle dan Bapak MENKOMINFO dipisahkan oleh 2 kursi
B. Bapak MENLU dan Ibu Ani duduk berhadapan
97. Azizah menentukan prioritas dalam hidupnya sebagai berikut: C. Obama duduk di sebelah Bapak SBY
Azizah ingin TENTRAM sebelum KAWIN, POPULER setelah D. Bapak MENKOMINFO duduk di dekat Ibu Ani
punya JABATAN, KAWIN sebelum punya JABATAN, dan
KAYA setelah KAWIN. Maka keinginan Azizah adalah: 103. Bila Michelle dan Bapak MENKOMINFO duduk berseberangan,
A. POPULER dulu sebelum punya JABATAN. pernyataan di bawah ini yang benar adalah ….
B. Punya JABATAN dulu baru KAWIN A. Bapak SBY duduk berhadapan dengan Bapak MENLU
C. KAYA prioritas utama B. Bapak SBY duduk di sebelah Michelle
D. TENTRAM prioritas utama C. Obama duduk di sebelah Bapak MENLU
E. KAWIN prioritas utama D. Bapak MENKOMINFO duduk di antara Bapak Panglima TNI
dan Bapak MENLU
98. Jika saya sibuk, saya tidak akan datang ke pesta, maka:
A. Meskipun sibuk, saya tetap datang ke pesta 104. Beberapa kemungkinan di bawah ini yang tidak benar adalah
B. Sibuk seperti apapun, saya ada di pesta itu ….
C. Saya ada di pesta karena saya tidak sibuk A. Bapak Panglima TNI duduk di antara Bapak MENKOMINFO
D. Saya tidak sibuk dan saya tidak datang ke pesta dan Bapak SBY
E. Saya tidak datang ke pesta tapi saya tidak sibuk B. Obama dan Bapak MENLU dipisahkan oleh Bapak
KAPOLRI
99. Semua hewan yang termasuk mamalia tidak pernah bertelur. C. Michelle duduk di antara Ibu Ani dan Obama
Semua yang bertelur pastilah hewan. D. Bapak MENLU duduk di antara Bapak SBY dan Ibu Ani
A. Mamalia tidak termasuk hewan.
B. Terdapat hewan yang tidak bertelur Keterangan berikut untuk soal no. 104 – 110
C. Mamalia hidup di dua alam Delapan ekor monyet – A, B, C, D, E, F, G, dan H – mengendarai
D. Ikan paus mamalia yang bertelur. sebuah pesawat ruang angkasa menuju ke Mars. Setiap monyet
E. Semua mamalia bertelur akan duduk di delapan bangku yang berbeda. Bangku-bangku
tersebut di susun menjadi empat baris dan diberi angka berurutan
100. Jika tulisanku dimuat di surat kabar, maka aku bersemangat. dari depan ke belakang yaitu 1, 2, 3, dan 4. Tiap-tiap baris memiliki
Maka: 2 buah bangku yaitu satu bangku dengan jendela yang menghadap

Marketplace untuk Staners. Platform online yang menyediakan seluruh kebutuhan yang berkualitas untuk
(calon) mahasiswa, alumni, dan orang-orang yang mempunyai ketertarikan terhadap PKN STAN.
www.stanersstore.com @stanersstore stanersstore
Soal TO Mingguan

ke matahari dan satu bangku dengan jendela yang menghadap ke Manik-manik hijau tidak bersebelahan dengan manik-manik hijau
bulan. Persyaratan berikut harus dilakukan: lainnya.
• Jendela E tidak pernah menghadap matahari, tetapi jendela D Sebuah manik-manik merah di salah satu ujung tali dan sebuah
selalu menghadap matahari manik-manik hijau di ujung yang lainnya.
• F duduk di barisan 1 atau 2, tetapi D tidak duduk di barisan 2
atau 3 111. Jika manik-manik yang keenam dan ketujuh berwarna biru dan
• B duduk di barisan tepat di belakang barisan D yang kesepuluh berwarna merah, manakah dari yang berikut
• Jika jendela B menghadap matahari, maka jendela A yang pasti benar?
menghadap bulan A. Manik-manik yang kedua berwarna hijau.
• Jika D duduk di barisan 1, maka G duduk di barisan 4 B. Manik-manik yang kelima berwarna kuning.
• Jika B di barisan yang sama dengan F, maka jendela G C. Manik-manik yang kedelapan berwarna hijau.
menghadap matahari D. Sebuah manik-manik hijau berada di sebelah sebuah manik-
manik kuning.
105. Manakah dari yang berikut yang pasti benar? E. Manik-manik kedua berwarna kuning
A. Baris 1: D, C; Baris 2: B, F; Baris 3: A, E; Baris 4: G, H
B. Baris 1: D, E; Baris 2: F, B; Baris 3: H, A; Baris 4: C, G 112. Jika keempat manik-manik kuning bersebelahan, dan jika
C. Baris 1: D, F; Baris 2: B, A; Baris 3: G, E; Baris 4: H, C manik-manik yang kesepuluh berwarna kuning, manik-manik
D. Baris 1: D, F; Baris 2: B, E; Baris 3: C, A; Baris 4: H, G manakah yang berwarna biru?
A. manik-manik keempat
106. Jika E duduk di baris 2, manakah dari yang berikut yang B. manik-manik kelima
merupakan daftar lengkap dari monyet yang dapat duduk di C. manik-manik keenam
baris 4? D. manik-manik ke tujuh
A. A, G, H C. A, C, F, H E. manik-manik pertama
B. C, G, H D. A, C, G, H
113. Jika setiap manik-manik biru berada di sebelah manik-manik
107. Jika jendela F menghadap ke bulan, tetapi F tidak duduk di hijau, dan jika keempat manik-manik kuning saling
baris 1, manakah dari yang berikut yang tidak mungkin benar? bersebelahan, maka manakah dari yang berikut yang
A. jendela A juga menghadap ke bulan. merupakan manik-manik kuning?
B. jendela C juga menghadap ke bulan. i. manik-manik keempat
C. jendela E juga menghadap ke bulan. ii. manik-manik kelima
D. jendela G juga menghadap ke bulan. iii. manik-manik keenam
iv. manik-manik ketujuh
108. Jika G dan H duduk di baris yang sama, semua dari yang
berikut adalah daftar lengkap dari baris 1 hingga 4 dari monyet A. I dan II saja
yang jendelanya menghadap ke bulan KECUALI: B. II dan III saja
A. F, A, E, G C. E, F, A, H C. III dan IV saja
B. F, E, A, G D. F, C, E, G D. II, III, dan IV
E. I dan IV saja
109. Jika jendela A menghadap matahari, tetapi F tidak duduk di
baris 1, manakah dari yang berikut yang pasti benar? 114. Jika manik-manik kelima dan keenam berwarna biru dan
A. C dan G duduk di baris yang sama manik-manik kesembilan berwarna merah, manakah dari yang
B. A dan B duduk di baris yang sama. berikut yang pasti benar?
C. A dan F duduk di baris yang sama. a. salah satu dari manik-manik hijau berada di sebelah manik-
D. Jendela E menghadap ke arah yg sama dengan jendela G manik biru.
b. salah satu dari manik-manik merah berada di sebelah
110. Jika jendela B dan jendela F menghadap ke arah yang manik-manik hijau
berlawanan, manakah dari yang berikut yang pasti benar? c. manik-manik kedua berwarna kuning
A. Jendela G menghadap ke bulan, dan F duduk di baris 2 d. manik-manik kedelapan berwarna kuning
B. F duduk di baris 2 dengan jendela menghadap ke bulan, dan e. manik-manik ketujuh berwarna kuning
jendela A menghadap matahari
C. D dan F duduk di baris yang sama 115. Jika manik-manik kelima, kedelapan, kesembilan dan
D. G duduk tepat di depan H kesepuluh berwarna kuning, manakah dari yang berikut yang
pasti benar?
Keterangan berikut untuk soal no. 111 – 116 i. manik-manik keempat berwarna hijau
Seorang anak menyusun sebelas manik-manik yang berbeda warna ii. manik-manik keenam berwarna biru
pada seutas tali. iii. setiap manik-manik berada di sebelah paling sedikitnya
Dari kesebelas manik-manik tersebut, empat berwarna kuning, tiga satu manik-manik kuning
berwarna merah, dua berwarna biru, dan dua berwarna hijau.
Manik-manik merah bersebelahan dengan manik-manik merah A. I saja
lainnya. B. II saja
Manik-manik biru bersebelahan dengan manik-manik biru lainnya. C. I dan II saja

Marketplace untuk Staners. Platform online yang menyediakan seluruh kebutuhan yang berkualitas untuk
(calon) mahasiswa, alumni, dan orang-orang yang mempunyai ketertarikan terhadap PKN STAN.
www.stanersstore.com @stanersstore stanersstore
Soal TO Mingguan

D. I, II, dan III A B C D


E. Tidak ada jawaban yang benar
119.
116. Jika satu manik-manik hijau berada di sebelah sebuah manik-
manik merah dan manik-manik hijau yang lainnya berada di
sebelah sebuah manik-manik biru, manakah dari yang berikut ?
yang pasti benar?
A. manik-manik kedua berwarna biru
B. manik-manik keempat berwarna hijau
C. manik-manik keempat berwarna kuning
D. manik-manik ketujuh berwarna kuning
A B C D
117.

? 120.

A B C D

118.

A B C D

Marketplace untuk Staners. Platform online yang menyediakan seluruh kebutuhan yang berkualitas untuk
(calon) mahasiswa, alumni, dan orang-orang yang mempunyai ketertarikan terhadap PKN STAN.
www.stanersstore.com @stanersstore stanersstore
Soal TO Mingguan

BAGIAN KEDUA
TES BAHASA INGGRIS
(NOMOR 121 s.d. 180)

Untuk bagian ini, jawaban benar kurang dari 1/3 dari jumlah soal (kurang dari 20) berarti nilai mati.

121. Despite their differences, they had enough ……… to resolve D. during
their disputes and work together again. E. therefore
A. alike
B. the best 128. When this remarkable flight expert was a small child, he once
C. similar ……….a giant bird who could fly anywhere he wanted to go.
D. same A. only would be
C. in common B. wished he had been
C. would wished he were
122. Patty really had a talent and love ……….., so her parents took D. would wish
off their pressure for her to go into a profession. E. would be wished
A. cooking
B. to cook 129. Almost everything Rico did was ……….; he rarely displayed
C. has cooked any sense of adventure.
D. for cooking A. practically reasonable
E. cooked B. for practical reasons
C. in order to reason practically
123. Evan had …………..time at the party that he didn’t leave until D. for practicing reasons
the sun rose the next day. E. reasonable practically
A. the more wonderful
B. the most wonderful 130. We really fancy ……….to see Beyonce in London. Do you want
C. such a wonderful to come with me?
D. so wonderful A. going
E. so a wonderful B. to go
C. go
124. The retail industry has strenuously ………to federal efforts to D. went
create basic protections for night workers. E. goes
A. objected
B. objections 131. The men ……….. to Australia because they know much about
C. objective it.
D. object A. should have been
E. objects B. must have
C. must be
125. After the accident, the thought that she ………….more careful D. must have been
went through Santi’s mind that her brooding led to a deep E. might have been
depression.
A. could be 132. Employers look for …………when interviewing perspective
B. had been employees.
C. will had been A. dependability
D. could have been B. dependent
E. will has been C. dependence
D. dependable
126. Wagiman does not have the dark soul of a tortured artist, nor E. depend
……..about the pointlessness of everyday existence.
A. broods 133. She ………..spend her time revising her school lessons rather
B. was he brooding than take a walk with her friends.
C. brooding A. has better
D. does he brood B. prefers
E. brooded C. would rather
D. would prefer to
127. The Newton City School District Board of Education will meet to E. had better
vote on the budget …….it is presented to the public.
A. since 134. The man ……….for smoking marijuana is talking about how
B. after unfair the law is.
C. then A. arresting

Marketplace untuk Staners. Platform online yang menyediakan seluruh kebutuhan yang berkualitas untuk
(calon) mahasiswa, alumni, dan orang-orang yang mempunyai ketertarikan terhadap PKN STAN.
www.stanersstore.com @stanersstore stanersstore
Soal TO Mingguan

B. arrested A. will have been finished


C. is arrested B. be finished
D. is arresting C. will be finished
E. has arrested D. is finished
E. have been finished
135. The 12 km race route, ……., goes from San Francisco Bay
across town to the Pacific Ocean. 143. Having been announced as one of the best colleges in
A. which most of it is lined with people Indonesia, ……………….
B. which most of people line it A. many students compete to be admitted to it
C. it is lined with most people B. Indonesian University plans to build some new departments
D. most of which is lined with people C. the lecturers of Indonesian University are urged to work
E. was the lined with most people more professionally
D. the government will give more funding to the university
136. While my brother …….., my grandmother was making some E. a lot of students complete to be admitted
muffins.
A. was studying 144. Most of the students in this classroom have ………...argued
B. studied that they need to be involved in the training.
C. had studied A. persistently
D. studying B. persistence
E. has studied C. persisted
D. persistent
137. Their team got the best players. ………… they lost the game. E. persist
A. Therefore
B. In addition 145. ………the president has an affair with a widow has become the
C. Moreover deadline in several newspapers these three days.
D. For A. That
E. Nevertheless B. Since
C. Hence
138. Dolok Masihul, ………………….., is located about 20 D. Whereas
kilometers away from Tebing Tinggi. E. Since
A. which one of the biggest towns in Serdang Bedagai
B. is one of the biggest towns in Serdang Bedagai 146. To make the house look more beautiful, it needs ……….
C. one of the biggest towns in Serdang Bedagai A. some more furnitures
D. one of the biggest towns in Serdang Bedagai which is B. some more furniture
E. it is the biggest towns in Serdang Bedagai C. several new furniture
D. a new furniture
139. According to the lecturers’ union, the large-scale project E. an new furniture
………………..by the end of this year.
A. would be accomplished 147. When ……….whether he knew who had killed the woman, the
B. will be accomplished little boy just kept silent.
C. will have accomplished A. was asked
D. will have been accomplished B. asking
E. would have been accomplished C. to ask
D. asked
140. Very little ………….we have in store for her. E. he is asking
A. she knows what surprises
B. does she know what surprises 148. While the girl was having a birthday party in the hall, her father
C. does she know what surprising was taking part in an ESQ training held in Hotel Indonesia.
D. she knows what kind of surprise It means that………….
E. she knows kind of suprise A. the girl cancelled having the birthday party
B. the girl’s father attended the birthday party despite the
141. Announcements ……..on this bulletin board must receive training
approval from the dean. C. the ESQ training was held in the same place as the birthday
A. posted party
B. posting D. the girl’s father didn’t attend her birthday party
C. which posted E. the girl’s father hadn’t attend to the birthday party because of
D. are posted the same time
E. that is posted
149. He wishes he could have lent some money to the poor woman
142. It is of great importance for all of the students that the living next to his house.
homework …………before next Tuesday. It means that ……….

Marketplace untuk Staners. Platform online yang menyediakan seluruh kebutuhan yang berkualitas untuk
(calon) mahasiswa, alumni, dan orang-orang yang mempunyai ketertarikan terhadap PKN STAN.
www.stanersstore.com @stanersstore stanersstore
Soal TO Mingguan

A. The poor woman didn’t live next to his house C. would have to turn
B. He couldn’t lend money to the poor woman D. could turned
C. He can’t lend money to the poor woman E. could have turned
D. He has lent some money to the poor woman
E. He will not lend some money to the poor woman 151. How many people ………….to the wedding party?
A. were invited
150. If you had worked faster, we …………in the project before the B. invited
deadline. C. did they invite them
A could turn D. has been invited
B. would turn E. had invited

Error Recognition
152. It was nothing short of a phenomena that this particular Board A B
of either to go skiing and to go skating.
A B C C D E
directors came to such a quick consensus.
D E 158. To make the event more exciting, Cheltam Publishers will ask
an
153. Complaining about the lack of police protection in the A B
A B award-winning novelist to host the written workshop in June.
neighborhood was the minister of the church and several C D E
parents
C D 159. An importance of the project requires that every employee
of children attending the local school. A B C
E become involved.
D
154. This is a film that proves not only that Duval can direct but, as if 160. The trade representative is asking for an acceptance fee for
A B A B
we didn’t know it, that he is one of America strongest screen $2,000 to represent the company in the latter’s foreign
C D operation.
presences. C D
E E

155. The teacher asked me, “Why can study English be so 161. The young woman standing next to my mother is equal as the
confusing A B C
A B little girl in that they have dimples on their cheeks.
for most of the students eating lunch in the cafeteria that I need D E
C D 162. At least three quarters of the monkeys in the zoo suffer from
to teach them very patiently?” A B C D
E
nutritionally deficiencies.
156. A total of ten thousand of dollars was spent on radio E
advertising,
A B 163. The girl lying on the sofa seems to have known much about
despite recent requests for budget-cutting measures. what
C D E A B
is the boy going to do once he graduates from Senior High
157. We have two alternative activities for our winter fraternity C D E
outing: School.

SOAL RUMPANG
Fill in the blanks with grammatically suitable words or phrases from the four choices given.
In France, the development of the judicial system after the break-up of the Carolingian Empire was similar to that in England: Both
involved the vesting of central legal authority in the Crown after a protracted struggle with feudal manorial courts. The essential features of the
…..(164)….. system now in effect in France were established after the French Revolution of 1789 by the Code Napoleon. This system
includes lower courts of wide jurisdiction, intermediate courts of appeal, a court to resolve jurisdictional conflicts among courts, and a supreme
appellate tribunal ….(165)…. the Court of Cassation. Many European and Latin American judicial systems are modeled on that of France.
In the Islamic world, the Koran is the source of law; justice traditionally has been dispensed by ……(166)….. priests in conjunction
with the king, or sultan. In the 20th century, this system still prevails in such Islamic countries as Yemen and Saudi Arabia.
In Turkey, however, executive, legislative, and judicial functions have been separated, and a judicial system similar to those of
Western countries has evolved.

Marketplace untuk Staners. Platform online yang menyediakan seluruh kebutuhan yang berkualitas untuk
(calon) mahasiswa, alumni, dan orang-orang yang mempunyai ketertarikan terhadap PKN STAN.
www.stanersstore.com @stanersstore stanersstore
Soal TO Mingguan

In other Middle Eastern and Asian countries that have attained independence since World War II, notably Sri Lanka, India, and
Israel, the courts also operate ….(167)…. to those of the West, that is, as relatively independent institutions within a parliamentary framework.
In Communist countries, the judicial system was usually patterned after that of the USSR, which included a hierarchy of courts
culminating in a supreme court. In the former Yugoslavia, however, all judges, even those of the highest tribunals, ….(168)…., not appointed.

164. A. judicial D. special trained


B. judicially E. specialed training
C. judge
D. judging 167. A. similar
E. judged B. similarity
C. similarly
165. A. calling D. similarities
B. called E. similary
C. calls
D. to call 168. A. elected
E. had called B. were elected
C. have been elected
166. A. specially trained D. to be elected
B. special training E. had been elected
C. specially training

Reading Comprehension I
From experience we know that investors may temporarily pull financial prices away from their long term trend level. Over-reactions
may occur – so that excessive optimism (euphoria) may drive prices unduly high or excessive pessimism may drive prices unduly low. New
theoretical and empirical arguments have been put forward against the notion that financial markets are efficient. According to the efficient
market hypothesis (EMH), only changes in fundamental factors, such as profits or dividends, ought to affect share prices. But the efficient
market hypothesis is sorely tested by such events as the stock market crash in 1987, when the Dow Jones index plummeted 22.6 percent –
the largest-ever one-day fall in the United States. This event demonstrated that share prices can fall dramatically even though, to this day, it is
impossible to fix a definite cause: a thorough search failed to detect any specific or unexpected development that might account for the
crash. It also seems to be the case more generally that many price movements are not occasioned by new information; a study of the fifty
largest one-day share price movements in the United States in the post-war period confirms this. Moreover, while the EMH predicts that all
price movement (in the absence of change in fundamental information) is random ( i.e., non-trending), many studies have shown a marked
tendency for the stock market to trend over time periods of weeks of longer. Various explanations for large price movements have been
promulgated. For instance, some research has shown that changes in estimated risk, and the use of certain strategies, such as stop-loss
limits and Value at Risk limits, theoretically could cause financial markets to overreact. Other research has shown that psychological factors
may result in exaggerated stock price movements. Psychological research has demonstrated that people are predisposed to ‘seeing’ patterns,
and often will perceive a pattern in what is, in fact, just noise.

169. It is stated in the passage that.... B. The largest disaster the US stock market has ever
A. financial markets generally act inefficiently if the investors experienced is a 22.6 fall in the crash of 1987.
act based on excessive optimism. C. Time periods of weeks or longer are the main reason why
B. financial prices may not be kept away from their long trend stock markets basically start to trend.
level for long periods of time. D. Price movement, according to the EMH, is not non-trending
C. investors’ optimism and their pessimism have little or no if there is change in fundamental information.
effect on financial long-term trend levels.
D. profits and dividends have only a temporary influence on the 172. The passage refers to the fact that....
validity of the efficient market hypothesis. A. psychological factors have the greatest role in large stock
price movements.
170. The passage mentions that....... B. financial markets overreact mainly due to stop-loss limits
A. the EMH depends on the Dow Jones index for the and value at risk factors
consistency of its hypotheses. C. changes in estimated risk take place where the stock-
B. the fall in shares may not always be accounted for by the holders are least likely to see them.
efficient market hypothesis. D. people are usually interested in detecting ways of behavior
C. the stock market crash in 1987 followed after a short period in the stock-market which in fact do not exist.
of economic development.
D.price movements are often the result of unexpected 173. The word” thorough” in the paragraph means....
developments in shareholders’ information. A. basic C. professional
B. complete D. specialized
171. Which of the following is TRUE according to the passage?
A. There were about fifty large one-day price movements in the 174. The word “promulgated” is closest in meaning to ………..
US in the post-war period. A. declared C. deteriorated

Marketplace untuk Staners. Platform online yang menyediakan seluruh kebutuhan yang berkualitas untuk
(calon) mahasiswa, alumni, dan orang-orang yang mempunyai ketertarikan terhadap PKN STAN.
www.stanersstore.com @stanersstore stanersstore
Soal TO Mingguan

B. kept secret D. differentiate

Reading Comprehension II
GATT (Article 6) allows countries to take action against dumping. The Anti-Dumping Agreement clarifies and expands Article 6, and
the two operate together. They allow countries to act in a way that would normally break the GATT principles of binding a tariff and not
discriminating between trading partners-typically anti-dumping action means changing extra import duty on the particular product from the
particular exporting country in order to bring its price closer to the “normal value” or to remove the injury to domestic industry in the importing
country. There are many different ways of calculating whether a particular product is being dumped heavily or only lightly. The agreement
narrows down the range of possible options. It provides three methods to calculate a product’s “normal value”. The main one is based on the
price in the exporter’s domestic market. When this cannot be used, two alternatives are available-the price charged by the exporter in another
country, or a calculation based on the combination of the exporter’s production costs, other expenses and normal profit margins. And the
agreement also specifies how a fair comparison can be made between the export price and what would be a normal price. Calculating the
extent of dumping on a product is not enough. Anti-dumping measures can only be applied if the dumping is hurting the industry in the
importing country. Therefore, a detailed investigation has to be conducted according to specified rules first. The investigation must evaluate all
relevant economic factors that have a bearing on the state of the industry in question. If the investigation shows dumping is taking place and
domestic industry is being hurt, the exporting company can undertake to raise its price to an agreed level in order to avoid anti-dumping import
duty. Detailed procedures are set out on how anti-dumping cases are to be initiated, how the investigations are to be conducted, and the
conditions for ensuring that all interested parties are given an opportunity to present evidence. Anti-dumping measures must expire five years
after the date of imposition, unless an investigation shows that ending the measure would lead to injury.

C. anti-dumping measures should normally be applied if the


175. It is stated in the passage that.... dumping is not hurting the importing country’s industry
A. the Anti-Dumping Agreement cannot exist without clarifying D. in case of proven dumping, the exporting company may
and expanding GATT’s article 6. have to increase the price of its products for export
B. some countries allow GATT to take responsibility for anti-
dumping activities in their domains 178. The passage states that.......
C. Article 6 and the Anti-Dumping Agreement are related but A. there is time-limit to anti-dumping measures
act independently of each other B. anti-dumping measures are rarely lifted, if ever
D. taking action against dumping may in cases lead to the C. injury usually follows the lifting of dumping measures
violation of the GATT principle of binding a tariff. D. it takes a maximum of five years to develop anti-dumping
measures
176. Which of the following is TRUE according to the passage?
A. the value of a product differs considerably in the exporting 179. The word ‘margin’ in the paragraph most closely relates to a (n)
and importing countries. ......point.
B. Domestic industry is in most cases injured if anti-dumping A. legal
measures are not taken for imports B. ideal
C. The normal value of a particular product depends on C. minimum
charging the extra import duty on it D. business
D. An imported product’s normal value is mainly calculated E. page
based on its price in its country of origin.
180. The word “discriminating” is closest in meaning to ………
177. The passage mentions that .... A. distinguishing
A. exporters usually try to sell their products at the highest B. extinguishing
possible C. reciting
B. the normal value of a product depends on fair comparison D. differing
between the normal price and the export price E. surfering

Marketplace untuk Staners. Platform online yang menyediakan seluruh kebutuhan yang berkualitas untuk
(calon) mahasiswa, alumni, dan orang-orang yang mempunyai ketertarikan terhadap PKN STAN.
www.stanersstore.com @stanersstore stanersstore
Kunci TO Mingguan
TES POTENSI AKADEMIK (TPA) X = 30 Km / Jam
Vm = 2X
KOSAKATA Vm = 60 Km / Jam
1. SIMPTOM = A. tanda 35. Kunci : D
2. YUSTISI = C. peradilan X
B 34 Km N
3. VULGAR = B. tidak sopan
4. TRIBUNE = D. panggung tB = 6 Jam tN = 5 Jam
5. RUIN = E. puing maka : S = SB + SN
6. SAFARI = C. perjalanan 34 = VB x tB + VN x tN
7. EPIK = D. cerita kepahlawanan 34 = (2)vm x 6 + VN x 5
SINONIM VN = 2
8. HOTEL PRODEO = C. penjara 36. Kunci : B
9. KOMITMEN = E. kontrak Biaya deposito 3 bulan = 5% / tahun
10. MODUS = C. cara 3 5
x10.000.000  125.000
11. DAUR = A. siklus 12 100
12. INTIM = D. karib Total uang = 10.125.000
13. ISBAT = D. penetapan 37. Kunci : C
14. ANALOG = E. identik Deposito + Tabungan
ANTONIM 6
x
5 6
x6000.000  x
3
x6.000.000
15. DELUSI X B. de facto 12 100 12 100
16. CENDALA X C. terpuji =15.000 + 9000
17. CENDEKIA X A. dungu Total uang = 12.024.000
18. LESAP X D. muncul 38. Kunci : C
19. REMUK X C. utuh 70%X = 42
20. GEMPAL X A. kurus 70
X  42
21. GENCAR X E. jarang 100
ANALOGI X = 60 2Y
3
 40

22. BERAS : NASI GORENG ::: Y  60


C. kayu : gelondongan
23. KOTAK : BALOK ::: 39. Kunci :E
B. drum : silinder Kwantiti = 650.000 – 250.000 = 400.000
24. KENYANG : MAKANAN : LAPAR ::: 10 X= 400.000
x
D. terang : lampu : gelap 100
25. BANTING : PECAH : PECAH BELAH
C. tembak : mati : makhluk hidup X = 4 Jt
26. C. alergi. Tidak disebutkan dalam wacana bahwa alergi dapat menjadi Penjualan = 4Jt + 4Jt = 8Jt
penyebab lamanya penyembuhan luka di kaki pada penderita 40. Kunci : C
diabetes. 5 tahun
27. D. Mengurangi beban pikiran dapat membantu mempercepat 3 Rumah 36 hari
penyembuhan luka di kaki pada penderita diabetes. X tahun
KEMAMPUAN KUANTITATIF 8 Rumah 60 hari
28. Kunci : B O1 O2 3 8
   8
Bilangang yang habis dibagi 5 maupun 4 : 20,40,60,80 S1t1 S 2t 2 5.36 X .60
29. Kunci : C 41. Kunci : B
1,.2,4,8,16,32,64 Jarak Bensin
30. Kunc : D 15 8

140 : 40 + 30 + 70 25 3 15 8

220 : 30 + 70 + 120 40
25 X
X 
230 : 40 + 70 + 120 3
31. Kunci : B
Berat minimum : 3 x 15 + 9 x 20 42. Kunci : A
= 45 + 180 = 225 Kecepatan Waktu

32. Kunci : A 70 3
1+ M + A + N = 9+13+1+14 X 2
= 37
33. Kunci : A 70 2
M = 4SP   X  105
X 3
M = 8SP 43. Kunci : A
SP = 2St 15 detik =1 1 menit= 4
4M + 6SP + 10SP + = 162Jt 30 menit = 30.4 = 120 org
32St + 12St + 10St = 162Jt 44. Kunci : B
54St = 162 142 x117 + 173 x 142 + 142 x 210 = 142 (117 + 173 +210 ) =
St = 3 Jt 71.000
34. Kunci : C 45. Kunci : B
S = 30 Km, t = 1/3 Jam Banyak bendera = (1918 : 14 ) + 1
SM + SB = S = 137 + 1
Vm x t + VB x t = 30 = 138
2X x 1/3 + X x 1/3 = 30 46. Kunci : D
2x x
  30 47. Kunci : C
3 3

Marketplace untuk Staners. Platform online yang menyediakan seluruh kebutuhan yang berkualitas untuk
(calon) mahasiswa, alumni, dan orang-orang yang mempunyai ketertarikan terhadap PKN STAN.
www.stanersstore.com @stanersstore stanersstore
Kunci TO Mingguan
 3   4    5
2 2 2
Jumlah anak = 226 x 5 – 596 = 530
Peluang yang terserang virus = ½ x 530 = 267
48. Kunci : B
56. BONUS
Ty = 20
57. Kunci : A
tBersama Tb
tT = 10 Jumlah bilangan prima = 13 bilangan, P = 13
1 1 1 1 1 3 2 36
     58. Kunci : D
tg ty ty 20 30 60
Cara :(8 – 1)= 7! X 6 x 5 x 4 x 3 x 1
Tg = 60  12menit 59. Kunci : B
5
xx  2 
1
49. Kunci : A L
2
Jrk pd peta Y
XY
Jrk sebenarnya
 y
Skala = 2
2 X-2 X
1 x
 x4
40 160 60. Kunci : D
Di perbesar 5 kali =>X= 4.5 = 20cm Bimbel di SPC

50. Kunci : B
TPA = 5
D 4 C
BI = 25 Lulus = STAN
3

5 F

Y
1
E
A B X
Keliling = 1 + 1/3 + 4/3 = 4cm 61. Kunci : C
BEF  CDF Besar sudut segi n beratuan = 180o – 360o/n; 144 =\ 180o–360o/n ;
x 2 4 n=10
 
4 6 3 62. Kunci : D
4
2
l=80%f + f = 18/10f
Y  12   
 
3 =3/5f
5 r+ 150 = 3 (f-150)
Y
3 r-3f=-600
r=750 ; f = 450 ; l=810
51. Kunci : B l+f+r = 2010
LABCD  14mn
63. Kunci : C 7
Larsir  14mn  mn  6mn
14 14 X A 7m B
36
2
X= 84-(14 +14 + 36 )= 20 9
2n F
 mn
52. Kunci : C 2
7 19
D 6m E C
Ltdkdiarsri  mn  mn
2X X 40-x
2 2
5
9 mn
Larsir 9
A B  2 
Ltdkdiarsir 19 mn 19
2x + x + 40 – x + 5 = 65 2
2x = 20 64. Kunci : D
53. Kunci : E (20032 – 20022)+…..(72 – 62)+(52- 42)+(32-22)+1
x 4005+….+13+9+5+1
 1+5+9+13+….4005
n
x A=1 ; b= 4 ; n=1002 ; un = 4005
a Sn = 2.007.006
40
65. Kunci : A
x  40a
Tiap siswa di tambah 5 nilai tambahan total 1001,1014,1027,…..2002
40.5  200 U1=1001 ; b =17 Un= 2002
n = 78
x  40a  200
66. Kunci : A
40a  200 N0.terkecil: 1 1 1 2 =5

40 N0.Besar : 9 9 9 8 =35
  a5 40
54. Kunci : D 67. Kunci : C
x S = keliling lingkaran

n = 22/7 x 3,5 = 11m
x V= s/t= 11m/ 22s = 0,5m/s = 1,8 km/jam
7
40 68. Kunci : D
x  280  (6.5  8,5.10) V6hiasan = 90 x 80 x 3 = 9.600 cm3
 280  (30  85) V1Hiasan = 9600/6 = 1600cm3
x  165 69. Kunci : B
tm m' tm 23,3
165   
  6,6 t Maya Maya ' 1,5 1,25
25
55. Kunci : A tm  27,96m

Marketplace untuk Staners. Platform online yang menyediakan seluruh kebutuhan yang berkualitas untuk
(calon) mahasiswa, alumni, dan orang-orang yang mempunyai ketertarikan terhadap PKN STAN.
www.stanersstore.com @stanersstore stanersstore
Kunci TO Mingguan
70. Kunci : A H = E – 10  H  10 5

 1  1  1  1  E  10 6
1  1  1  1   H  10 5
 2  3  4  5  H
  H  60  E  70
6
1 2 3 4
 x x x 85. Kunci : D
2 3 4 5 N – 3 = 60  N = 63

1 Maka, umur nenek 10 tahun ke depan = 73 tahun
5 86. Kunci: E
71. Kunci : B 5/4 = 1,25

a ?b 27/25 = 1,08

9
9! ? 10 10! 87. Kunci : B
1/3v + 3 = 1/2v
 9! ?
9
90
10
10! 
10
V= 18 L
9!10 ?10!9 KEMAMPUAN PENALARAN
88. C. 9, 0
9!9 x9!?10 x9!9
9! 10 9
72. Kunci : B 9, 9, 9, 6, 9, 3, ….., …..
7-3+1=5 ; 4-3+2 = 3 ; 4-8+7 = 3
73. Kunci : B -3 -3 -3
80 20
haribiasa  x 200 xY  xX 89. A. 100 , 25
100 100
7x7 8x8 9x9 10x10
70 30
haribiasa  x 200 xX  x 200 xX 49, 64, 64, 49, 81, 36, ….., …..
100 100
100Y  100 X
74. Kunci : C 8x8 7x7 6x6 5x5
Awal = 100% 90. E. 43, 39
Potong 50%= 50% Bilangan Prima
% dinaikkan = 50% / 50%=100%
75. Kunci : E 31, 31, 33, 37, 35, 41, 37, ….., …..
` P x L= 120
P x t = 96 => p = 96/T Bilangan ganjil
LT = 80 ; L = 80/T 91. D. 31, 37
96/t x 80/t=120 ; t2= 64 => t = 8cm Seri bilangan prima
76. Kunci : C 92. B. 38, 39
X + 65 = 90 =>X = 25 Polanya adalah +1, X2
Y = 180 -25 =155
77. Kunci : D
a =16,3 S10 = 10/2 (16, 3+ 23, 5)
U10 = 23,5 = 199 cm 93. B. 0, 15
a + 9b = 23,5 -4 -8 -16 -32
b = 0,8
78. Kunci : E 60, 55, 56, 45, 48, 35, 32, 25, ….., …..
YBA  CBR  BAQ  50 0

79. Kunci : D -10 -10 -10 -10


94. D. Tidak ada jawaban yang tepat.
DAB  DBC  180  40  70
1
2 95. C. sementara analisa skripsi tidak terbukti benar
ACD  104  DAC  30 0
0
96. B. Sebagian binatang pemakan rumput
ADE  DAB  100 0 97. D. TENTRAM prioritas utama
80. Kunci : D 98. C. Saya ada di pesta karena saya tidak sibuk
81. Kunci : D 99. B. Terdapat hewan yang tidak bertelur
xW  73 nW  W 100. D. Jika aku tidak bersemangat, maka tulisanku tidak dimuat di surat
x L  71 nL  L kabar
x gab  71,8 n W  L Soal No. 101 – 104
nW 0,8 2 SBY
 
n L 1,2 3
TNI Ibu Ani
% selisih = 1
100%  20%
5
82. Kunci : C MENLU Michelle
x  155 ; n  20 ;  x  155  20  3100
Setelah terkoreksi:
 x  3100  198 189  3109 MENKOMINFO Obama
3190
x  155,45 KAPOLRI
20
83. Kunci : B
Harga buku I = 120% = 12juta
Harga buku II = 12juta – 20/100 . 12juta 101. D. Ibu Ani duduk di seberang Bapak MENKOMINFO
= 9,6 juta 102. B. Bapak MENLU dan Ibu Ani duduk berhadapan
84. Kunci : Bonus 103. D. Bapak MENKOMINFO duduk di antara Bapak Panglima TNI dan
Bapak MENLU
104. D. Bapak MENLU duduk di antara Bapak SBY dan Ibu Ani
Marketplace untuk Staners. Platform online yang menyediakan seluruh kebutuhan yang berkualitas untuk
(calon) mahasiswa, alumni, dan orang-orang yang mempunyai ketertarikan terhadap PKN STAN.
www.stanersstore.com @stanersstore stanersstore
Kunci TO Mingguan
Soal no. 104 – 110 132. Kunci : A
105. D. Baris 1: D, F; Baris 2: B, E; Baris 3: C, A; Baris 4: H, G Topik : Derivative
106. D. A, C, G, H For adalah preposisi dan diikuti oleh NOUN. Dependability dan
107. D. jendela G juga menghadap ke bulan. Dependence sama-sama NOUN tetapi maknanya berbeda.
108. D. F, C, E, G Dependability = bisa diandalkan
109. A. C dan G duduk di baris yang sama Dependence= kebergantungan
110. C. D dan F duduk di baris yang sama 133. Kunci : D
Soal no. 111 – 116 Topik : Preference
111. D. Sebuah manik-manik hijau berada di sebelah sebuah manik-manik Would prefer + TO infinitive + Rather Than + Kata Dasar.
kuning. 134. Kunci : B
112. B. manik-manik kelima Topik : Participle
113. D. II, III, dan IV The man arrested …..(The man who is arrested) = Lelaki yang
114. C. manik-manik kedua berwarna kuning ditahan.
115. D. I, II, dan III 135. Kunci : D
116. D. manik-manik ketujuh berwarna kuning Topik : Adjective Clause
117. C. Most of which pada pilihan (D) mengacu kepada most of the race
118. C. route.
119. B. 136. Kunci : A
120. C. Topik : Tenses
Salah satu pola tenses adalah:
TES BAHASA INGGRIS (TBI) While + past Continuous, past continuous
121. Kunci : E 137. Kunci : E
Topik : The use of ENOUGH Topik : Adverbial Clause
Kata “Enough” tidak boleh diikuti oleh adjective, karena Adjective Konjungsi yang tepat untuk menghubungkan dua kalimat pada soal
harus terletak didepan “enough”. Misal: Strong Enough. Pilihan selain adalah Nevertheless (Meskipun Demikian).
(C) semuanya adalah adjective. 138. Kunci : C
122. Kunci : D Topik : Appositive
Topik : Preposition + Gerund Appositive (keterangan tambahan) biasanya diapit oleh dua tanda
LOVE apabila bertindak sebagai noun (benda) membutuhkan koma. Kemudian appositive tidak bisa diawali oleh S + P, tidak bisa
preposisi FOR. Kemudian, preposisi harus diikuti oleh Gerund. diawali TO BE, dan bila diawali oleh WHICH harus diikuti oleh verb
123. Kunci : C atau To be.
Topik : Adverbial Clause 139. Kunci : D
Dalam soal kita melihat ada kata “THAT”. Artinya, kita harus Topik : Tenses/ Passive Voice
menggunakan kata SUCH atau SO. Bila ada keterangan waktu BY + waktu datang, maka yang digunakan
Pattern: Such + Adjective + NOUN + THAT … adalah pola future perfect tense. Jawaban (D) dipakai karena pola
SO + Adjective/adverb + That … kalimat adalah pasif.
124. Kunci : A 140. Kunci : B
Topik : Derivative Topik : Inverted Sentence
Strenuously adalah adverb dan diikuti oleh verb/ adjective. Kita LITTLE adalah kata yang bermakna negative, berarti harus diikuti oleh
menggunakan objected (verb 3) karena dalam soal ada auxiliary HAS. Auxiliary + Subject (jawaban antara B dan C). jawaban (C) salah
125. Kunci : D karena kata WHAT tidak bisa diikuti langsung oleh verb ING.
Topik : Modals 141. Kunci : A
Could HAVE + Verb 3/BEEN bermakna “Seharusnya Bisa” dan Topik : Participle
digunakan untuk sesuatu yang bertentangan dengan masa lalu. She Announcements POSTED = Announcements which are Posted
could have been more careful ..(Dia seharusnya bisa lebih hati- (pengumuman yang diposting).
hati..)artinya: Dia tidak hati-hati. 142. Kunci : B
126. Kunci : D Topik : Subjunctive
Topik : Inverted Sentences Dalam soal ada kita temukan kata IMPORTANCE yang asalnya
NOR adalah salah satu kata negative dan bila diikuti kalimat harus adalah IMPORTANT.
berpola inverse. Temukan jawaban yang diawali Auxiliary +Subject. Salah satu pola subjunctive adalah:
Pilihan (D) dipilih karena konteks kalimat adalah present. Important + THAT + Subject + Kata Dasar/ BE.
127. Kunci : B Misal: It is important that he be more careful.
Topik : Conjunction 143. Kunci : B
Salah Satu Pola kalimat majemuk: Topik : Participle
Kalimat (S + P) + KONJUNGSI + Kalimat (S+P) Bila kalimat diawali oleh participle, maka induk kalimat adalah subject
Then bukan konjungsi. During tidak bisa diikuti oleh S + P. Since yang dimaksud pada participle itu.
bila bermakna sejak harus diikuti oleh Past tense. Having been announced as one of the best colleges in Indonesia
128. Kunci : B (Setelah diumumkan sebagai salah satu kampus terbaik di Indonesia),
Topik : Subjunctive tentu yang menjadi subject harus nama Kampusnya. Karena itu
Wished + S + Had + Verb 3/ been jawaban (B) yang benar.
129. Kunci : A 144. Kunci : A
Topik : Derivative Topik : Derivative
Salah satu pola derivative adalah: Auxiliary (HAVE) + Adverb + verb (ARGUED).
TO BE + Adverb + Adjective (WAS + PRACTICALLY + Reasonable). Soal membutuhkan jawaban Adverb.
130. Kunci : A 145. Kunci : A
Topik : Gerund Topik : Noun Clause
FANCY + Gerund Kalimat mengandung dua predikat (HAS (setelah the president) dan
131. Kunci : D HAS (setelah widow). Karena itu ini pola noun clause. Noun clause
Topik : Modals dibentuk salah satunya dengan kata THAT + S + P dan bila berada
MUST + Have + Verb 3/been = pasti telah/pasti pernah.

Marketplace untuk Staners. Platform online yang menyediakan seluruh kebutuhan yang berkualitas untuk
(calon) mahasiswa, alumni, dan orang-orang yang mempunyai ketertarikan terhadap PKN STAN.
www.stanersstore.com @stanersstore stanersstore
Kunci TO Mingguan
diawal kalimat baru bertindak sebagai subject. Karena itu An harus diubah menjadi THE karena importance itu dimiliki oleh
membutuhkan predicate lagi. Project. Sama halnya dengan The door of the house …(Pintu rumah).
146. Kunci : B 160. Kunci : D
Topik : Uncountable Noun Topik : ---
Furniture adalah benda yang tidak bisa dihitung dan tidak bisa LATTER tidak memerlukan apostrophe („). In the latter operation yang
dijamakkan. benar.
147. Kunci : D 161. Kunci : C
Topik : Participle Topik : Preposition
WHEN/ IF/ THOUGH bisa langsung diikuti Verb3 bila bermakna Equal TO, bukan Equal as.
pasif. WHEN ASKED ….(Ketika ditanya). 162. Kunci : E
148. Kunci : D Topik : Derivative
Topik : Meaning Deficiencies adalah NOUN, berarti kata didepannya adalah Adjective.
Pilihan (D) paling tepat menyimpulkan pernyataan dalam soal. Nutritionally harus diubah menjadi nutritional.
149. Kunci : B 163. Kunci : C
Topik : Subjunctive Topik : Noun Clause
Bila Setelah WISH kita menggunakan Modals (Could/would/should) + WHAT (kata Tanya) Bila terletak ditengah kalimat tidak diikuti oleh
HAVE + verb3, maka meaning/fact nya harus dalam bentuk past Auxilary + Subject, tetapi subject predicate. Posisi IS harus diletakkan
tense. He could have lent some money…(Dia seharusnya bisa setelah the boy.
meminjamkan uang…) Artinya dia tidak bisa meminjamkan uang. 164. Kunci : A
150. Kunci : E Topik : Derivative
Topik : Conditional Sentence System adalah kata benda dan dimodifikasi oleh adjective. Pilihan (A)
Pola Conditional Tipe III: adalah adjective.
If + S + Had + Verb3,S +could/ would/should + have + verb3. 165. Kunci : B
151. Kunci : A Topik : Participle
Topik : Question Called = which is called (yang disebut).
How many people were invited to the wedding party? (Berapa banyak 166. Kunci : A
orang diundang …(pasif)). Topik : Derivative
152. Kunci : B Salah satu pola noun phrase adalah:
Topik : Plural/ Singular Form ADVERB + Verb3 (passive) + Noun.
A/AN harus diikuti benda tunggal. Phenomena adalah jamak. Specially trained priests (Pengkhotbah yang dilatih secara khusus).
Phenomenon tunggal. 167. Kunci : C
153. Kunci : C Topik : Derivative
Topik : Inverted Sentences Operate (verb) yang tidak masuk dalam golongan linking-verb. Karena
Complaining (mengeluh…). Kalau kita lihat kalimat secara seksama, itu harus dimodifikasi oleh adverb.
subject pada kalimat itu adalah the minister and several parents 168. Kunci : B
(subject jamak). Karena itu, WAS harus diubah menjadi WERE. Topik : Passive Voice
154. Kunci : D All judges, ……, were elected, not appointed. (Semua hakim, …..,
Topik : Derivative dipilih, bukan ditunjuk).
America harus diubah menjadi American atau America‟s karena diikuti 169. Kunci : B
benda..Misal American culture. 170. Kunci : B
155. Kunci : A 171. Kunci : B
Topik : Gerund 172. Kunci : D
WHY CAN + S + verb/be? Artinya, Study disitu harus dibuat menjadi 173. Kunci : B
subject. Kata kerja harus diubah menjadi gerund (studying) bila 174. Kunci : A
bertindak sebagai subject. 175. Kunci : D
156. Kunci : A 176. Kunci : D
Topik : --- 177. Kunci : D
Yang benar adalah Ten thousand dollars, bukan ten thousands of 178. Kunci : D
dollars. Berbeda kalau kata TEN dihilangkan, maka berubah menjadi 179. Kunci : C
thousands of dollars. 180. Kunci : A
157. Kunci : D
Topik : Parallelism
Ada kata alternative (Pilihan) dalam soal. Artinya harus salah satu
yang dipilih. Maka EITHER …OR yang dipakai.
158. Kunci : E
Topik : Participle
Written workshop (Pelatihan Ditulis) harus diubah menjadi writing
workshop (pelatihan menulis).
159. Kunci : A \
Topik : Article

Marketplace untuk Staners. Platform online yang menyediakan seluruh kebutuhan yang berkualitas untuk
(calon) mahasiswa, alumni, dan orang-orang yang mempunyai ketertarikan terhadap PKN STAN.
www.stanersstore.com @stanersstore stanersstore
SOAL TPA PRE AND POST

PRE TEST 16
MATERI KOMBINASI 3
(ARITMATIKA SOSIAL, PERBANDINGAN, JARAK, WAKTU, KECEPATAN, HIMPUNAN DAN PELUANG)

1. Dua mobil meninggalkan sebuah parkiran tersebut adalah Rp173.000, berapakan biaya
pada pukul 10.00, mobil yang satu pergi perbaikannya?
kearah timur dengan kecepatan 60 km/jam A. 22.000
dan mobil yang lainnya pergi kearah barat B. 27.000
dengan kecepatan 40 km/jam. Pukul berapa C. 35.000
mereka berjarak dua mobil itu 450 km ? D. 43.000
A. 13.30 E. 47.000
B. 14.00 5. Sebuah barang dijual dengan potongan
C. 14.30 pertama dan kedua berturut-turut 25%
D. 15.00 kemudian 20%. Bila pada tahun bersangkutan
E. 15.30 terjadi inflasi sehingga menaikan harga
2. Pada pukul 08.00 Fakhri berangkat dari seluruh barang, dan dengan menaikan harga
rumahnya menuju rumah Vina dengan 1
sebesar 33 % akan mengembalikan harga
3
kecepatan 25 km/jam. 30 menit kemudian barang tersebut pada harga awal, maka
Vina berangkat dari rumahnya menuju arah berapakah inflasi pada tahun tersebut?
Fakhri dengan kecepatan 15 km/jam. Bila A. 20%
jarak rumah Fakhri dan rumah Vina adalah B. 25%
72,5 km, maka pukul berapa mereka akan 2
C. 16 %
bertemu? 3

A. 09.00 D. 10%
B. 09.15 E. 50%
C. 09.30 6. Raga membel sebuah mobil dengan diskon
D. 09.45 30% namun harus dikenakan pajak dengan
E. 10.00 tarif 30%. Maka harga mobil menjadi...
3. Sebuah ruangan bila dicat oleh Alya A. 130%
membutuhkan waktu 5 jam sedangkan bila B. 100%
dicat oleh seorang tukang, hanya C. 91%
membutuhkan waktu 1 jam. Berapa waktu D. 70%
yang dibutuhkan untuk mencat sebuah E. 85%
ruangan bila dilakukan bersama? 7. Sebuah proyek diselesaikan oleh 12 pekerja
A. 30 menit dalam 18 hari. Maka berapakah tambahan
B. 40 menit pekerja jika proyek tersebut ingin diselesaikan
C. 50 menit dalam waktu 8 hari?
D. 60 menit A. 12
E. 75 menit B. 15
4. Sebuah toko elektronik menjual sebuah TV C. 18
dengan harga Rp275.000. Harga tersebut D. 21
termasuk untung penjualan sebesar 25%. E. 25
Sebelum dijual, TV tersebut sempat diperbaiki 8. Dalam sebuah kotak terdapat bola berwarna
dan keuntungan diambil dari biaya perbaikan biru dan merah dengan perbandingan 2:7. Bila
ditambah harga beli. Bila harga beli TV 6 bola biru ditambahkan dalam kotak

Marketplace untuk Staners. Platform online yang menyediakan seluruh kebutuhan yang berkualitas
untuk (calon) mahasiswa, alumni, dan orang-orang yang mempunyai ketertarikan terhadap PKN STAN.

www.stanersstore.com @stanersstore stanersstore


SOAL TPA PRE AND POST

tersebut, maka perbandingannya menjadi 1:2. A. 44.48%


Maka berapakah jumlah bola merah dalam B. 44,84%
kotak tersebut? C. 48,44%
A. 21 D. 48,48%
B. 24 E. 48,88%
C. 27 10.Berapa susunan angka yang bisa dibentuk dari
D. 28 angka 2, 3, 4, 5 namun diatas tiga ribu dan
E. 30 ganjil serta diperbolehkan adanya angka yang
9. Dalam suatu tes, peserta tes diberikan 2 jenis sama?
soal, yaitu tes potensi akademik (TPA) dan tes A. 72
bahasa inggris (TBI). Dari 60 ribu peserta tes, B. 80
sebanyak 31.235 menyukai TPA dan sebanyak C. 96
3476 menyukai keduanya. Maka berapa D. 108
persen peserta tes yang menyukai bahasa E. 120
inggris?

Marketplace untuk Staners. Platform online yang menyediakan seluruh kebutuhan yang berkualitas
untuk (calon) mahasiswa, alumni, dan orang-orang yang mempunyai ketertarikan terhadap PKN STAN.

www.stanersstore.com @stanersstore stanersstore


SOAL TPA PRE AND POST

POST TEST 16
MATERI KOMBINASI 3
(ARITMATIKA SOSIAL, PERBANDINGAN, JARAK, WAKTU, KECEPATAN, HIMPUNAN DAN PELUANG)

1. Perusahaan memiliki karyawan sebanyak tiga yang memberikan jawaban yang paling
kali jumlah karyawan tahun lalu. Jumlah benar, panjang dari berbagai potongan tadi?
karyawan awal tahun lalu sejumlah 29 orang, a. 1 meter, 1.5 meter, dan 2 meter
tetapi 3 orang keluar dan perusahaan b. 1 meter dan 1.5 meter
menerima karyawan sebanyak dua kali dari c. 0.5 meter, 1meter, dan 1.5 meter
jumlah karyawan yang keluar. Berapakah d. 1.5 meter dan 2 meter
jumlah karyawan tahun ini? e. Tidak ada jawaban yang benar
A. 96 orang 5. Beni dapat menyelesaikan suatu pekerjaan
B. 87 orang dalam satu jam dan Hani dapat
C. 98 orang menyelesaikan pekerjaan yang sama dalam
D. 85 orang 40 menit. Suatu hari mereka bekerja
E. 105 orang bersama-sama dalam 12 menit, kemudian
2. Seorang pedagang menjual dagangannya Beni pergi dan diteruskan oleh Hani
dengan harga Rp120.000,00; tetapi pembeli sendirian. Berapa lama Hani menyelesaikan
meminta diskon sebesar 15%. Apabila sisa pekerjaannya?
pedagang tersebut membeli barang a. 8 menit
dagangan hanya sebesar 72% dari harga jual b. 16 menit
sebelum diskon, berapa % keuntungan yang c. 20 menit
ia dapat jika mengabulkan permintaan d. 28 menit
pembeli? e. 30 menit
A. 15,294% 6. Satu orang juru masak dapat membuat 2 kue
B. 18,056% berukuran besar atau 35 kue berukuran kecil
C. 13,00% dalamwaktu 1 jam. Berapa orang juru masak
D. 16,667% yang dibutuhkan untuk membuat 20 kue
E. 18,45% berukuran besar dan 700 kue berukuran kecil
3. Berat sebuah dadu yang terbuat dari logam dalam waktu 3 jam?
adalah 6 kg. Berapakah berat sebuah dadu A. 10
yang juga terbuat dari logam yang sama B. 15
dengan panjang sisi 2 kali lebih besar? C. 20
A. 48 D. 25
B. 32 E. 30
C. 24 7. Seorang anak yang tingginya 150 cm difoto
D. 18 dalam ukuran kecil dengan skala 1 : 30,
E. 12 kemudian foto tersebut ukurannya
4. Sebuah pipa yang panjangnya 6 meter diperbesar dengan skala 4 : 1, maka tinggi
1 1 anak dalam foto yang terakhir adalah…
ditandai pada jarak bagian dan bagian.
4 3
Jika pipa tersebut dipotong pada bagian yang A. 100 cm
ditandai ini menjadi potongan-potongan B. 85 cm
terpisah, yang manakah dari jawaban berikut C. 45 cm
D. 20 cm
E. 18 cm

Marketplace untuk Staners. Platform online yang menyediakan seluruh kebutuhan yang berkualitas
untuk (calon) mahasiswa, alumni, dan orang-orang yang mempunyai ketertarikan terhadap PKN STAN.

www.stanersstore.com @stanersstore stanersstore


SOAL TPA PRE AND POST

a. 491,67 m
b. 800 m
8. Usia rata-rata sekelompok orang yang terdiri c. 525 m
dari Arsitek dan Guru adalah 30 tahun. Jika d. 833,33 m
usia rata-rata arsitek adalah 20 tahun dan e. 583 m
usia rata-rata guru adalah 45 tahun, 10. Seorang wisatawan berangkat dengan
berapakah perbandingan antara jumlah pesawat terbang dari Jakarta menuju
arsitek dan guru? Jayapura. Pesawat lepas landas dari bandara
a. 1:2 Jakarta pada pukul 20.00 waktu setempat
b. 1:1 dan mendarat di bandara Jayapura pada
c. 3:2 pukul 06.00 waktu setempat. Jika selama
d. 2:3 penerbangan tersebut pesawat singgah di
e. 2:1 bandara Surabaya dan Makassar masing-
9. Jarak kota A dan kota B adalah 1325 m. masing selama 30 menit. Berapa jamkah
Yenny ingin pergi ke kota A dari kota B lama seluruh penerbangan tersebut?
dengan mengendarai kendaraan dengan a. 9
kecepatan konstan 108 km/jam. Ternyata di b. 10
lain pihak setelah 2 ½ detik Yenny berangkat, c. 11
Xena mulai meninggalkan kota A untuk d. 12
menuju ke kota B dengan kecepatan konstan e. 13
216 km/jam. Berapakah jarak dari kota A
saat Xena dan Yenny berpapasan?

Marketplace untuk Staners. Platform online yang menyediakan seluruh kebutuhan yang berkualitas
untuk (calon) mahasiswa, alumni, dan orang-orang yang mempunyai ketertarikan terhadap PKN STAN.

www.stanersstore.com @stanersstore stanersstore


SOAL TPA PRE AND POST
PRE TEST 17
MATERI KOMBINASI 4
(GEOMETRI, PERSAMAAN, TEORI BILANGAN, DAN STATISTIKA)
1 1 1
1. Berapakah nilai dari + + B. 𝜋𝑥 2
2𝑥4 4𝑥6 4
1 1 1
+ .......+ = ......? C. 𝜋𝑥 2
6𝑥8 18 𝑥 20 8
1
A. 9 / 20 D. 𝜋𝑥 2
16
B. 1 / 20 1
E. 𝜋𝑥 2
C. 1 / 40 64

D. 9 / 40 6. Sebuah kertas dengan luas 60 cm2 di bagi dua,


E. 19 / 40 sehingga luas yang satu 2/3 dari luas yang lain.
Berapakah luas salah satu potongan kertas itu
2. Jika 0,3 + 0,08 = 𝑎+ 𝑏, maka 1/a +
?
1/b = . . . .
A. 15
A. 25
B. 20
B. 20
C. 24
C. 15
D. 30
D. 10
E. 35
E. 5
3 2
7. Diketahui sistem persamaan Linear sebagai
3. Diketahui persamaan 𝑚 − 𝑛 = 17. Maka berikut: a+3b-2c =13 , 2a-b-c =4, 2a+b-2c=8.
2 3
nilai dari 32n – 72m adalah.... Maka nilai dari b+c-2a adalah
A. 816 a. -4
B. -816 b. 17
C. 861 c. 9
D. -861 d. 13
E. 168 e. -13
4. Enam buah lingkaran masing-masing berjari- 8. Jika 40x-24y = 120, maka nilai dari 3y-5x
jari 3 disusun seperti gambar, maka t adalah?
adalah…… A. 15
A. 3 + 6√3 cm3 B. -15
B. 4 + 6√3 cm3 C. 30
C. 6 + 6√3 cm3 t D. -30
3
D. 7 + 6√3 cm E. 45
E. 9 + 6√3 cm3
5. Sebuah persegi memiliki sisi x cm. Didalam
persegi tersebut terdapat 4 buah lingkaran
sehingga memenuhi ruang yang ada. Dari 4
buah lingkaran tersebut, terbentuk sebuah
persegi yang berasal dari 4 titik pusat
lingkaran. Bila dalam persegi kecil yang
terbentuk tersebut digambar sebuah lingkaran
9. Suatu larutan mengandung 40% cuka,
sehingga sisi-sisinya saling menyinggung,
dicampur dengan larutan cuka kandungan
berapakan luas lingkaran tersebut?
1 25% dan menjadi campuran baru dengan
A. 𝜋𝑥 2
2

Marketplace untuk Staners. Platform online yang menyediakan seluruh kebutuhan yang berkualitas
untuk (calon) mahasiswa, alumni, dan orang-orang yang mempunyai ketertarikan terhadap PKN STAN.

www.stanersstore.com @stanersstore stanersstore


SOAL TPA PRE AND POST
kandungan cuka 35%. Berapa bagian larutan
yang dicampurkan ?
A. ¼
B. 1/3
C. ½
D. 2/3
E. 1
10.Sebuah keluarga mempunyai 5 orang anak
yang masing-masing berusia (x-3), (x+1), (2x-
2), (2x), dan (2x+5). Jika rata-rata umur
mereka sekarang adalah 16 tahun, berapakah
rata-rata usia mereka 10 tahun yang akan
datang ?
A. 24
B. 26
C. 27,67
D. 24,33
E. 30

Marketplace untuk Staners. Platform online yang menyediakan seluruh kebutuhan yang berkualitas
untuk (calon) mahasiswa, alumni, dan orang-orang yang mempunyai ketertarikan terhadap PKN STAN.

www.stanersstore.com @stanersstore stanersstore


SOAL TPA PRE AND POST
POST TEST 17
MATERI KOMBINASI 4
(GEOMETRI, PERSAMAAN, TEORI BILANGAN, DAN STATISTIKA)

1. 6,90 ÷ 3 10 × 27% × 3,75% = ⋯. e. 96


A. 3,194 6. Sebuah bak penampung air dengan luas alas
B. 3,149 sebesar 45 dm2, akan diisi air dengan
C. 3,214 kecepatan 15L/menit. Tinggi dari bak
D. 3,241 tersebut adalah 150 cm dan pada 100 cm
E. 3,354 dari dasar terdapat lubang yang membuat
air keluar 3L/menitnya. Berapakah waktu
2. Jika𝑝 = 5625, 𝑞 = (15 + 20)2 dan
untuk mengisi bak penampung air tersebut?
𝑟 = 0,53 maka 𝑝 + 𝑞 + 𝑟 = ⋯.
A. 52,5 menit
A. 1225,125
B. 50,25 menit
B. 1290,125
C. 48,75 menit
C. 1300,125
D. 45 menit
D. 1425,000
E. 42,35 menit
E. 1435,125
7. Sebuah bola dengan volume 38.808 cm3
3. 12 – 22 + 32 – 42 + 52 – 62 + ...... + 20092 –
dimasukan kedalam kubus dengan volume
20102 + 20112 =
125.000 cm3. Maka berapa banyak bola
a. 2023066
maksimal yang bisa dimasukan kedalam
b. 2033056
kubus tersebut?
c. 2023055
A. 1
d. 2033066
B. 2
e. 2023065
C. 3
4. Nilai rata-rata UTS Ekonomi 20 Anak kelas
D. 4
Akuntansi A adalah 77. Tidak semua dari
E. 5
mereka mendapat nilai yang sama. Manakah
8. Sebuah kubus ABCD.EFGH memiliki panjang
pernyataan yang selalu benar?
4 cm. Titik X dan Y berturut-turut merupakan
a. Setengah dari mereka mendapat nilai di
bawah 77. titik tengah dari CG dan DH. Maka jarak
b. Ada yang mendapat nilai 77. antara titik B dan bidang XYHG adalah...
c. Paling sedikit ada 1 siswa yang nilainya A. 26
diatas 77 B. 27
d. Ada yang mendapat nilai 78
C. 28
e. Tidak ada jawaban yang benar
5. Dari 40 siswa ada 30 orang yang mendapat D. 29
nilai tidak kurang dari 48 tetapi tidak lebih E. 30
6 8
dari 76. Jika nilai yang bisa diraih adalah 9. Jika 𝑥 adalah dari 87,5% dan 𝑦 = × 5%
7 0,4
bilangan bulat dari 0 sampai 100, maka nilai maka …
rata – rata yang tidak mungkin dari ke-40 A. 𝑥 > 𝑦
siswa tersebut adalah ... B. 𝑥 < 𝑦
a. 35 C. 𝑥 = 𝑦
b. 47 D. Hubungan 𝑥 dan 𝑦 tidak dapat
c. 77 ditentukan
d. 82 E. 2x = 3y

Marketplace untuk Staners. Platform online yang menyediakan seluruh kebutuhan yang berkualitas
untuk (calon) mahasiswa, alumni, dan orang-orang yang mempunyai ketertarikan terhadap PKN STAN.

www.stanersstore.com @stanersstore stanersstore


SOAL TPA PRE AND POST
C. p = q
10. Jika p + q = 28 dan p.q = 196, maka … D. Hubungan p dan q tidak dapat ditentukan
A. p > q E. 2p =3q
B. p < q
MINITEST 1
NON –KUANTITATIF
SINONIM merupakan aset perusahaan? Kalau ya, mengapa
1. MAHLIGAI nilainya tidak tercantum dalam neraca.
A. Anggun Status aset bagi sumber daya manusia
B. Pesona didasarkan atas konsep ilmu ekonomi yang
C. Kediaman raja menggolongkan sumber daya manusia sebagai
D. Pernikahan salah satu faktor produksi yang mempunyai
ANTONIM potensi memberi manfaat dan keuntungan di
2. UTOPIS masa depan.namun dari sisi akuntansi sumber
A. Praktis daya manusia bukanlah aset perusahaan.
B. Prgamatis Menurut prinsip akuntansi status aset
C. Empiris perusahaan bagi suatu harta diberikan bila harta
D. Statis itu merupakan milik perusahaan.
3. BOTANI Walaupun tenaga kerja yang dipekerjakan
A. Hewani perusahaan itu bukan milik perusahaan yang
B. Nabati bersangkutan, perusahaan itu mempunyai rasa
C. Calyx memiliki, buktinya personil dibina, dilatih, dan
D. Epifit ditingkatkan kualitasnya dengan penyediaan
ANALOGI biaya yang cukup besar. Tak heran jika tiba-tiba
4. SEL : SITOLOGI pegawai yang sudah dibina dan dilatih itu pindah
A. bangsa : ethnologi ke perusahaan lain maka perusahaan yang
B. serangga : etologi bersangkutan merasa kehilangan, merasa hak
C. manusia : sosiologi miliknya dirampas atau dibajak oleh orang lain.
D. kesehatan : patologi Dari sinilah timbul istilah yang populer
5. OVERTURE : OPERA “pembajakan” personal perbankan oleh bank
A. Epilog : sinema yang baru tumbuh.
B. Konserto : piano 6. Masalah yang dibicarakan paragraf pertama
C. Kata pengantar : buku mengenai…
D. Catatan kaki : paragraf A. aset nasional
Bacaan 1 B. gedung peralatan merupakan aset
Akhir-akhir ini sering didengar ucapan nasional
pejabat yang mengatakan bahwa perlunya C. sumber daya manusia merupakan aset
pengembangan sumber daya manusia itu adalah perusahaan
aset nasional. Gedung, peralatan dan sebagainya D. gedung, peralatan, dan manusia adalah
adalah aset perusahaan yang masing-masing aset perusahaan
nilainya dapat ditemukan dalam neraca 7. Sumber daya manusia sebagai aset
perusahaan yang bersangkutan, tetapi nilai perusahaan didasarkan atas…
sumber daya manusia tidak tercantum dalam A. prinsip akuntansi
neraca perusahaan. Hal ini akan menimbulkan B. prinsip ilmu ekonomi
pertanyaan benarkah sumber daya manusaia itu C. faktor produksi

Marketplace untuk Staners. Platform online yang menyediakan seluruh kebutuhan yang berkualitas
untuk (calon) mahasiswa, alumni, dan orang-orang yang mempunyai ketertarikan terhadap PKN STAN.

www.stanersstore.com @stanersstore stanersstore


SOAL TPA PRE AND POST

D. potensinya memberi manfaat dimasa A. Kamar B di sebelah kamar C


depan B. Kamar E di sebelah kamar A
C. Kamar B ada di lantai 1
D. Kamar D ada di lantai 2
8. Paradoksal atas sumber daya manusia sebagai 12.Jika D dan F dua-duanya tinggal di lantai 1,
aset terdapat pada.. manakah diantara pernyataan ini yang pasti
A. paragraf 2 benar?
B. paragraf 1 A. F tinggal dikamar 1
C. paragraf 3 B. B tinggal dikamar 5
D. paragraf 1 dan 3 C. Kamar C tepat di atas kamar D
9. Makna kata neraca pada paragraf satu D. Kamar B tepat diatas kamar A
dibawah ini semua salah kecuali… 13.Jika kamar B tepat disebelah kamar F,
A. timbangan manakah diantara pernyataan ini yang pasti
B. dafar posisi harta,utang,dan modal benar?
C. pertimbangan politik dan kekuatan A. Kamar F di sebelah kamar C
D. piranti untuk menimbang B. Kamar C di sebelah kamar B
C. Kamar E di sebelah kamar A
Enam orang karyawan pemasaran PT D. Kamar D tepat diatas kamar C
Cantabile yaitu A, B, C, D, E, dan F melakukan
perjalanan dinas ke luar kota dan menginap di Enam mahasiswa yaitu A, B, C, D, E, F masing-
motel. Tiga kamar yang mempunyai ukuran yang masing akan presentasi di depan kelas. Masing-
sama, saling bersebelahan di lantai pertama. Tiga masing hanya akan maju ke depan kelas satu kali
kamar lainnya di lantai dua, masing-masing tepat saja dan sesuai dengan urutan-urtan sebagai
diatas kamar yang ada di lantai satu. Kamar- berikut:
kamar lantai pertama diberi nomor 1, 2, dan 3  C akan maju pada urutan keempat atau
dan tepat di lantai atasnya masing-masing diberi terakhir
nomor 4, 5, dan 6. Saat membagi kamar mereka  D akan menjadi pembicara ketiga setelah A
menetapkan aturan sebagai berikut:  B akan maju sebelum C
 Satu kamar hanya boleh diisi satu orang  F akan maju sebelum D
 A akan menempati kamar nomor 2
 B tidak boleh bersebelahan dengan kamar 14. Dari urutan-urutan berikut ini, manakah yang
E memnuhi persyaratan tersebut?
 Kamar A tidak boleh bersebelahan dengan A. E, F, A, C, B, D
kamar C B. A, F, C, D, B, E
 Kamar D tidak boleh berada langsung di C. E, F, A, B, D, C
atas atau di bawah kamar E D. B, A, F, E, D, C
10. Manakah diantara pernyataan ini yang pasti 15. Manakah dari pernyataan berikut ini yang
benar? benar?
A. Kamar D ada di lantai 1 A. D akan maju pada urutan ketiga
B. Kamar C ada di lantai 2 B. A akan maju pada urutan pertama
C. Kamar F di sebelah kamar A C. F akan maju pada urutan keenam
D. Kamar A tepat dibawah kamar E D. E akan maju pada urutan keenam
11.Jika kamar C tepat diatas kamar F, manakah 16. Apabila B maju pada urutan kelima, manakah
diantara pernyataan ini yang pasti benar? dari pernyataan berikut ini yang benar?

Marketplace untuk Staners. Platform online yang menyediakan seluruh kebutuhan yang berkualitas
untuk (calon) mahasiswa, alumni, dan orang-orang yang mempunyai ketertarikan terhadap PKN STAN.

www.stanersstore.com @stanersstore stanersstore


SOAL TPA PRE AND POST

A. B maju pada urutan pertama Semua yang ada pada kedudukan utama
B. E maju pada urutan kedua mendatangkan uang.
C. A maju pada urutan ketiga A. Sementara artis tidak pada kedudukan
D. D maju pada urutan keempat utama
B. Sementara yang mendatangkan uang
17. Apabila E maju persis sebelum C, ada berapa adalah artis
urutan yang berbeda keenam presentasi C. Sementara yang pada kedudukan utama
tersebut dapat dibuat? adalah artis
A. 2 D. Sementara artis tidak mendatangkan
B. 3 uang
C. 4 22.Semua jenis ikan bernafas dengan insang.
D. 5 Ikan paus bernafas dengan paru-paru.
Enam mahasiswa yaitu A, B, C, D, E, F masing- A. Sementara ikan bernafas dengan paru-
masing akan presentasi di depan kelas. Masing- paru
masing hanya akan maju ke depan kelas satu kali B. Semua jenis ikan bernafas dengan insang
saja dan sesuai dengan urutan-urtan sebagai C. Semua ikan paus termasuk jenis ikan
berikut: D. Semua ikan paus bukan termasuk jenis
 C akan maju pada urutan keempat atau ikan
terakhir 23.Semua karyawan adalah pembaca.
 D akan menjadi pembicara ketiga setelah A Beberapa karyawan adalah penulis.
 B akan maju sebelum C A. Beberapa penulis adalah pembaca
 F akan maju sebelum D B. Beberapa pembaca bukan karyawan
18. Dari urutan-urutan berikut ini, manakah yang C. Semua penulis adalah pembaca
memnuhi persyaratan tersebut? D. Beberapa karyawan bukan penulis
A. E, F, A, C, B, D 24.Amin adalah tamatan SMP. Ia bekerja. Sebagai
B. A, F, C, D, B, E buruh di pabrik. Banyak buruh di pabrik yang
C. E, F, A, B, D, C malas. Gito adalah teman Amin. Jadi :
D. B, A, F, E, D, C A. Amin itu malas
19. Manakah dari pernyataan berikut ini yang B. Gito itu malas
benar? C. Teman-teman Amin semuanya malas.
A. D akan maju pada urutan ketiga D. Mungkin Gito tamatan SMP seperti Amin.
B. A akan maju pada urutan pertama 25.Semua B adalah sebagian A. Semua A adalah
C. F akan maju pada urutan keenam bukan C.
D. E akan maju pada urutan keenam A. Sebagian A adalah bukan C
20. Apabila B maju pada urutan kelima, manakah B. Sebagian B adalah C
dari pernyataan berikut ini yang benar? C. Tidak ada C yang merupakan B
A. B maju pada urutan pertama D. Semua B adalah sebagian C
B. E maju pada urutan kedua 26.Semua mahasiswa harus membayar SPP
C. A maju pada urutan ketiga Sementara mahasiswa mengikuti kuliah
D. D maju pada urutan keempat kedokteran.
Pilihlahkemungkinan kesimpulan yang paling tepat! A. Sementara mahasiswa yang membayar
21. Sementara artis ada yang pada kedudukan SPP tidak mengikuti kuliah kedokteran.
pertama atau utama. B. Sementara mahasiswa yang membayar
SPP mengikuti kuliah kedokteran.

Marketplace untuk Staners. Platform online yang menyediakan seluruh kebutuhan yang berkualitas
untuk (calon) mahasiswa, alumni, dan orang-orang yang mempunyai ketertarikan terhadap PKN STAN.

www.stanersstore.com @stanersstore stanersstore


SOAL TPA PRE AND POST

C. Sementara mahasiswa yang tidak


membayar SPP mengikuti kuliah
kedokteran. C D
D. Sementara mahasiswa yang mengikuti 29.
kuliah kedokteran tidak membayar SPP
E.
27.

A B C D
A B C D 30.

28.

A B C D

A B

Marketplace untuk Staners. Platform online yang menyediakan seluruh kebutuhan yang berkualitas
untuk (calon) mahasiswa, alumni, dan orang-orang yang mempunyai ketertarikan terhadap PKN STAN.

www.stanersstore.com @stanersstore stanersstore


SOAL TPA PRE AND POST

MINITEST 2
KUANTITATIF
5. Pada suatu acara, panitia telah menyusun
kursi tempat duduk sebanyak 70 buah. Setiap
1. 2+ 2+ 2+ 2+ …=⋯
baris berisi kursi dengan jumlah yang sama.
Setelah diperiksa, susunan kursi harus diubah
A. -2
sebagai berikut: setiap baris harus dikurangi
B. 0
dengan 2 kursi, akan tetapi jumlah baris
C. 1
harus ditambah 4. Dengan perubahan ini
D. 2
jumlah seluruh kursi tetap 70 buah dan setiap
2. Pada gambar di bawah, O adalah sudut pusat
baris berisi kursi dengan jumlah yang sama.
lingkaran dan ABP adalah sebuah garis lurus.
Berapakah jumlah baris sebelum ada
Besar sudut AOC adalah…
perubahan?
A. 5
B. 7
C. 10
D. 14
6. X= 4YZ2 jika Z dikalilipatkan 2 kali maka harga
X menjadi berapa kali harga semula?
A. 1
A. 108°
B. 4
B. 120°
C. 8
C. 132°
D. 16
D. 144°
7. Sebuah bak mandi sisi luarnya berukuran alas
3. Agus berangkat ke kampus yang berjarak 25
95cmx75 cm dan tinggi 105 cm dengan tebal
km dengan naik sepeda motor yang
tembok 5 cm. berapakah luas permukaan sisi
berkecepatan 20 km/jam, 5 menit kemudian
dalam bak meter?
Budi menyusul agus naik mobil berkecepatan
A. 40.725
30 km/jam. mereka akan bertemu pada jarak
B. 47.850
berapa dari kampus?
C. 37.025
A. 5 km
D. 42.550
B. 10 km
8. Sejenis sel dapar membelah diri njadi dua
C. 15 km
setiap 15 detik. Berapakah jumlah sel setelah
D. 20 km
15 menit jika mula-mula hanya ada dua sel?
4. Dalam suatu pesta yang dihadiri 35 orang,
A. 120
banyaknya pengunjung pria dibanding
B. 260
pengunjung wanita adalah 5 : 2. Jika setelah X
C. 230
pria pergi, perbandingan tersebut menjadi 2 :
D. 261
1. Berapa nilai X (pria yang pergi)?
9. Sebuah survei dilakukan terhadap x siswa
A. 3
sekolah musik. Dari siswa tersebut diketahui
B. 5
bahwa 72 siswa dapat memainkan gitar klasik,
C. 25
47 siswa dapat memainkan biola, dan 21 siswa
D. 35
dapat memainkan keduanya. Dari keterangan

Marketplace untuk Staners. Platform online yang menyediakan seluruh kebutuhan yang berkualitas
untuk (calon) mahasiswa, alumni, dan orang-orang yang mempunyai ketertarikan terhadap PKN STAN.

www.stanersstore.com @stanersstore stanersstore


SOAL TPA PRE AND POST

di atas dapat disimpulkan bahwa x tidak memiliki saudara sama dengan jumlah
mungkin bernilai ... saudarinya, sedangkan setiap anak
A. 96 perempuan memiliki saudara laki-laki dua kali
B. 98 lebih banyak dari jumlah saudarinya. Banyak
C. 100 anak dalam keluarga itu adalah…
D. 102 a. 5
10.Seorang pengecat telah mengecat sepertiga b. 7
tembok berbentu persegi panjang yang c. 9
tingginya 10 meter. Apabila ia selesai d. 11
1
mengecat bagian tembok lain seluas 75 meter 15. Jika 𝑎−5 = 1024 dan 𝑏 4 = , maka …
81
persegi, maka ia akan menyelesaikan ¾ dari 1
a. 𝑎+𝑏 =
pekerjaannya. Berapa panjang tembok 12
1
tersebut ? b. 𝑎−𝑏 =
12
A. 18 meter 1
c. 𝑎. 𝑏 =
12
B. 9 meter 4
d. 𝑎: 𝑏 =
C. 10 meter 3

D. 12 meter
11. Seorang pedagang mencampur a kg kedelai 16. Jika −3 ≤ 𝑝 ≤ 7 dan −2 < 𝑞 < 6, maka
seharga b rupiah per kg dengan c kg kacang a. 13 ≤ 𝑝2 + 𝑞 2 ≤ 85
seharga d rupiah per kg. Berapakah harga jual b. 13 ≤ 𝑝2 + 𝑞 2 < 85
c. 0 ≤ 𝑝2 + 𝑞 2 ≤ 85
kacang campuran jika ia berharap
d. 0 ≤ 𝑝2 + 𝑞 2 < 85
memperoleh laba 1.000 rupiah per kg?
A. (ab + cd)/a + c) + 1.000 17. Kran A jika dinyalakan akan memenuhi
B. (b + d)/(a + c) + 1.000 sebuah bak dalam waktu 5,25 jam. Kran B
C. (b + d + 1.000)/(a + c) akan memenuhi bak yang sama dalam
D. Tidak dapat ditentukan. waktu 3,5 jam. Berapa waktu yang
12. Jika A = keliling lingkarang berjari-jari 2 cm dibutuhkan jika 2 kran tersebut dinyalakan
dan B = keliling segi delapan beraturan bersama?
dengan sisi 2 cm, maka… A. 4 jam
a. A < B B. 4,5 jam
b. A > B C. 2,1 jam
c. A = B D. 1,75 jam
d. Hubungan A dan B tidak dapat 18.Untuk membuat donat selama satu bulan,
ditentukan dibutuhkan bahan baku: 50 kg tepung
13. Volume kerucut terbesar yang ada di dalam terigu (Rp 4.500 per kg), 30 kg gula pasir
kubus dengan sisi 𝑥 satuan adalah … satuan. (Rp 7.100 per kg), 40 kg cokelat (Rp3.500
𝜋 3
a. 𝑥 per kg), dan 7 kg margarine (Rp 18.000 per
12
𝜋 3
b. 𝑥 kg). Berapa besar keuntungan dalam satu
6
𝜋 3 bulan jika besar modal adalah 65% dari
c. 𝑥
4 total penjualan dalam sebulan?
𝜋 3
d. 𝑥 A. Rp 316.000
3
B. Rp 418.500
14.Suatu keluarga memiliki beberapa anak laki- C. Rp 560.000
laki dan perempuan. Setiap anak laki-laki D. Rp 585.500

Marketplace untuk Staners. Platform online yang menyediakan seluruh kebutuhan yang berkualitas
untuk (calon) mahasiswa, alumni, dan orang-orang yang mempunyai ketertarikan terhadap PKN STAN.

www.stanersstore.com @stanersstore stanersstore


SOAL TPA PRE AND POST

19.Valentino bekerja sebagai customer C. ¼


service part time dengan upah mingguan. D. 1/6
Jam kerjanya adalah dimulai pada pukul
08.15 dan berakhir pada pukul 16.15. Ia 22. Tarif iklan borongan di suatu surat kabar lokal
mendapat upah sebesar Rp 312.500 per adalah Rp. 250 per baris untuk hari pertama;
minggu dan uang lembur Rp 23.000 per Rp. 150 per baris Untuk lima hari berikutnya
jam. Minggu ini, Valentino bekerja hingga dan Rp. 100 per baris untuk hari-hari
pukul 17.15 pada hari Selasa dan hingga berikutnya. Apabila seseorang membayar Rp.
pukul 18.15 pada hari Kamis. Berapa 6.000 untuk iklan tiga baris, berapa harikah
jumlah upah yang diterima Valentino iklan tersebut dipasang?
minggu ini? A. 60
A. Rp 391.500 B. 16
B. Rp 381.500 C. 15
C. Rp 371.500 D. 10
D. Rp 361.500 23. Imron bekerja pada sebuah perusahaan
20. Sebuah perusahaan penyewaan mobil bagian penjualan barang dengan gaji
mengenakan sewa Rp. 350.000 setiap minimum bulanan Rp1.500.000,00. Selain
pemakaian 24 jam untuk 72 jam pertama, dari gaji tersebut, ia juga dapat memperoleh
ditambah Rp. 50.000 untuk setiap 6 jam komisi 25 % dari penjualan barang di atas
atau bagian dari enam jam setelah 72 Rp90.000.000,00 selama sebulan. Jika ia ingin
jam pertama. Jika sebuah mobil diambil memperoleh uang Rp2.500.000,00 pada
pada pukul 08.00 hari Senin dan bulan itu, maka ia harus dapat melakukan
dikembalikan pada Kamis pukul 21.45 penjualan barang dalam bulan tersebut
minggu yang sama, berapakah jumlah paling sedikit …
sewa yang harus dibayar ? A. Rp104.000.000,00
A. Rp. 1.200.000 B. Rp96.250.000,00
B. Rp. 450.000 C. Rp92.500.000,00
C. Rp. 500.000 D. Rp94.000.000,00
D. Rp. 1.150.000 24.Jika a²b/c = 1, maka pernyataan yang benar
21.Pada sebuah pabrik, jumlah produk rusak adalah …
pada bulan Januari adalah 7 persen dari a. b ≠ c
seluruh produk yang dihasilkan, dan pada b. a² < 0
bulan februari adalah 8 persen dri seluruh c. b ≠ 0
produk yang dihasilkan, jika persentase d. c positif
produk rusak pada kedua bulan tersebut
digabungkan adalah 7,8 persen, 25.Jika 2m – n adalah 70% dari L dan 40% dari L
berapakah jumlah produk yang dihasilkan adalah 1/3m, tentukan berapa persen 2n dari
dalam bulan Januari dibandingkan dengan L?
jumlah produk yang dihasilkan dalam A. 170%
bulan Februari ? B. 340%
A. 6/5 C. 120%
B. 1/8 D. 350%

Marketplace untuk Staners. Platform online yang menyediakan seluruh kebutuhan yang berkualitas
untuk (calon) mahasiswa, alumni, dan orang-orang yang mempunyai ketertarikan terhadap PKN STAN.

www.stanersstore.com @stanersstore stanersstore


KUNCI TPA PRE AND POST

PRE TEST 16
MATERI KOMBINASI 3
(ARITMATIKA SOSIAL, PERBANDINGAN, JARAK, WAKTU, KECEPATAN, HIMPUNAN DAN PELUANG)

1. C. 14.30 (4/3)X = 100


450 / (60+40) = 4,5 jam X=75  75-60=15
10.00 + 4,5 jam = 14.30 15/60 x 100% = 25%
2. E. 10.00 6. C. 91%
25 km x 0,5 jam = 12,5 km 100 − 30 = 70%
(jaraktempuhawal) 130% 𝑥 70% = 91%
72,5 – 12,5 = 60 km 7. B. 15
(jarakketikaVinaberangkat) 18.12 = 8.x
60 km / (25+15 km/jam) = 1,5 jam X = 27
08.00 + 30 menit + 1,5 jam = 10.00 27 – 12 = 15
3. C. 50 menit 8. D. 28
1 1 1+5 6 Cara paling
= +1= =
𝑡 5 5 5 mudahadalahdenganmemasukanopsi.
5 Masukanangka 28 keperbandingan
𝑡 = 𝑗𝑎𝑚 = 50 𝑚𝑒𝑛𝑖𝑡
6 2 : 7  8 : 28
4. E. 47.000
8 + 6 = 14 maka14 : 28 = 1 : 2 (cocok)
100
𝑥275.000 = 220.000 9. A. 44,48%
125
220.000 − 173.000 = 47.000 (31.235-3476)/50.000 x100% = 44,48%
5. B. 25% menyukaibahasainggris ≠
Dimisalkanharganya 100 hanyamenyukaibahasainggris
100 x 20% x 25% = 60 10. C. 96
X + (1/3)X=100 3 x 4 x 4 x 2 = 96

Marketplace untuk Staners. Platform online yang menyediakan seluruh kebutuhan yang berkualitas
untuk (calon) mahasiswa, alumni, dan orang-orang yang mempunyai ketertarikan terhadap PKN STAN.

www.stanersstore.com @stanersstore stanersstore


KUNCI TPA PRE AND POST

POST TEST 16
MATERI KOMBINASI 3
(ARITMATIKA SOSIAL, PERBANDINGAN, JARAK, WAKTU, KECEPATAN, HIMPUNAN DAN PELUANG)

1. A. 96 orang 1
Tinggianakdalamfotosemula × 150 cm = 5
30
Jumlahkaryawantahunlalu = 29 karyawan di cm
awaltahun – 3 karyawanygkeluar + 6 4
Tinggianakdalamfototerakhir × 5 = 20 cm
karyawanygditerima = 32 1

Jumlahkaryawantahunini = 3 x 8. C
30 − 20 𝐴 45 − 30 𝐺
Jumlahkaryawantahunlalu = 3 x 32 = 96 =
2. B. 18,056% 30 30
10𝐴 15𝐺
Hargasetelahdiskon = Rp120.000,00 x 85% = =
30 30
Rp102.000,00 𝐴 15 3
Hargabeli = Rp120.000,00 x 72% = = =
𝐺 10 2
Rp86.400,00 9. D
102.000−86.400
% keuntungan = × 100% =
86.400
18,056%
3. Volume dadu1 : Volume dadu2 = (s)3 : (2s)3 = 1
:8
Rasioberatproporsionaldenganrasio volume. tx = ty
Jadi, berat dadu2 = 6 x 8 kg = 48 (A) 𝑠 𝑠
( )x = ( )y
𝑣 𝑣
4. C 𝑛 (1250−𝑛)
Perpotongan ¼ bagian, makamasing-masing =
60 30
1,5 meter. n = 2(1250 – n)
Perpotongan 1/3 bagian, makamasingmasing n = 2500 – 2n
2 meter. 3n = 2500
n = 833,33 m
10. A
5. C 20.00 s.d. 06.00 = 10 jam
Dalam satu menit Beni dapat menyelesaikan 10-2(1/2 jam) = 9 jam
1/60 bagian. Hani: 1/40 bagian. Bersama-
sama dalam 12 menit: 12/60 + 12/40 = ½
bagian. Sisa 1-1/2 = ½ bagian. Hani: satu
bagian dalam 40 menit, maka ½ bagian = 20
menit

6. A. 10 orang
Untukmembuat 20 kuebesardiperlukan 10
orang jurumasakdalam 1 jam
Untukmembuat 700 kuekecildiperlukan 20
orang jurumasakdalam 1 jam
Karenawaktu yang
disediakanuntukmembuatkueadalah 3 jam,
makahanyadiperlukan 10 orang jurumasak.
7. D. 20 cm

Marketplace untuk Staners. Platform online yang menyediakan seluruh kebutuhan yang berkualitas
untuk (calon) mahasiswa, alumni, dan orang-orang yang mempunyai ketertarikan terhadap PKN STAN.

www.stanersstore.com @stanersstore stanersstore


KUNCI TPA PRE AND POST
PRE TEST 17
MATERI KOMBINASI 4
(GEOMETRI, PERSAMAAN, TEORI BILANGAN, DAN STATISTIKA)
1 1 1 1
1. D. + + + .......+ =
2𝑥4 4𝑥6 6𝑥8 18 𝑥 20
1 1 1 1 1
( + ...+ ) 5. D. 𝜋𝑥 2
4 1𝑥2 2𝑥3 9 𝑥 10 16
1 1 1 9
Makajawabannyaadalah − = Jari-jaridarilingkarang yang keciladalah ¼
4 1 10 40
darisisipersegi
2. C. 0,3 + 0,08 = 0,2 + 0,1 + 2 0,2 𝑥 0,1 =
𝜋𝑟 2 = 𝜋(¼𝑥)2
0,2 + 0,1
6. C.
Maka a = 0,2dan b= 0,1 ; 1/a + 1/b = 1/0,2 +
La = 2/3 Lb
1/0,1 = 5 + 10 = 15
3 2 La + Lb = 60
3. 𝑚 − 𝑛 = 17 (kalikan dengan 6) maka
2 3 5/3 Lb = 60
9m – 4n = 17 Lb = 36 dan La = 24
Maka 9m = 102 + 4n 7. A
32n – 72m = 8 (4n – 9m) 8. B
= 8 {4n – (102 + 4n)} (40x-24y = 120) :8 kemudian di kalikan -1
= 8 (4n – 102 – 4n) 9. B
= 8 (-102) (1 – n ) x 40 +n x 25 = 35  n= 1/3
= - 816 (B) 10. A.
4.

Marketplace untuk Staners. Platform online yang menyediakan seluruh kebutuhan yang berkualitas
untuk (calon) mahasiswa, alumni, dan orang-orang yang mempunyai ketertarikan terhadap PKN STAN.

www.stanersstore.com @stanersstore stanersstore


KUNCI TPA PRE AND POST

POST TEST 17
MATERI KOMBINASI 4
(GEOMETRI, PERSAMAAN, PEMODELAN MATEMATIKA, TEORI BILANGAN, DAN STATISTIKA)

1. A. 3,194 7. A. 1
6,90 ÷ 3 10 × 27% × 3,75% (4/3)πr3 = 38.808
= 6,90 ÷ 0,081 × 3,75% r3 = 9261
= 3,194 r = 21
2. 𝑝 = 5625 = 75 d = 42
𝑞 = 15 + 20 2 = 1225 s3 = 125.000
𝑟 = 0,53 = 0,125 s = 50
𝑝 + 𝑞 + 𝑟 = 1300,125 (𝐶) makamaksimal bola yang bisamasukadalah 1
3. A karena S kubushanya 50cm dan diameter
12-22 = -3 = -1-2 bola 42cm
32-42 = -7 = -3-4 8. D. 29
-1-2-3-4-............-2009-2010+20112 Jaraktitiktengahbidang XYHG ke alas adalah
-(1+2+3+4...............+2009+2010)+20112 3 cm.
2010 Jarak B ketitiktengah CD adalah:
-(2011 x )+20112
2
2011(-1005+2011) 22 + 42 = 20
2011(1006) = 2023066 Makajarak B ke XYHG adalah:
4. C ( Tipesoalsepertiinijawabannyahanyadua) 2
32 + 20
Paling sedikitsatudiatas rata-rata
atausatudibawah rata-rata. 9. 𝑥 = 0,75; 𝑦 = 1 maka 𝑥 < 𝑦 (B)
10. C. p = q di mana p = q = 14
5. A
6. C. 48,75
450/15 + 225/12 = 48,75

Marketplace untuk Staners. Platform online yang menyediakan seluruh kebutuhan yang berkualitas
untuk (calon) mahasiswa, alumni, dan orang-orang yang mempunyai ketertarikan terhadap PKN STAN.

www.stanersstore.com @stanersstore stanersstore


KUNCI TPA PRE AND POST

MINITEST 1
NON –KUANTITATIF
1. C. Kediaman raja 16. Kunci : D
2. C. Empiris 17. Kunci : D
3. B. Nabati 18. Kunci : D
4. A 19. Kunci : B
5. C 20. Kunci : D
6. A 21. Kunci : B
7. C 22. Kunci : D
8. D 23. Kunci : A
9. B 24. Kunci : D
10. Kunci : B 25. Kunci : C
11. Kunci : D 26. Kunci :B
12. Kunci : A 27. Kunci: C
13. Kunci : B 28. Kunci: A
14. Kunci : D 29. Kunci: A
30. Kunci: B
15. Kunci : B
5. Kunci: C
MINITEST 2
KUANTITATIF Jlh.
Baris Isi
1. Kunci: D Kursi
Awal 10 7 70
2  2  2       x kuadratkan
Perubahan 14 5 70
x2 – x – 2 = 0
6. Kunci: B
(x + 1)(x – 2) = 0  x = –1 atau x = 2
x = 4 y z2 z diduakalilipatkan z = 2z
2. Kunci: D
x2 = 4y (2z)2 = 4 y z2 . 4
ABC = 1080 7. Kunci: C
ALC = 720 Sisidalam: p = 85 ; l = 65 ; t = 105
AOC = 1440 Luas sisidalam = Lalas + LDinding= 37.025
3. Kunci: A 8. Kunci: D
SA = SB VAtA = VB .tB Tiap 15 detik = 2 kali lipat
25(t + 5) = 30 .t U1 = 2
t = 10 menit = 1/6 jam 15 menit = 15  60  60KaliPembel ahan
SB = VB .tB = 30 . 1/6 = 5 jam 15
25(t + 5) = 30 .t Banyaksel = 21 . 260 = 261
4. Kunci: B 9. Kunci : A
L + W = 35 Minimal (72+37) - 21 = 98 sehingga yang
L : W = 5 : 2  L = 25 ; W = 10 tidakmungkin 96
Lx 2 25  x 10. Kunci : A
  2
W 1 10 11. Kunci : A
x=5 𝑎𝑏 + 𝑐𝑑
+ 1.000
𝑎+𝑐
12. Kunci: A
Luas segi n beraturan>luaslingkaran

Marketplace untuk Staners. Platform online yang menyediakan seluruh kebutuhan yang berkualitas
untuk (calon) mahasiswa, alumni, dan orang-orang yang mempunyai ketertarikan terhadap PKN STAN.

www.stanersstore.com @stanersstore stanersstore


KUNCI TPA PRE AND POST

13. Kunci: A Lembur = 23.000/jam


rk = ½ x Selasa : 9 jam lembur = 1 jam
tk = x Kamis : 10 jam lembur = 2 jam (+)
1  Gaji = 312.500 + 23.000 . 3 = 381.500
Vol k  r 2  t   x 3
3 12
20. Kunci : A
14. Kunci: B
Selasa – kamis 350 * 3 = 1050
L > P  dan berselisih 1
Kamismalam (50 * 3) = 150
L = 4 dan P = 3
1.200
15. Kunci: C
21. Kunci : C
a–5 = 1024 a = ¼
7….. 7,8 ….. 8
b4 = 1/81b = 1/3 Maka:
0,8 0,2
1
a b  4 : 1
2
16. Kunci: D Sehingga ¼
–3  P  7 ; –2 < Q < 6 22. Kunci : B
0  P2 49 Haripertama : 750
0  Q2< 36 (+) Harikedua : 2.250
Sisa : 3.000
0  P2 + Q2 < 36
Sehinggasisamerupakan 10 hari, ditambah 10 + 5
17. Kunci: C
1 21 1 7 + 1 = 16 hari
tA  5 Jam  Jam  t B  3 Jam  Jam
4 4 2 2 23. Kunci: D
1 1 1
   t G  2,1Jam 2.500.000=1.500.000+1.000.000
tG t A t B

18. Kunci: A Omset = 90 juta + (100/25) 1.000.000


Biaya: Tepung = 225.000 = 94.000.000
Gula = 213.000 24. Kunci: C
Coklat = 140.000 Karna hasilbagiada, b tidakboleh 0
Margarin = 126.000 (+) 25. Kunci: B
Biaya = 704.000 2m – n= 70/100 l
Modal = 64% Penjualan 2m – n = 7/12 m
Penjualan = 1.100.000 n=17/12 m
Laba = 396.000 40/100 l = 1/3 m
19. Kunci: B l = 10/12 m
Jam kerja = 8 jam 2n/l (100%)=2(17/12m)/10/12 m (100%)
Upah = 312.500/minggu =340%

Marketplace untuk Staners. Platform online yang menyediakan seluruh kebutuhan yang berkualitas
untuk (calon) mahasiswa, alumni, dan orang-orang yang mempunyai ketertarikan terhadap PKN STAN.

www.stanersstore.com @stanersstore stanersstore


SOAL TRY-OUT 4

UJIAN SARINGAN MASUK


PROGRAM DIPLOMA I DAN DIPLOMA III KEUANGAN
POLITEKNIK KEUANGAN NEGARA STAN TAHUN 2016

BAGIAN PERTAMA-TES POTENSI AKADEMIK (NOMOR 1 s.d 120)

Untuk soal nomor 1-6 pilihlah kata atau frasa yang mempuyai arti sama atau arti paling
dekat dengan kata yang dicetak dengan huruf kapital yang terdapat di atas pilihan-pilihan
tersebut.
1. Demisioner 4. BRIGADE
A. keadaan tanpa kekuasaan A. perintang yang dibuat untuk
B. Pelaksanaan suatu misi menghambat kemajuan musuh
C. Pengosongan suatu lokasi atau untuk melindungi kubu-kubu
D. Pengisian survei pertahanan terhadap serangan
E. Pembubaran partai politik musuh
2. Etatisme B. satuan berseragam dengan tugas
A. Cairan dalam kelenjar prostat khusus
B. Persewaan tempat semacam real C. sebuah blok bahan yang dapat
estate dibakar yang digunakan sebagai
C. Paham yang lebih mementingkan bahan bakar untuk memulai dan
bidang ekonomi daripada bidang mempertahankan nyala api
sosial D. pangkat perwira tinggi peringkat
D. Paham yang lebih mementingkan keempat dalam kepolisian
negara daripada rakyatnya E. jaringan tiang pada daun
E. Pasal mengenai hukum indonesia 5. ARBITER
yg harus disamakan dengan A. pembelian dan penjualan secara
hukum di belanda simultan atas barang yang sama di
3. Aneksasi dalam dua pasar atau lebih dengan
A. Gangguan makan yang ditandai harapan akan memperoleh laba
dengan rasa takut yang berlebihan dari perbedaan harganya
terhadap peningkatan berat badan B. warga dusun atau kampung kecil
B. Rasa nyeri yg amat sangat yg yang merupakan penduduk local
hilang-timbul di daerah lambung pada daerah tersebut
atau sekitarnya C. Hal yang menjadi perdebatan para
C. Pengambilan dengan paksa tanah ahli
negara lain untuk disatukan D. orang yang disepakati oleh dua
dengan tanah negara sendiri belah pihak yang bersengketa
D. Proses pembiusan dengan kadar untuk memberikan keputusan
alkohol yang tinggi yang akan ditaati oleh kedua belah
E. semburan sumber minyak dan uap pihak
panas

Marketplace untuk Staners. Platform online yang menyediakan seluruh kebutuhan yang berkualitas
untuk (calon) mahasiswa, alumni, dan orang-orang yang mempunyai ketertarikan terhadap PKN STAN.

www.stanersstore.com @stanersstore stanersstore


SOAL TRY-OUT 4

E. pengawal khusus seseorang C. Hasil dari seseorang yang


seperti presiden, pejabat tinggi melakukan sesuatu seperti yang
negara diperbuat orang lain dan
6. DUPLIK sebagainya
A. jawaban kedua dari terdakwa atau D. Ruang medis yang bersebelahan
pembela sebagai jawaban atas E. bentuk pemerintahan yang
replik berkedaulatan rakyat dan dikepalai
B. Melakukan sesuatu seperti yang oleh seorang presiden
diperbuat orang lain dan
sebagainya
Untuk soal nomor 7-11 pilihlah kata atau frasa yang mempuyai padanan kata atau padanan
pengertian yang paling dekat dengan kata yang dicetak dengan huruf kapital yang terdapat
di atas pilihan-pilihan tersebut.
7. Protesis D. Menjenguk
A. Ketidaksetujuan E. Surat
B. Protein 10. Testamen
C. buatan A. percobaan
D. Diskriminasi B. wasiat
E. Purba C. sampel
8. Feodalisme D. Buah zakar
A. Fasisme E. lingkungan
B. Demokrasi 11. Kolokium
C. Federal A. Peninggalan
D. Bermanfaat B. Satuan
E. Layak C. horizontal
9. Ulayat D. vertikal
A. Rakyat jelata E. Seminar
B. wilayah
C. serangga
Untuk soal nomor 12-16 pilihlah kata atau frasa yang mempuyai lawan kata atau lawan
pengertian yang paling dekat dengan kata yang dicetak dengan huruf kapital yang terdapat
di atas pilihan-pilihan tersebut.
12. integritas C. Ragu
A. Individualis D. Persegi
B. Sosialis E. Tidak beraturan
C. Hipokrit 14. Prominen
D. Otonom A. Ungkap
E. ketidakadilan B. Berani
13. Skeptis C. Takut
A. Lukis D. Biasa
B. yakin E. Keras
Marketplace untuk Staners. Platform online yang menyediakan seluruh kebutuhan yang berkualitas
untuk (calon) mahasiswa, alumni, dan orang-orang yang mempunyai ketertarikan terhadap PKN STAN.

www.stanersstore.com @stanersstore stanersstore


SOAL TRY-OUT 4

15. Takzim A. Binokuler


A. Lancang B. Persegi
B. Tidak biasa C. Keagamaan
C. Halus D. Demokrat
D. Karangan E. Aristokrat
E. Liar
16. Sekuler

Untuk soal nomor 17-25 pilihlah kata atau frasa yang mempuyai padanan hubungan atau
hubungan yang paling serupa dengan kata yang dicetak dengan huruf kapital yang terdapat
di atas pilihan-pilihan tersebut.
17. GEMPA : RITCHER= … D. Tepung : beras
A. Ombak : knot E. Tanah : pegunungan
B. Jarak : dinamo 22. ANALGESIK: NYERI = …
C. Obat : dosis A. Ketahanan : Perkakas
D. Suhu : fahrenheit B. Improvisasi : Musik
E. Banjir : air C. Pelumas : Gesekan
18. FIKTIF : FAKTA =… D. Kepercayaan : Tipuan
A. Dagelan – Sandiwara E. Masalah : Pening
B. dongeng : peristiwa 23. SUARA : .…= …. : ARTI
C. dugaan : rekam A. Homofon - sinonim
D. data : estimasi B. Dengar - pikir
E. rencana : projeksi C. Mulut - otak
19. ILMU : GENIUS = … D. Kata benda - kata sifat
A. Banjir Air E. Ucap - Rasa
B. malas juara kelas 24. APEL : RUANG TERBUKA:
C. gula diabetes PENCOKLATAN
D. darah anemia A. Siswa : kampus : pembelajaran
E. mata tunanetra B. Air : lemari es : pencairan
20. FILM: SKENARIO = … C. Ikan : penggorengan kompor
A. Drama : Panggung D. Lilin : terbakar : penyubliman
E. Laptop : servis : harddisk
B. Sandiwara : Plot
25. MURID : BUKU : PERPUSTAKAAN
C. Teater : Acara
A. Anak : kelereng: rumah
D. Resital : Tinjauan
B. Nasabah : uang : bank
E. Musikal : Pantomim
C. Orang tua : anak : ibu
21. Kertas : Pohon = …
D. Pembeli : makanan : penjual
A. Besi : baja
E. Dosen : buku : kuliah
B. Es : lemari es
C. Balon : udara

Marketplace untuk Staners. Platform online yang menyediakan seluruh kebutuhan yang berkualitas
untuk (calon) mahasiswa, alumni, dan orang-orang yang mempunyai ketertarikan terhadap PKN STAN.

www.stanersstore.com @stanersstore stanersstore


SOAL TRY-OUT 4

Untuk soal nomor 26-35, bacalah tiap bacaan dengan seksama, kemudian jawablah
pertanyaan yang berkaitan dengan bacaan tersebut dengan memilih jawaban A, B, C, D ,
atau E.
BACAAN 1 berikut untuk soal nomor 26-30

Plastik dan polimer banyak digunakan masyarakat. Hampir setiap produk menggunakan
plastik sebagai kemasan atau bahan dasar. Setiap tahun sekitar 100 juta ton plastik
diproduksi dunia untuk digunakan di berbagai sektor industri. Kira-kira sebesar itulah
sampah plastik yang dihasilkan setiap tahun.

Material plastik banyak digunakan karena mempunyai sifat unggul, seperti ringan,
transparan, tahan air, serta harganya relatif murah dan terjangkau oleh semua kalangan
masyarakat. Sebaliknya, plastik masih mempunyai sifat kurang menguntungkan. Plastik
tidak mudah hancur karena lingkungan, baik oleh cuaca hujan dan panas matahari
maupun mikroba yang hidup dalam tanah. Beranjak dari permasalahan itu, muncul
pemikiran menggunakan bahan alternatif untuk membuat material polimer yang ramah
lingkungan (biodegradable, Red).

Di beberapa negara maju, bahan plastik biodegradable sudah diproduksi secara


komersial, seperti poli hidroksi alkanoat (PHA), poli e-kaprolakton (PCL), poli butilen
suksinat (PBS), dan poli asam laktat (PLA). Namun, kebanyakan bahan baku untuk bahan
plastik biodegradable masih menggunakan sumber daya alam yang tidak diperbarui
(non-renewable resources, Red) dan tidak hemat energi. Dengan demikian, tentu
pengembangan bahan plastik biodegradable yang memanfaatkan bahan-bahan alam
terbarui (renewable resources, Red) sangat diharapkan.

Beranjak dari pemikiran itu, Feris Firdaus, Sri Mulyaningsih, dan Endang Darmawan dari
DPPM (Direktorat Penelitian dan Pengabdian Masyarakat, Red) Universitas Islam
Indonesia (UII) Jogjakarta meneliti plastik kemasan yang ramah lingkungan dan dari
renewable resources. Riset yang berlangsung sejak awal 2006 itu adalah riset
pengembangan bahan plastik baru yang dapat hancur dan terurai dalam lingkungan.
Dengan kata lain, ini merupakan salah satu alternatif memecahkan masalah penanganan
sampah plastik.

"Plastik biodegradable dari pati singkong dan khitosan ini menjadi salah satu alternatif
bahan pembungkus. Selain ramah lingkungan karena mudah terurai, juga memiliki
karakteristik awet dan tahan hingga bulan ke-3 dari pemakaian," tandas Feris, peneliti
muda bidang kimia material dan komposit andalan DPPM UII itu.

Penelitian yang bertujuan mensintesis komposit pati-khitosan dan membentuk film


plastik biodegradable itu pada akhir 2006 lalu juga berhasil mendapat dana dari
Marketplace untuk Staners. Platform online yang menyediakan seluruh kebutuhan yang berkualitas
untuk (calon) mahasiswa, alumni, dan orang-orang yang mempunyai ketertarikan terhadap PKN STAN.

www.stanersstore.com @stanersstore stanersstore


SOAL TRY-OUT 4

Menristek untuk pengembangan penelitiannya. Dalam penelitian ini, film plastik


biodegradable diartikan sebagai film yang dapat didaur ulang dan dihancurkan secara
alami. Aman, Mudah Didaur Ulang

Pati merupakan biopolimer karbohidrat yang dapat terdegradasi secara mudah di alam
dan bersifat dapat diperbarui. Pati sendiri memiliki batasan bervariasi terkait dengan
kelarutan dalam air. Lapisan tipis dari pati dapat dengan mudah rusak. Untuk
meningkatkan karakteristik, biasanya pati dicampur biopolimer yang bersifat hidrofobik
atau bahan tahan air. Salah satu biopolimer hidrofobik yang direkomendasikan adalah
khitosan yang dapat disintesis dari limbah cangkang udang dan crustacea lainnya.
Khitosan direkomendasikan sebagai biomaterial berpotensi tinggi untuk dikompositkan
dengan pati atau amilum sebagai bahan utama pembuatan komposit pati-khitosan.
Khitosan merupakan senyawa yang tidak larut dalam air, larutan basa kuat, sedikit larut
dalam HCl dan HNO3, dan H3PO4, dan tidak larut dalam H2SO4.

Selain itu, khitosan tidak beracun, mudah mengalami biodegradasi, dan bersifat
polielektrolitik. Karakteristik lain khitosan adalah dapat dengan mudah berinteraksi
dengan zat-zat organik lain, seperti protein dan lemak. Karena itu, khitosan relatif lebih
banyak digunakan pada berbagai bidang industri terapan dan industri farmasi dan
kesehatan.

26. Keunggulan singkong sebagai bahan B. Karena mudah terurai


baku plastik adalah, kecuali… C. Mudah berinteraksi dengan zat
A. Terdegradasi dengan mudah organic lain
B. Aman D. Campuran yang mudah didapatkan
C. Mudah di daur ulang 29. Berikut ini karakteristik khitonan,
D. Ramah lingkuangan kecuali …
E. Larut dalam air A. Tidak beracun
27. Judul yang tepat untuk bacaan diatas B. Dapat disintesis dengan udang
adalah… C. Larut dalam air
A. Bahan Plastik Ramah Lingkungan D. Bahan utama komposit pati khotan
B. Plastik dan Polimer banyak E. Mengalami biodegradasi
digunakan masyarakat 30. Apa yang dimaksud dengan
C. Pengembangan bahan baku plastic bioegradable…
D. Manfaat Plastik A. Tidak dapat terdaur ulang
E. Singkong sebagai Bahan pembuat B. Ramah lingkungan
plastik C. Mudah terurai
28. Apakah yang menghubungkan D. Modern
Khitonan dan industry terarapan… E. Konservatif
A. Karena sifat tidak beracunnya

Marketplace untuk Staners. Platform online yang menyediakan seluruh kebutuhan yang berkualitas
untuk (calon) mahasiswa, alumni, dan orang-orang yang mempunyai ketertarikan terhadap PKN STAN.

www.stanersstore.com @stanersstore stanersstore


SOAL TRY-OUT 4

BACAAN 2 berikut untuk soal nomor 31-35

Tanaman hias umumnya ditanam di luar ruangan. Namun ternyata ada juga tanaman
hias yang dapat ditanam di dalam ruangan. Bukan sekedar sebagai penghias ruangan
belaka, beberapa tumbuhan dalam ruangan ini ternyata mampu membersihkan udara,
menyerap polutan dan gas-gas berbahaya.

Adalah National Aeronautics and Space Administration (NASA) dan Associated


Landscape Contractors of America (ALCA) yang telah melakukan penelitian terhadap
beberapa jenis tumbuhan. Hasilnya, beberapa tanaman hias dalam ruangan ternyata
mampu menyerap polutan dan gas-gas berbahaya seperti benzena, xylene,
formaldehida, xilena, nitrogen oksida dan berbagai bahan kimia lain yang hadir di udara.
Sehingga udara dalam ruangan bisa menjadi bersih dan sehat bagi penghuninya.
Diantaranya adalah Lidah Buaya (Aloe vera). Tumbuhan yang dimanfaatkan sebagai
penyubur rambut, penyembuh luka dan mulai dikembangkan sebagai bahan industri
kosmetik, farmasi, dan makanan-minuman kesehatan ini ternyata mempunyai
kemampuan sebagai penyerap polutan. Aloe vera mampu menetralisir racun benzena,
formaldehida (formalin), dan berbagai bahan kimia lainnya. Yang kedua adalah Spider
Plant (Chlorophytum comosum). Tanaman hias yang berasal dari Afrika dan kini tersebar
ke segala penjuru dunia ini mempunyai tinggi mencapai 60 cm. Sebagai tanaman hias,
tumbuhan ini dapat ditanam dalam pot dalam ruangan. Sebagai pembersih udara
tumbuhan ini mampu menyerap benzena, formaldehida, karbon monoksida dan xilena,
bahan kimia pada industri kulit, karet dan pencetakan.

Yang ketiga adalah tanaman Gerber daisy. Di Indonesia lebih dikenal sebagai Herbras.
Tumbuhan asal Afrika Selatan ini mempunyai bunga warna warni dan kerap dijadikan
bunga potong bersama mawar, anyelir, seruni, dan tulip. Dapat tumbuh baik di dalam
pot dan di dalam ruangan. Tanaman ini efektif menghilangkan trichloroethylene, Cocok
diletakkan di kamar mandi, ruang mencuci atau kamar tidur. Yang keempat, Lidah
mertua atau snake plant merupakan tanaman penyerap racun dengan bentuk khas dan
mudah dikenali. Berbagai jenis racun yang mampu diserap oleh Sansevieria antara lain
karbonmonoksida, nikotin, benzene, formaldehyde, trichloroethylene, hingga dioksin.
Bahkan hanya dengan 5 helai daun sansevieria dewasa mampu menyerap dan
membersihkan ruangan seluas 100 m3 dari berbagai jenis polutan. Selanjutnya Sirih
gading disebut juga golden pothos atau devil’s ivy adalah tanaman berbunga dari
keluarga Araceae. Tanaman ini berasal dari Australia, Malenesia (termasuk Indonesia),
Indochina, Jepang, China, dan India. Sehingga bisa dikatakan ini merupakan tumbuhan
asli Indonesia. Meskipun tanaman empifit dan semi-merambat namun baik dalam
menjadi pembersih udara dalam ruangan dan menyerap racun formaldehida dan
berbagai polutan lainnya.

Marketplace untuk Staners. Platform online yang menyediakan seluruh kebutuhan yang berkualitas
untuk (calon) mahasiswa, alumni, dan orang-orang yang mempunyai ketertarikan terhadap PKN STAN.

www.stanersstore.com @stanersstore stanersstore


SOAL TRY-OUT 4

31. Hal yang dapat disimpulkan dari B. Pengaruh Tanaman Hias pada
bacaan diatas adalah… Kesehatan
C. Tanaman Hias Pembersih Udara
A. NASA dan ALCA menemukan Dalam Ruangan
bahwa tanaman hias dalam D. Berbagai Manfaat Tanaman Hias
ruangan ternyata mampu
E. Keunggulan Tanaman Hias
menyerap polutan dan gas-gas
berbahaya 34. Peryataan yang tidak sesuai dengan
B. Tanaman hias Indonesia dapat peryataan yang ada didalam bacaan
dijadikan pemberseih ruangan, adalah…
menyerap polutan, dan gas A. Lidah buaya merupakan tanaman
berbahaya hias yang biasa digunakan dalam
C. Selain untuk hiasan, tanaman hias bidang kecantikan
dalam ruangan ternyata mampu B. Lima helai daun Snake plant
menyerap polutan dan gas-gas dewasa mampu menyerap dan
membersihkan ruangan seluas 100
berbahaya
m3 dari berbagai jenis polutan
D. NASA dan ALCA kembali C. Sirih gading adalah tumbuhan asal
menemukan fakta baru mengenai Australia yang merupakan tanaman
tanaman hias dalam ruangan. empifit dan semi-merambat namun
E. Tanaman hias dalam ruangan baik dalam menjadi pembersih
sangat dibutuhkan masyarakat udara dalam ruangan dan
mengingat tingkat pencemaran menyerap racun formaldehida dan
udara yang semakin meningkat tiap berbagai polutan lainnya
tahunnya. D. Tanaman gerber daisy berbunga
32. Terdapat beberapa jenis tanaman hias E. Herbras merupakan tanaman asli
yang dapat digunakan sebagai Indonesia yang dapat diletakkan
pembersih udara sebagaimana dalam pot dan merupakan salah
tercantum dalam artikel tersebut, satu tanaman hias ruangan yang
kecuali… dapat membersihkan udara,
A. Lidah mertua menyerap polutan, dan gas-gas
B. Chlorophytum comosum berbahaya.
C. Gerber daisy 35. Diantara tanaman hias tersebut ada
D. English ivy yang dapat menyerap racun, kecuali…
E. golden pothos A. Lidah buaya D. Spider
33. Judul bacaan yang paling tepat plant
adalah… B. Lidah mertua E. Snake
A. Tanaman Hias di Indonesia plant
C. Sirih gading
Untuk soal 36-80 pilihlah jawaban yang paling tepat!
36. Jika 54 + 37 = 909, maka A. 809
93 + 74 =.. B. 811

Marketplace untuk Staners. Platform online yang menyediakan seluruh kebutuhan yang berkualitas
untuk (calon) mahasiswa, alumni, dan orang-orang yang mempunyai ketertarikan terhadap PKN STAN.

www.stanersstore.com @stanersstore stanersstore


SOAL TRY-OUT 4

C. 825 A. 5
D. 833 B. 15
E. 847 C. 25
3 9 D. 35
37. 8𝑚 − 12𝑛 − 𝑛 + 𝑚 = ⋯
8 7
2 E. 45
A. (520𝑚 − 651𝑛) 0,8 4,5 0,3
56
520 651 42. Jika diketahui × : ×
6,4 0,9 0,9
B. ( 𝑛− 𝑚)
56 56 0,6
65 83 = 𝑚, maka nilai dari 𝑚
C. 8( 𝑚 − 𝑛) 0,36
7 8
520 651 adalah…
D. 𝑚− 𝑛 A. 1,125
56 56
245𝑚 522𝑚
E. − B. 2,125
45 45
1 C. 1,115
38. Jika diketahui = 0,75 maka nilai
𝑚
1 D. 11,15
adalah… E. 21,15
5𝑚 −7
1
A. 43. Jika a=5 dan b=2, maka nilai dari
3
1 a ³ - 3a ²b + 3ab² - b ³ =....…
B. −
3
A. 16
C. 3
B. 27
D. -3
C. 36
E. 1
D. 45
39. Diketahui f: R−→ R dan g: R−→ R
E. 49
ditentukan oleh 𝑔 𝑥 = 2𝑥 − 126
1 1
44. Jika diketahui 𝑚3 𝑛7 = 43904 dan
dan (𝑓(𝑔))(𝑥) = ( 𝑥)2 − 𝑥 + 𝑚𝑛 = 14, maka nilai 𝑚 = ⋯
56 4
5, maka tentukan 𝑓 322 ! A. 3
A. 35 B. 4
B. 40 C. 5
C. -40 D. 6
D. -35 E. 7
E. -30 45. jika x1 dan x2 merupakan akar dari
21 26 14 7
40. Jika diketahui 𝑚 = + + + , persamaan 32x + 33-2x - 28 = 0, maka
99 76 29 16
maka nilai bulat yang mendekati tentukanlah jumlah kedua akar
𝑚+1=⋯ tersebut
17 1
A. A.
30 2
47 3
B. B.
30 4
77 C. 1
C.
30 3
D.
D. 30 2
E. 33 E. 2
41. Jika 𝑚=
2 2
𝑛, 𝑜 = 𝑛, 2𝑚𝑛𝑜 = 46. Jika diketahui 𝑎 merupakan selisih
25 5
jumlah kelereng adimas dan adel,
15625
sedangkan 𝑏 adalah selisih jumlah
, maka nilai 𝑚 adalah…

Marketplace untuk Staners. Platform online yang menyediakan seluruh kebutuhan yang berkualitas
untuk (calon) mahasiswa, alumni, dan orang-orang yang mempunyai ketertarikan terhadap PKN STAN.

www.stanersstore.com @stanersstore stanersstore


SOAL TRY-OUT 4

kelereng mereka ketika adel telah E. hubungan antara x dan y tak dapat
memberikan 5 buah kelerengnya ditentukan
𝑥
kepada adimas, maka… 51. suatu pecahan. Jika𝑥ditambah 10
𝑦
A. 𝑎 > 𝑏 dan 𝑦ditambah 45 maka hasilnya
B. 𝑎 < 𝑏 2
adalah − . Jika x dan y dikuadratkan,
C. 𝑎 = 𝑏 9
D. 3𝑎 = 2𝑏 maka hasilnya adalah…
E. Hubungan tidak dapat ditentukan A. x > y
47. Apabila x dan y adalah volume dua B. x < y
buah bola dengan jari-jari masing- C. x = y
masing 6 dan 9. Maka : D. 𝑥 = 𝑦 2
A. x > y E. hubungan antara x dan y tak dapat
B. x = y ditentukan
C. x < y 52. Jika 𝑥adalah volume kubus dengan
D. hubungan x dan y tidak dapat diagonal ruang 10 3, dan 𝑦 adalah
ditentukan volume limas segitiga dengan sisi 15
E. semua jawaban benar dan tinggi 7
48. jika x= 1 ² + 2 ² + 3² + 4 ² + 5 ² + A. 𝑥 < 𝑦
.............+ 9 ² B. 𝑥 > 𝑦
y= 1+2+3+4+5+ ....+29, maka C. 𝑥 = 𝑦
A. x > y D. 𝑥 ≥ 𝑦
B. x < y E. Hubungan x dan y tidak dapat
C. x=y ditentukan
D. x < y, x=y
53. Jika
E. hubungan x dan y tidak dapat
ditentukan dan y = 4-2 + 4-3 maka…
49. Jika x + y = 100 dan x/y = ¼, maka… A. x > y
A. x > y B. x < y
B. x < y C. x = y
C. 𝑥 + 𝑦 = 0 D. 𝑥 = 𝑦 2
D. X=y E. hubungan antara x dan y tak
E. X –y = 0 dapat ditentukan
50. Jika 𝑥 = jumlah semua bilangan prima 54. Jika x adalah bilangan asli yang habis
di bawah 50 dan𝑦 = jumlah dua dibagi 3 dan y bilangan bulat kelipatan
puluh bilangan asli pertama yang 4 dengan8 < 𝑥 < 11, dan dengan
habis dibagi 7 maka … 10 < 𝑦 < 15, maka ....
A. x > y A. x > y
B. x < y B. x < y
C. x = y C. x = y
D. 3𝑥 = 2,5𝑦 D. 3𝑥 = 4𝑦
E. hubungan antara x dan y tak dapat
ditentukan

Marketplace untuk Staners. Platform online yang menyediakan seluruh kebutuhan yang berkualitas
untuk (calon) mahasiswa, alumni, dan orang-orang yang mempunyai ketertarikan terhadap PKN STAN.

www.stanersstore.com @stanersstore stanersstore


SOAL TRY-OUT 4

55. Jika Y% dari 80 adalah 32, dan 32 % keuntungan/kerugian jika dibandingan


dari 80 adalah x,… dengan harga swalayan?
A. x > y A. 1,5%
B. x < y B. 1,75%
C. x = y C. 2%
D. 3𝑥 = 𝑦 D. 2,1%
E. hubungan antara x dan y tak E. 3%
dapat ditentukan 59. Sebuah tangga berjalan (eskalator)
56. Seorang mahasiswi yang sedang menghubungkan lantai 1 dan lantai 2.
berlibur ingin menyimpan mainan Agar cepat sampai, Agus dan Jaka
kubiknya ke dalam sebuah kardus. menapaki anak-anak tangga eskalator
Satu kubik Neni mempunyai panjang yang sedang bergerak. Agus dan Jaka
sisi 6 cm. sedangkan kasrdus yang masing-masing melangkah sebanyak
ditemukannya berukuran 30 cm x 42 25 anak tangga dan 9 anak tangga
cm x 13 cm. berapa buah kubik yang untuk sampai ke lantai 2. Waktu yang
dapat disimpan Neni bila ia digunakan Jaka untuk sampai ke lantai
mempunyai 2 buah kardus? 2 sama dengan dua kali waktu yang
A. 75 diperlukan Agus. Dalam keadaan
B. 76 eskalator terhenti, berapa banyakkah
C. 150 anak tangga yang menunjukkan jarak
D. 152 lantai 1 ke lantai 2 ? ….
E. 2730 A. 39
57. Tessi memiliki persamaan 𝑎 + 𝑏 = B. 40
7, 𝑎 + 𝑐 = 13, 𝑏 + 𝑐 = 18, maka jika C. 41
Tesi menjumlahkan ketiga bilangan D. 42
tersebut hasilnya adalah… E. 43
A. 19 60. Imran dan sembilan sepupunya
B. 20 sedang bermain sambil belajar.
C. 21 Mereka menuliskan bilangan 1 sampai
D. 22 dengan 100 di atas kertas lipat. Satu
E. 23 kertas lipat bertuliskan satu bilangan.
58. Seorang anak bernama Tono ingin Imran bertugas membuat semua
membeli handuk di sebuah swalayan, angka tujuh (7) yang dibutuhkan.
harga handuk tersebut Rp 250.000, Berapa banyak angka 7 yang harus
namun karena membeli di swalayan dibuat? ….
maka handuk tersebut dikenakan A. 20
diskon 30% dan pajak sebesar 40% B. 11
setelah diskon, Jika dia beli di pasar C. 19
handuk tersebut seharga 250.000, D. 12
berapa persenkah E. 10

Marketplace untuk Staners. Platform online yang menyediakan seluruh kebutuhan yang berkualitas
untuk (calon) mahasiswa, alumni, dan orang-orang yang mempunyai ketertarikan terhadap PKN STAN.

www.stanersstore.com @stanersstore stanersstore


SOAL TRY-OUT 4

61. Helen menjumlahkan bilangan– B. −1, −1, −1 𝑎𝑡𝑎𝑢 (−2, −2, −2)
bilangan prima secara berurutan mulai C. 1, 1, 1 𝑎𝑡𝑎𝑢 (−2, −2, −2)
dari 2, 3, 5, 7 dan seterusnya. Ia D. 0,1,1 𝑎𝑡𝑎𝑢 (0,1,1)
berhenti begitu hasil penjumlahannya E. 1,1,0 𝑎𝑡𝑎𝑢 (1,1, 1)
melebihi 200. 65. Jika jari-jari sebuah kerucut dibesarkan
Bilangan terbesar yang dijumlahkan sampai 4 kali ukuran semula dan
Helen adalah? tingginya dibesarkan sampai 6 kali
A. 37 ukuran semula. Berapa volume
B. 41 kerucut setelah perubahan….
C. 43 A. 1 : 48
D. 47 B. 1 : 64
E. 51 C. 1 : 96
D. 1 : 72
62. Jika x = 56 56 56 56 … dan x E. 1 : 84
66. August De Morgan seorang
adalah bilangan positif, maka x2 -1 = … matematikawan hidup pada tahun
A. 37 1800an dan dia pernah berkata "saya
B. 48 pernah berumur x tahun pada tahun
C. 65 x2". pada tahun berapakah ia lahir …
D. 82 A. 1802
E. 101 B. 1805
63. Sebuah akuarium berbentuk balok C. 1803
(kotak) dengan luas alas 400cm2 diisi D. 1806
air E. 1808
Setinggi 25cm. Sebuah balok kayu 67. Dilakukan penelitian terhadap dua
dengan luas alas 100cm2 dimasukkan bakteri, yaitu bakteri A dan bakteri B.
ke dalam akuarium sampai seluruh Dalam tujuh hari pengamatan
balok kayu terendam air. Sesudah diketahui bahwa perkembangan
balok kayu tersebut dimasukkan bakteri A dan B sebanding. Jika A yang
ketinggian air naik menjadi 30cm. semula berjumlah 24 berkembang
Tinggi balok tersebut adalah ...cm. menjadi 108. B yang semula berjumlah
A. 10 14 akan berkembang menjadi …
B. 15 A. 112
C. 20 B. 96
D. 25 C. 72
E. 30 D. 63
64. Tentukan tripel (x, y, z) yang E. 54
memenuhi bahwa salah satu bilangan 68. Berapakah sisa 31000 dibagi 4?
jika ditambahkan dengan hasil kali A. 0
kedua bilangan yang lain hasilnya B. 1
adalah 2. C. 2
A. −1, 3 ,1 𝑎𝑡𝑎𝑢(3, −1, 1)

Marketplace untuk Staners. Platform online yang menyediakan seluruh kebutuhan yang berkualitas
untuk (calon) mahasiswa, alumni, dan orang-orang yang mempunyai ketertarikan terhadap PKN STAN.

www.stanersstore.com @stanersstore stanersstore


SOAL TRY-OUT 4

D. 3 E. 20
E. 4 72. Isba, seorang pelancong melakukan
69. Suatu persamaan kuadrat pencatatan atas perjalanannya
x2 - px + p + 1 = 0 memiliki akar-akar berkeliling kota Blitar. Tercatat 1024
x1 dan x2. jam 2025menit dan 3249 detik.
Jika diketahui x1 - x2 = 1, tentukanlah Berapa menitkah sebenarnya waktu
nilai p yang memenuhi persamaan yang diperlukan Isba (pembulatan)?
tersebut A. 1956
A. 5 atau 1 B. 1966
B. 5 atau -1 C. 1976
C. -5 atau 1 D. 1986
D. -5 atau -1 E. 1996
E. 1 atau 3 73. Tulus ditugaskan untuk mengantar
70. Ruang kamar tidur Asep berbentuk apel dari kebun ke pasar dengan
persegi panjang berukuran panjang gerobaknya. Tulus akan mendapat
5.3 m , lebar 3.7 m, dan tinggi 4 m. upah Rp48.000 untuk setiap gerobak
Rencananya dinding ruangan tersebut apel yang diantarnya. Gerobak Tulus
akan dilapisi dengan wallpaper dengan hanya mampu menampung 40 kg apel.
harga 1450 per cm2. . berapa biaya Jika Tulus ingin mendapat upah
total yang dibutuhkan Asep? sebesar Rp 240.000 maka ia perlu
A. 11.373.800 mengangkut apel sebanyak?
B. 10.440.000 A. 160
C. 8.294.000 B. 200
D. 5.917.450 C. 240
E. 2.146.000 D. 280
71. Adi, Bayu, Budi, dan Aji berencana E. 320
mengumpulkan pakaian bekas untuk 74. Seorang anak bernama Tono
diberikan kepada anak jalana. Adi mengendarai sepedha motor, ia
1
berhasil mnegumpulkan 6 lusin menempuh 9.235.000 km dalam 5
2
1 hari. Pada hari pertama ditempuhnya
pakaian. Budi sebanyak gross. Bayu
4
2 2.750.000 km, hari kedua 102.000 km,
sebanyak 3 kodi. Sedangkan Aji
5 hari ketiga 3.374.600 km dan hari
sebanyak 58 potong pakaian. Baju keempat 1.899.000 km. Berapakah
tersebut dibagikan kepada 24 anak jarak yang ditempuh pada hari kelima?
jalanan, maka masing-masing anak A. 1.009.600
jalanan mendapat ...potong pakaian. B. 1.109.600
A. 7 C. 1.109.400
B. 10 D. 1.009.400
C. 11 E. 1.019.400
D. 15

Marketplace untuk Staners. Platform online yang menyediakan seluruh kebutuhan yang berkualitas
untuk (calon) mahasiswa, alumni, dan orang-orang yang mempunyai ketertarikan terhadap PKN STAN.

www.stanersstore.com @stanersstore stanersstore


SOAL TRY-OUT 4

Grafik untuk nomor 75-77


Berikut adalah Jumlah Pesanan Kain Pabrik Tekstil ABC pada Tahun 2015

Banyaknya Pesanan

3,000
2,750 2,700
2,500
2,300 2,370 2,275
2,000
1,500 1,570 1,550
1,120 1,245
1,000 1,005
720
500
0

75. Jumlah rata-rata pesanan sepanjang B. 73,19%


tahun 2015 adalah? C. 73,91%
A. 1782 D. 79,31%
B. 1872 E. 73,39%
C. 1927 77. Berapa perbandingan pesanan pada
D. 1829 tahun awal dan tahun terakhir?
E. 1992 A. 2,35
76. Berapakah persentase perubahan B. 3,35
kenaikan dari bulan Januari ke C. 3,75
Februari, dibandingkan Juli ke D. 4,35
Agustus? E. 4,95
A. 71,39%

Marketplace untuk Staners. Platform online yang menyediakan seluruh kebutuhan yang berkualitas
untuk (calon) mahasiswa, alumni, dan orang-orang yang mempunyai ketertarikan terhadap PKN STAN.

www.stanersstore.com @stanersstore stanersstore


SOAL TRY-OUT 4

Diagram untuk nomor 78-80


Berikut adalah data Penjualan Ternak Tahun 2015

Penjualan Ternak Tahun 2015

Kambing
AYAM
29%

SAPI
24%
BURUNG DARA
21%
BEBEK
11%

78. Apabila total penjualan adalah 11.300, A. 595


maka median penjualan B. 559
adalah…(lakukan pembulatan C. 955
kebawah) D. 2155
A. 18 E. 2515
B. 19 80. Berapa persen jumlah penjualan yang
C. 20 melebihi rata-rata?
D. 21 A. 32%
E. 22 B. 35%
79. Jika total penjualan sapi pada kuartal C. 45%
berikutnya menurun 35% dari total D. 50%
penjualan Kambing, maka total E. 55%
penjualan sapi adalah…

Untuk soal nomor 81-90, pilihlah jawaban yang benar


81. 1, 2, 3, 3, 5, 5, 7, 7, …, ... C. 9 dan 11
A. 9 dan 9 D. 1 dan 2
B. 2 dan 1 E. 8 dan 8
Marketplace untuk Staners. Platform online yang menyediakan seluruh kebutuhan yang berkualitas
untuk (calon) mahasiswa, alumni, dan orang-orang yang mempunyai ketertarikan terhadap PKN STAN.

www.stanersstore.com @stanersstore stanersstore


SOAL TRY-OUT 4

82. 1, 0, 4, 1, 9, 2, ... C. H dan K


A. 16 dan 2 D. I dan L
B. 16 dan 3 E. J dan L
C. 16 dan 4 87. A, Q, I, O, U, M, E, …, …
D. 16 dan 5 A. B dan I
E. 16 dan 6 B. K dan O
83. 18, -19, -15, -17, 12, -15, ..., … C. K dan P
A. 9 dan -13 D. L dan H
B. -9 dan -13 E. H dan L
C. 10 dan -14 88. A, B, C, E, H,…, …
D. -10 dan -14 A. M dan U
E. 14 dan -10 B. S dan M
84. 123, 2, 153, 3, 194, 5, 245, 7, …, … C. S dan N
A. 346 dan 9 D. T dan F
B. 306 dan 9 E. P dan Q
C. 306 dan 11 89. Z, A, Y, E, X, I, W, M, …, …
D. 346 dan 11 A. S dan D
E. 305 dan 9 B. T dan R
85. 3, 5, 8, 13, 21, 34, …, … C. U dan E
A. 55 dan 89 D. W dan F
B. 23 dan 47 E. V dan Q
C. 59 dan 78 90. B,E, D,G, G, I, L, …, …
D. 59 dan 89 A. L dan Q
E. 23 dan 78 B. P dan J
86. Q, E, N, G, K, I, …, … C. E dan R
A. Z dan A D. J dan S
B. W dan J E. K dan S
Untuk nomor 91-104, masing-masing terdiri atas premis-premis dan lima kemungkinan
kesimpulan. Pilihlah jawaban yang paling tepat.
91. Tiap satu jam suhu lemari es naik 1 92. Semua wanita sayang putrinya.
derajat. Pada suhu 7 derajat es mulai Sebagian wanita tidak gemar
mencair. Sekarang pukul 11.00. memasak.
Dapat disimpulkan bahwa … Dapat disimpulkan bahwa …
A. Tiga jam lagi pukul 15.00 A. Semua wanita yang tidak gemar
B. Es mencair pukul 14.00 memasak sayang putrinya
C. Sekarang suhu lemari es 3 derajat B. Sebagian wanita yang sayang
D. naik satu derajat lagi dan es akan putrinya tidak gemar memasak
mencair C. Sebagian wanita tidak gemar
E. Es tidak akan mencair memasak dan tidak sayang putrinya
D. Sebagian wanita sayang putrinya
dan gemar memasak
Marketplace untuk Staners. Platform online yang menyediakan seluruh kebutuhan yang berkualitas
untuk (calon) mahasiswa, alumni, dan orang-orang yang mempunyai ketertarikan terhadap PKN STAN.

www.stanersstore.com @stanersstore stanersstore


SOAL TRY-OUT 4

E. Tidak ada wanita yang sayang Pernyataan yang sesuai dengan


putrinya dan gemar memasak premis di atas adalah …
93. Jika hewan hidup di darat, ia tidak bisa A. Baron membeli tanaman baru.
hidup di air. B. Baron menyirami kebunnya
Jika hewan hidup di air, ia tidak bisa C. Sepulang kerja, Oki mampir ke toko
hidup di darat tanaman hias
Kesimpulan yang tepat adalah … D. Tanaman hias Baron berbunga
A. Ada yang hidup di air dan juga di E. Besok sore Baron akan menyirami
darat tanamannya
B. Tidak ada yang hidup di air dan juga 96. Risa membuka toko laptop.
di darat. Semua laptop baru dengan merk
C. Tidak ada yang hidup di darat dan ternama di pajang di etalase toko.
juga tidak hidup di air. Laptop Paieo tidak dipajang di etalase
D. Ada yang tidak hidup di air dan juga toko.
tidak hidup di darat. Kesimpulan yang tepat adalah…
E. Tidak ada yang hidup di air dan juga A. Tidak dapat disimpulkan
tidak hidup di darat B. Laptop Paieo laptop baru dengan
94. Semua siswa pandai berbahasa merk ternama
Inggris. C. Laptop Paieo tidak dijual di toko
Siswa yang pandai berbahasa Inggris Risa
gemar berdiskusi. D. Sebagian laptop Risa tidak terjual
Siswa yang gemar berdiskusi tidak E. Laptop Paieo bukan laptop baru
pandai berdebat. dengan merk ternama
Dapat disimpulkan bahwa … 97. Setiap donor darah, Leli selalu
A. Sebagian siswa tidak pandai membeli satu kardus susu.
berdebat, tetapi pandai berbahasa Bulan ini Leli tidak membeli susu.
Inggris. Kesimpulan yang tepat adalah…
B. Sebagian siswa gemar berdiskusi A. Bulan ini Leli tidak gajian
dan pandai berbahasa Inggris. B. Leli donor darah
C. Semua siswa pandai berbahasa C. Susu langganan Leli tidak berjualan
Inggris, tetapi tidak gemar D. Panitia tidak menyediakan sekardus
berdiskusi. susu
D. Sebagian siswa gemar berdiskusi, E. Bulan ini Leli tidak donor darah
tetapi t idak pandai berbahasa 98. Karyawan adalah pegawai tetap.
Inggris dan tidak pandai berdebat. Sebagian karyawan tinggal di asrama,
E. Semua siswa tidak pandai kecuali Bonar.
berdebat, tetapi pandai berbahasa Kesimpulannya adalah …
Inggris. A. Bonar karyawan yang bukan
95. Air membuat tanaman menjadi segar. pegawai tetap.
Bunga lili di kebun Baron segar. B. Bonar bukan karyawan yang
pegawai tetap.
Marketplace untuk Staners. Platform online yang menyediakan seluruh kebutuhan yang berkualitas
untuk (calon) mahasiswa, alumni, dan orang-orang yang mempunyai ketertarikan terhadap PKN STAN.

www.stanersstore.com @stanersstore stanersstore


SOAL TRY-OUT 4

C. Kecuali Bonar, karyawan yang A. Semua pakaian yang murah bukan


tinggal di asrama adalah bukan pakaian musim dingin.
pegawai tetap. B. Semua pakaian musim dingin
D. Kecuali Bonar, karyawan adalah harganya murah.
pegawai tetap. C. Semua pakaian berbulu bukan
E. Bonar karyawan yang pegawai pakaian musim dingin.
tetap. D. Sebagian pakaian musim dingin
99. Metode penanganan kanker semakin mudah rusak.
modern.Sejak pertama diketahui E. Semua pakaian musim dingin tidak
mengidap kanker, pada tahun 1950- mudah rusak..
an, 60% penderita kanker 101. Jika hari panas, maka Ani memakai
hidupselama 5 tahun, sedangkan topi. Ani tidak memakai topi atau ia
tahun 1980-an, 65% penderita kanker memakai payung. Ani tidak memakai
hidup selama 8 tahun.. payung.Kesimpulan yang tepat adalah
A. Tahun 1950-an, hanya 60% …
penderita kanker yang ditangani, A. Hari panas.
sedangkan tahun 1980-an B. Hari tidak panas.
persentase penderita kanker yang C. Ani memakai topi.
ditangani lebih tinggi. D. Hari panas dan Ani memakai topi.
B. Tahun 1980-an, 35% penderita E. Hari tidak panas dan Ani memakai
kanker tidak ditangani seperti topi..
penderita pada tahun 1950-an. 102. Ada sekolah yang menerima dan
C. Tahun 1950-an tidak pernah tidak menerima dana Bantuan
dilakukan pendeteksian terhadap Operasional Sekolah (BOS).
pengidap kanker secara lebih awal Pencairan dana BOS mensyaratkan
pada tahun 1980-an. penyerahan laporan keuangan
D. Tahun 1980-an lebih banyak triwulan 00.
penderita kanker yang ditangani Dapat disimpulkan bahwa…
lebih baik daripada penderita A. Sekolah penerima BOS yang
kanker pada tahun 1950-an. belum menyerahkan laporan
E. Tahun 1980-an jumlah penderita keuangan triwulan dapat
kanker lebih banyak daripada mencairkan dana BOS.
jumlah penderita kanker tahun B. Sekolah penerima BOS yang
1950-an belum menyerahkan laporan
100. Semua pakaian musim dingin keuangan triwulan belum dapat
berbulu. mencairkan dana BOS.
Sebagian pakaian yang berbulu C. Sekolah bukan penerima BOS
harganya mahal.Sebagian pakaian yang sudah menyerahkan laporan
yang mahal tidak mudah rusak. keuangan triwulan belum dapat
Dapat disimpulkan bahwa… mencairkan dana BOS.

Marketplace untuk Staners. Platform online yang menyediakan seluruh kebutuhan yang berkualitas
untuk (calon) mahasiswa, alumni, dan orang-orang yang mempunyai ketertarikan terhadap PKN STAN.

www.stanersstore.com @stanersstore stanersstore


SOAL TRY-OUT 4

D. Sekolah bukan penerima BOS sebanyak 30 persen kematian


dapat mencairkan dana BOS tanpa akibatkecelakaan mobil.
menyerahkan laporan keuangan Dapat disimpulkan bahwa…
triwulan. A. angka kematian akibat
E. Sekolah bukan penerima BOS kecelakaan lalu lintas tidak
dapat mencairkan dana BOS dapat diturunkan kecuali dengan
dengan menyerahkan laporan pengunaan sabuk pengaman
keuangan triwulan. dan istirahat 15 menit.
103.Semua penipu pandai bicara dan B. angka kematian akibat
ramah. kecelakaan lalu lintas tidak
Tuan M tidak ramah, tetapi pandai dapat diturunkan dengan
bicara penggunaan sabuk pengaman
Kesimpulan yang tepat adalah … dan istirahat 15 menit.
A. Tuan M seorang penipu yang C. angka kematian akibat
pandai bicara. kecelakaan lalu lintas dapat
B. Tuan M seorang penipu yang tidak diturunkan dengan penggunaan
ramah. sabuk pengaman dan istirahat
C. Tuan M seorang penipu yang 15 menit.
pandai bicara dan tidak ramah. D. angka kematian akibat
D. Tuan M bukan seorang penipu, kecelakaan lalu lintas dapat
meskipun pandai bicara. diturunkan dengan penggunaan
E. Tuan M bukan seorang penipu sabuk pengamant anpa istirahat
yang ramah. 15 menit.
104.Penggunaan sabuk pengaman E. angka kematian akibat
menurunkan angka kematian akibat kecelakaan lalu lintas dapat
kecelakaan lalu lintas di diturunkan dengan istirahat 15
jalanraya.Istirahat 15 menit saat menitt anpa penggunaan sabuk
perjalanan di jalan tol menurunkan pengaman
Untuk nomor soal 105-107

Enam orang siswa Amir, boris, cinta,  Amir dan cinta memilih untuk
dona, eliza, dan faizal akan mengikuti mengikurti olahraga renang
ekstra kurikuler di sekolahnya. Jadwal  Boris dan faizal akan memilih
ekstrakurikuler adalah sebagai berikut: badminton
basket dan voli pada hari selasa;  Hanya eliza dan faizal yang tidak
badminton dan renang pada hari mengikuti olahraga basket.
kamis; futsaldan atletik pada hari  Faizal tidak memilih olahraga atletik.
sabtu.  Setiap ekstrakulikuler memiliki tempat
 Setiap siswa pasti mengikuti satu jenis berlatih yang berbeda- beda.
ekstrakurikuler tiap harinya. 105. Jika cinta dan faizal ingin
berolahraga bersama-sama di satu

Marketplace untuk Staners. Platform online yang menyediakan seluruh kebutuhan yang berkualitas
untuk (calon) mahasiswa, alumni, dan orang-orang yang mempunyai ketertarikan terhadap PKN STAN.

www.stanersstore.com @stanersstore stanersstore


SOAL TRY-OUT 4

ekstrakurikuler, maka mereka harus B. Badminton


memilih olahraga? C. Futsal
A. Badminton D. Renang
B. Basket E. Voli
C. Renang 107. Di bawah ini, olahraga apa saja
D. Voli yang dapat dipilih oleh Amir
E. Futsal A. voli, renang, dan futsal
106. jika eliza tidak ingin melihat Cinta B. atletik, basket, dan renang
dan Faizal berolahraga bersama, maka C. badminton, basket, dan atletik
olahraga yang dipilihnya adalah… D. atletik, voli, dan renang
A. Atletik E. voli, badminton, dan atletik
Soal no. 108 – 110 didasarkan pada keterangan berikut.

P, Q, R,S, T,U,V tinggal di sepanjang jalan B. P,Q,U,S,R,T,V


lurus di Pantura. Jarak masing-masing C. P,Q,S,U,T,R,V
rumah mereka adalah sebagai berikut: D. P,Q,T,S,U,R,V
 jarak P ke Q adalah 8 km E. P,U,Q,S,R,T,V
 jarak Q ke S adalah 9 km 109. Jarak R ke V adalah 23 km, maka
 jarak S ke T adalah 15 km ….
 jarak U ke R adalah 8 km A. Jarak R ke Q adalah 20 km
 P ke Q lebih dekat disbanding P ke B. Jarak R ke T adalah 3 km
S, dan U berada diantara Q dan S C. Jarak P ke R adalah 25 km
 P dan V berada di ujung timur dan D. Jarak R ke S adalah 12 km
ujung barat, berjarak 52 km E. Jarak V ke S adalah 27 km
 Jika ingin pergi ke rumah R, T harus 110. Jarak U ke S adakah 7 km, maka ….
menuju arah yang berbeda dengan A. Jarak U ke R adalah 19 km
jika ia ingin pergi ke rumah V B. Jarak Q ke U adalah 4 km kalinya
Jika terdapat ketentuan tambahan dalam jarak P ke Q
soal, maka ketentuan itu adalah informasi C. Jarak U ke V adalah 43 km
tambahan dan bersifat saling berkaitan D. Jarak P ke U adalah 12 km
antara soal satu dengan soal lainnya E. Jarak T ke U adalah 21 km ni, Lani,
108. urutan rumah yang benar adalah … dan Ita
A. P,Q,U,S,T,R,V
Soal no. 111 – 114 didasarkan pada keterangan berikut.

Sebuah kompetisi balap mobil diikuti oleh posisi runner-Up di depan Ricky. Ssteve
tujuh orang pembalap. Carlos selalu berada di posisi keenam dsn dibuntuti
membututi Ricky . mark harus melihat ke oleh Gabriel.
spion untuk melihat lawan-lawannya, 111. siapakah yang menjadi pemenang
sementara Steve harus bersabar menahan di balapan tersebut.
bau asap knalpot Peter. Hero berada di A. Ricky

Marketplace untuk Staners. Platform online yang menyediakan seluruh kebutuhan yang berkualitas
untuk (calon) mahasiswa, alumni, dan orang-orang yang mempunyai ketertarikan terhadap PKN STAN.

www.stanersstore.com @stanersstore stanersstore


SOAL TRY-OUT 4

B. Carlos A. Pertama
C. Mark B. Kedua
D. Peter C. Ketiga
E. Steve D. Keempat
112. Jika Carlos berhasil melewati dua E. Kelima
pembalap didepannya, berarti 114.jika Peter ingin berada di urutan
sekarang ia berada pada posisi ketiga, maka ia harus menyalip?
A. Pertama A. Mark,Carlos, Hero
B. Kedua B. Carlos, Hero, Mark, Ricky
C. Ketiga C. Gabriel
D. Keempat D. Carlos, ricky
E. Kelima E. Steve, gabriel n
113. jika Peter terjatuh dan tidak dapat
melanjutkan balapan, maka Hero
akan berada pada urutan??
A B
C D
E

Untuk nomor 117-18. Temukan jaring-


jaring dari kubus dibawah ini
Untuk nomor 115-16. Gambar dibawah ini
mempunyai sebuah pola. Pilih salah satu
gambar yang memiliki pola yang berbeda
115.

117.

A B
C D
E

116.

Marketplace untuk Staners. Platform online yang menyediakan seluruh kebutuhan yang berkualitas
untuk (calon) mahasiswa, alumni, dan orang-orang yang mempunyai ketertarikan terhadap PKN STAN.

www.stanersstore.com @stanersstore stanersstore


SOAL TRY-OUT 4

Untuk nomor 117-18. Temukan kubus dari


jaring-jaring berikut

119.

A B
C D
E

A B
C D
118. E

120.

A B
C D
E

Marketplace untuk Staners. Platform online yang menyediakan seluruh kebutuhan yang berkualitas
untuk (calon) mahasiswa, alumni, dan orang-orang yang mempunyai ketertarikan terhadap PKN STAN.

www.stanersstore.com @stanersstore stanersstore


SOAL TRY-OUT 4

A B
C D

Marketplace untuk Staners. Platform online yang menyediakan seluruh kebutuhan yang berkualitas
untuk (calon) mahasiswa, alumni, dan orang-orang yang mempunyai ketertarikan terhadap PKN STAN.

www.stanersstore.com @stanersstore stanersstore


SOAL TRY-OUT 4

1. Aji wished he ___ to his wife‟s funeral 9. The victims of the earthquake showed
a week before. their ___ to the local government
a. Had gone officials for their help
b. Went a. Appreciate
c. Has gone b. Appreciable
d. Were going c. Appreciation
2. They‟d rather I ___ do anything to the d. Appreciative
sick horses. 10. They dismiss the worker, but they
a. Do never told me about the ___
b. Didn‟t do a. Dismissing
c. Don‟t b. Dismissly
d. Didn‟t c. Dismission
3. I‟d rather not ___ the matters with you. d. Dismissive
a. Discussing 11. The North Platte River ___ from
b. To discuss Wyoming to Alaska.
c. Discuss a. It flowed
d. Discussion b. Flows
4. The girl screamed as if ___ c. Flowing
a. She saw a ghost d. With flowing water
b. She has seen a ghost 12. The Apollo 11 astronauts ___ of the
c. She had seen a ghost Earths inhabitants witnessed on the
d. She was seeing a ghost famous first moonwalk on July 20,
5. He‟d rather I ___ the debts tomorrow. 1969, were Neil Amstrong and Buzz
a. Pays Aldrin.
b. Has paying a. Whom
c. Paid b. Whom millions
d. pay c. Were some
6. The hall has been ___ decorated for the d. Whom some where
opening ceremony. (Modul 16-1) 13. Were ___ millions of dollars each year
a. Beautiful replenishing eroding beaches, the
b. Beautify coastline would be changing even
c. Beautifully more rapidly.
d. Beautified a. The US Army Corps of Engineers
7. They obey their teacher‟s ___ to look not spending
up the new words in the dictionary. b. The US Army Corps of Engineers
a. Instructed not spend
b. Instructive c. The US Army Corps of Engineers
c. Instruct do not spend
d. Instruction d. Not spending the US Army Corps
8. Indonesian batik is __ recognized. of Engineers
a. Internationally 14. Among bees ___ a highly elaborate
b. International form of communication.
c. Internationalization a. Occur
d. Internationalized b. Occurs
c. It occurs
d. They occur

Marketplace untuk Staners. Platform online yang menyediakan seluruh kebutuhan yang berkualitas
untuk (calon) mahasiswa, alumni, dan orang-orang yang mempunyai ketertarikan terhadap PKN STAN.

www.stanersstore.com @stanersstore stanersstore


SOAL TRY-OUT 4

15. Truman Capotes‟s In Cold Blood is b. Is an abundance of


neither journalistically accurate ___ c. It is abundant
a. A piece of fiction d. An abundance is
b. Not a fictitious work 23. As ___ in Greek and Roman
c. Or written in a fictitious way mythology, harpies were frightful
d. Not completely fictitious monsters that were half woman and
16. ___ in Stevenson‟s Landscape, the half bird.
more vitality and character the a. Described
paintings seem to possess. b. To describe
a. The brushwork is loose c. Description
b. The looser brushwork d. Describing
c. The loose brushwork is 24. ___ to occur in the Earth‟s crust, pus-
d. The looser the brushwork is pull and shake waves would be
17. ___ receipt must be removed from the generated simultaneously.
cashiers drawer and tallied. a. Were a break
a. Every b. If a break
b. Both c. A break was
c. Two d. If broken
d. Many 25. ___ tea plant are small and white
18. Mary and Mark invited ___ parents to a. The
see their new apartment. b. On the
a. Theirs c. Having flowers the
b. Them d. The flowers of the
c. Their 26. All the end of the nineteenth century,
d. They Alfred Binet developed a test for
19. Points will be subtracted for each __ measuring intelligence ___ served as
answered question. the basis of modern IQ tests.
a. Incorrect a. Has
b. Incorrects b. It has
c. Incorreted c. And
d. Incorrectly d. Which has
20. Everyone else was __ by the time I 27. Nowhere ___ more skewed than in the
arrived home. auto industry.
a. Asleep a. The retail trade figures
b. Sleeping b. Retail trade figures
c. Sleep c. Are retail trade figures
d. Slept d. Retail trade figures
21. He took ___ money from his wallet to 28. ___ heated by solar energy have
pay for sweater. special collectors on the roofs to trap
a. The sunlight.
b. A a. A home is
c. Many b. Homes are
d. Which c. A home
22. In the Antarctic Ocean ___ plankton d. Homes
and crustacean forms of life. 29. Because bone loss occur earlier in
a. An abundance woman than ___, the effects of

Marketplace untuk Staners. Platform online yang menyediakan seluruh kebutuhan yang berkualitas
untuk (calon) mahasiswa, alumni, dan orang-orang yang mempunyai ketertarikan terhadap PKN STAN.

www.stanersstore.com @stanersstore stanersstore


SOAL TRY-OUT 4

osteoporosis are more apparent in a. Must commit by


woman. b. Must be committed
a. Men do c. Must have committed
b. In men d. Must have been committed by
c. As men 37. The television ___ that was shown last
d. Similar to men night on the CBS network from 9:00 to
30. ___ discussed by the board of directors 11:00 was one of the best shows of the
when it was proposed again by the season.
supervisors. a. Produce
a. The problem had already b. Produced
b. The problem is already c. Production
c. The problem had already been d. Producer
d. The problem has already 38. In spite of ___ small size, these video
31. Henry Adams, born in Boston ___ recorders produce excellent tapes.
famous as a historian and novelist. a. Their
a. Became b. Its
b. And became c. Theirs
c. He was d. Them
d. And he became 39. The project was ___ close to being
32. The three basic chords in ___ the tonic, finished.
the dominant and the sub-dominant. a. Remarked
a. Functional harmony b. Remarkably
b. Functional harmony is c. Remark
c. Functional harmony are d. Remarkable
d. Functional harmony they are 40. The empty bottles are to the left, and
33. ___ variety of flowers in the show the ___ bottles are to the right.
from simple carnations to the most a. Filling
exquisite roses. b. Fills
a. A wide c. Fill
b. There was a wide d. Filled
c. Was there
d. Many
34. Vitamin C is necessary for prevention
and ___ scurvy.
a. It cures
b. Cures
c. Cure
d. For curing
35. The speed of light is ___ the speed of
the sound.
a. Faster
b. Much faster than
c. As fast
36. Much of the carnage of elephants,
giraffes and big cats ___ uncaring
hunters.

Marketplace untuk Staners. Platform online yang menyediakan seluruh kebutuhan yang berkualitas
untuk (calon) mahasiswa, alumni, dan orang-orang yang mempunyai ketertarikan terhadap PKN STAN.

www.stanersstore.com @stanersstore stanersstore


SOAL TRY-OUT 4

41. Event though we had been (A) to herhouse several times before, we did not remember (B)
exactly what (C) street was it (D) on.
42. The(A) executives demanded that a trained(B) technician repaired(C) the(D) copier.
43. Calcium is known(A) as an(B) nutrient(C) that will decrease(D) hypertension and colon
cancer.
44. In(A) the movie, the(B) actor has showed(C) a fiercelly(D), dreamy and brave character.
45. A body (A) of volunteers have been(B) organized to aid (C) the helpless in their(D)
struggle for survival.
46. Seriously burned(A) in a terrible car accident, the doctor was not sure that John could
be protected(B) from infection long enough(C) for his body to begin to heal itself(D).
47. His mother explicitly(A) told he (B) not to go out to the river by himself(C), but he did
not listen (D).
48. The victims of (A) the eviction (B) approved to (C) the compensation (D) given by the
company.
49. The report cites (A) that the number of employees in (B) the private sector have(C)
doubled between 2007 and(D) 2010.
50. A number of visitors of the zoo are warned (A) not feeding (B) the animals for (C)
several health reasons(D).

PASSAGE ONE (Question 51-55)

Dr. Trounson has gone one step further that the “test-tube” fertilization technique first
employed successfully in 1978 and since emulated in such places as the US, South Africa,
Britain itself and Australia, by setting up an “embryo bank” to keep a supply of frozen,
fertilized eggs available indefinitely. In case the first fertilized by the father at the same time
as the first, could be withdrawn from the “bank” for a second attempt.
The pioneers if successful “test-tube” births, Steptoe and Edwards had been the first
to come up with this storage idea, but they had been forced to withdraw their plan because of
the controversy of it aroused. The problem in both countries was, of course one of morality,
although that should not be taken to imply that there is necessarily more morality in Great
Britain than Australia. The concern has been that the embryo bank might be exploited by the
unscrupulous, or that conception might precede by nine or even ninety years, rather than by
nine month. As happened some years ago with heart transplants and as will doubtless happen
again, the present situation as far as embryo bank are concerned appears to be that “the
technology has outrun the morality”
51. The paragraph preceding this extract probably dealt with…
a. The intended recipient
b. Dr. Trounson
c. Embryo bank
d. Steptoe and Edwards
52. The passage implies that the first „test-tube‟ fertilization…
a. Occurred in the United States
b. Was carried out by Dr. Trounson
Marketplace untuk Staners. Platform online yang menyediakan seluruh kebutuhan yang berkualitas
untuk (calon) mahasiswa, alumni, dan orang-orang yang mempunyai ketertarikan terhadap PKN STAN.

www.stanersstore.com @stanersstore stanersstore


SOAL TRY-OUT 4

c. Took place in Australia


d. Was carried out by Steptoe and Edwards
53. It can be inferred from the passage that Dr/ Trounson works in….
a. Australia
b. South Africa
c. The United States
d. England
54. Where did this passage most probably appear?
a. In a specialized periodical for doctors
b. On the front page of a daily newspaper
c. In a weekly news magazine with a general readership
d. In a specialized periodical for moral philosophers
55. Which of the following can be inferred from the passage?
a. The intended recipient of a fertilized egg from the embryo bank is the original
product of the egg
b. These are at least two types of moral issue associated with embryo banks
c. New moral attitudes develop more quickly than new technology
d. These are similarities in the moral dilemmas surrounding heart transplants and
embryo banks

PASSAGE TWO (Question 56-60)

The Mary Celeste is a mystery ship. Something unexpected happened to it, and to this
day, no one is sure of the fate of the ship and its passengers. Here is what little is known
about the events leading up to the discovery of the empty ship. The Mary Celeste was found
abandoned in the mid-Atlantic in 1872. On December 4, 1872, the Mary Celeste was spotted
by the British ship Dei Gratia. The Mary Celeste looked strange because the sails were not
fully set and no crew members could be seen. Crew members of the Dei Gratia immediately
took a boat over the Mary Celeste. They identified the ship as the Mary Celeste, which had
left New York on November 7 with a full load and ten people on aboard.

The situation on the Mary Celeste was quite mysterious. No one was on the ship. The
ship had simply been left in the middle of the ocean. No obvious reason why the ship had
been abandoned could be seen. There were plenty of supplies, the complete cargo was still on
board, the crew belongings were there and there was no sign of violence disaster. However,
one of the longboats and the captain‟s navigational instrument were missing.

From the strange situation on the ship, a conclusion was drawn about the date of the
ship. The belief was that for some unknown reasons, the captain along with his wife, his
young daughter and the seven crew members, had decided to abandon the ship in the middle
of the ocean never to be seen again.

56. The best title for this passage is…


a. A Mysterious Cargo
Marketplace untuk Staners. Platform online yang menyediakan seluruh kebutuhan yang berkualitas
untuk (calon) mahasiswa, alumni, dan orang-orang yang mempunyai ketertarikan terhadap PKN STAN.

www.stanersstore.com @stanersstore stanersstore


SOAL TRY-OUT 4

b. The Mary Celeste


c. A Ship of Mystery
d. A Typical Voyage of the Mary Celeste
57. “Leading up to” in line 3 has closest meaning to…
a. Directing
b. Preceding
c. Pulling
d. Showing
58. According to the passage what did the crew of the Dei Gratia find?
a. A ship with full sails and no crew members
b. A ship with no sails and few crew members
c. A ship with sails not completely set and few crew
d. A ship with sails not completely set and no crew members
59. It can be inferred from the passage that when it was found, the Mary Celeste had been
abandoned for….
a. Less than a month
b. More than a month
c. More than a year
d. More than two years
60. What eventually happened to the people on the Mary Celeste?
a. They boarded the Dei Gratia
b. They headed to New York on a long boat
c. Their fate is unknown
d. They were found late in 1872

Marketplace untuk Staners. Platform online yang menyediakan seluruh kebutuhan yang berkualitas
untuk (calon) mahasiswa, alumni, dan orang-orang yang mempunyai ketertarikan terhadap PKN STAN.

www.stanersstore.com @stanersstore stanersstore


KUNCI TRY-OUT 4

KUNCI JAWABAN TPA


1. A. Demisioner keadaan tanpa kekuasaan (misalnya suatu kabinet dan sebagainya yang
telah mengembalikan mandat kepada kepala negara, tetapi masih melaksanakan tugas
sehari-hari sambil menunggu dilantiknya kabinet yang baru)
2. D. etatisme paham yang lebih mementingkan negara daripada rakyatnya
3. C.aneksasi/anek·sa·si/ /anéksasi/ n pengambilan dengan paksa tanah (wilayah) orang
(negara) lain untuk disatukan dengan tanah (negara) sendiri; penyerobotan;
pencaplokan;
4. B Brigade 1 satuan angkatan bersenjata yang terdiri atas dua, tiga, atau empat resimen,
merupakan bagian dari divisi; 2 satuan berseragam dengan tugas khusus;
5. D Arbiter orang yang disepakati oleh dua belah pihak yang bersengketa untuk
memberikan keputusan yang akan ditaati oleh kedua belah pihak
6. A Duplik jawaban kedua (dari terdakwa atau pembela) sebagai jawaban atas replik
7. C. Protesis penambahan vokal atau konsonan pada awal kata, untuk memudahkan lafal
misalnya e pada nyak menjadi enyakdan sebagainya
8. A. Feodalisme sistem sosial atau politik yang memberikan kekuasaan yang besar kepada
golongan bangsawan;
9. B. Ulayat wilayah
10. B. Testamen wasiat
11. E. Kolokium 1 pertemuan keahlian; seminar; 2 Dik kegiatan belajar (pada tataran pendidikan
sarjana) yang dilakukan dalam bentuk seminar untuk membahas proyek penelitian bertaraf
lanjutan
12. C. Integritas : kejujuran >< hipokrit : munafik
13. B Skeptis : ragu-ragu >< yakin
14. D. Prominen: terkemuka ><biasa
15. C. Takzim: amat hormat dan sopan ><lancang
16. C. Sekuler : keduniawian >< spiritualisme: keagamaan
17. D gempa dapat diukur dengan skala ritcher, sedang suku dapat diukur dengan skala
Fahrenheit
18. C. Padanan yang digunakan adalah ”sifat”.”Fiktif adalah fakta yang tidak benar-benar
terjadi, dongeng adalah peristiwa yang tidak benar-benar terjadi
19. B. seseorang yang kelebihan ilmu disebut genius, sedangkan seseorang yang kelebihan
gule disebut diabetes.
20. D. Di dalam sebuah film ada satu bagian yang disebut skenario. Kata-kata yang
mempunyai padanan hubungan dengan soal adalah Sandiwara: Plot, karena di dalam
sebuah sandiwara ada satu bagian yang disebut plot.
21. C. Kertas berbahan dasar pohon. Tepung diciptakan dari beras dimana prosesnya
dengan ditumbuk
22. B. Analgesik merupakan sesuatu yang dapat mengurangi nyeri. Kata-kata yang
mempunyai padanan hubungan dengan soal adalah Pelumas: Gesekan, karena pelumas
merupakan sesuatu yang dapat mengurangi gesekan.

Marketplace untuk Staners. Platform online yang menyediakan seluruh kebutuhan yang berkualitas
untuk (calon) mahasiswa, alumni, dan orang-orang yang mempunyai ketertarikan terhadap PKN STAN.

www.stanersstore.com @stanersstore stanersstore


KUNCI TRY-OUT 4

23. A. Suara adalah sesuatu yang dapat kita dengar, sedangkan arti adalah sesuatu yang
dapat kita pikir
24. A. APEL yang berada di RUANG TERBUKA mengalami proses PENCOKLATAN. Siswa yang
berada di kampus mengalami proses pembelajaran
25. B. murid membaca buku di perpustakaan, sedangkan nasabah menabung uang di bank.
26. E. lihat paragraph ketujuh
27. A. Dalam teks dibahas mengenai Bahan pembuat plastic yang ramah lingkungan yaitu
singkong dan khitonan.
28. C. terdapat pada alinea terakhir
29. C. khitonan tidak larut air. Lihat paragraph ketujuh
30. C. terdapat pada paragraph kedua
31. C. Dalam paragraf pembuka diuraikan tanaman hias dalam ruangan mampu
membersihkan udara, menyerap polutan dan gas-gas berbahaya
32. D. Tidak terdapat/tertulis dalam teks. Golden pothos adalah nama lain sirih gading. Baca
teks secara seksama
33. C. Sudah tercermin dalam no. 31 tentang simpulan isi wacana.
34. E. Di Indonesia Gerber daisy dikenal dengan nama Herbras tapi tanaman ini tidak asli
Indonesia.
35. D. di dalam artikel tidak terdapat keterangan yang menunjukkan spider plant dapat
menetralisir racun.
36. D.
54 + 37 = 91
1000 − 91 = 909

93 + 74 = 167
1000 − 167 = 833
37. D
9 65
8𝑚 + 𝑚 = 𝑚
7 7
3 93
−12𝑛 − 𝑛 = − 𝑛
8 8
65 93 520 651
𝑚− 𝑛= 𝑚− 𝑛
7 8 56 56

38. D
1 3
=
𝑚 4
4
𝑚=
3
1 1
= = −3
5𝑚 − 7 5 ∗ 4 − 7
3
39. D

Marketplace untuk Staners. Platform online yang menyediakan seluruh kebutuhan yang berkualitas
untuk (calon) mahasiswa, alumni, dan orang-orang yang mempunyai ketertarikan terhadap PKN STAN.

www.stanersstore.com @stanersstore stanersstore


KUNCI TRY-OUT 4

𝑓 322 = 𝑓 𝑔 2 × 224 + 3 = 42 − 56 + 5 = −35


40. C
21 26 14 7 2 1 1 1 17
𝑚= + + + ≈ + + + =
99 76 29 16 10 3 2 2 30
47
𝑚+1=
10
41. A
2 2
𝑚= 𝑛, 𝑜 = 𝑛, 2 𝑚𝑛𝑜 = 15625
25 5
25
15625 = 2 × 𝑚 × 𝑚×5𝑚
2
125 3
125 𝑥 125 = 2 𝑥 𝑚
2
𝑚3 = 125
m=5

𝑜 = 15 × 17 = 255

42. A
0,8 4,5 0,3 0,6
× : × =𝑚
6,4 0,9 0,9 0,36
45
𝑚= = 1,125
40
43. B
𝐵𝑖𝑙𝑎 𝑏𝑒𝑙𝑢𝑚 𝑚𝑒𝑛𝑔𝑒𝑛𝑎𝑙 𝑟𝑢𝑚𝑢𝑠 (𝑎𝑏) ³, 𝑚𝑎𝑘𝑎 𝑠𝑜𝑎𝑙 𝑡𝑒𝑟𝑠𝑒𝑏𝑢𝑡 𝑏𝑖𝑠𝑎 𝑑𝑖𝑠𝑒𝑙𝑒𝑠𝑎𝑖𝑘𝑎𝑛 𝑑𝑒𝑛𝑔𝑎𝑛 𝑐𝑎𝑟𝑎 𝑚𝑒𝑚𝑎𝑠𝑢𝑘𝑎
− 𝑚𝑎𝑠𝑖𝑛𝑔 𝑛𝑖𝑎𝑙𝑖 𝑎 𝑑𝑎𝑛 𝑛𝑖𝑙𝑎𝑖 𝑏, 𝑠𝑒𝑕𝑖𝑛𝑔𝑔𝑎 𝑑𝑖𝑝𝑒𝑟𝑜𝑙𝑒𝑕:
= 5 ³ − 3.5 ². 2 + 3.5.2² − 2 ³
= 125 − 150 + 60 − 8
= 185 − 158
= 27
𝐵𝑎𝑔𝑖 𝑦𝑎𝑛𝑔 𝑠𝑢𝑑𝑎𝑕 𝑡𝑎𝑕𝑢 𝑏𝑎𝑕𝑤𝑎 𝑝𝑒𝑟𝑠𝑎𝑚𝑎𝑎𝑛 𝑡𝑒𝑟𝑠𝑒𝑏𝑢𝑡 𝑡𝑒𝑟𝑛𝑦𝑎𝑡𝑎 𝑠𝑎𝑚𝑎 𝑑𝑒𝑛𝑔𝑎𝑛 (𝑎
− 𝑏) ³, 𝑚𝑎𝑘𝑎 𝑑𝑒𝑛𝑔𝑎𝑛 𝑚𝑢𝑑𝑎𝑕 𝑚𝑒𝑛𝑔𝑕𝑖𝑡𝑢𝑛𝑔 (5 − 2) ³ = 3³ = 27
44. E
𝑚3 𝑛7 = 43904
𝑚𝑛 = 14
𝑚 𝑛 = 𝑚𝑛 3 × 𝑛4
3 7

43904 = 14 3 × 𝑛4
𝑛4 = 16
𝑛=2
𝑚 = 7 (mn =14)
45. D
𝑈𝑛𝑡𝑢𝑘 𝑚𝑒𝑛𝑦𝑒𝑙𝑒𝑠𝑎𝑖𝑘𝑎𝑛 𝑠𝑜𝑎𝑙 𝑠𝑒𝑝𝑒𝑟𝑡𝑖 𝑖𝑛𝑖, 𝑘𝑖𝑡𝑎 𝑝𝑒𝑟𝑙𝑢 𝑚𝑒𝑛𝑔𝑢𝑏𝑎𝑕 𝑝𝑒𝑟𝑠𝑎𝑚𝑎𝑎𝑛 𝑡𝑒𝑟𝑠𝑒𝑏𝑢𝑡 𝑚𝑒𝑛𝑗𝑎𝑑𝑖 𝑝𝑒𝑟𝑠𝑎𝑚
32𝑥 + 33 − 2𝑥 − 28 = 0;

Marketplace untuk Staners. Platform online yang menyediakan seluruh kebutuhan yang berkualitas
untuk (calon) mahasiswa, alumni, dan orang-orang yang mempunyai ketertarikan terhadap PKN STAN.

www.stanersstore.com @stanersstore stanersstore


KUNCI TRY-OUT 4

𝑚𝑖𝑠𝑎𝑙𝑘𝑎𝑛 32𝑥 = 𝑎
⇒ 32𝑥 + (33)/32𝑥 − 28 = 0
⇒ 𝑎 + 27/𝑎 − 28 = 0
⇒ 𝑎2 − 27 − 28𝑎 = 0
⇒ 𝑎2 − 28𝑎 − 27 = 0
⇒ (𝑎 − 1)(𝑎 − 27) = 0
⇒ 𝑎 = 1 𝑎𝑡𝑎𝑢 𝑎 = 27 𝑈𝑛𝑡𝑢𝑘 𝑎 = 1, 𝑚𝑎𝑘𝑎 ∶ 32𝑥 = 𝑎
⇒ 32𝑥 = 1
⇒ 32𝑥 = 30
⇒ 2𝑥 = 0
⇒ 𝑥1 = 0 𝑈𝑛𝑡𝑢𝑘 𝑎 = 27,
𝑚𝑎𝑘𝑎 ∶ 32𝑥 = 𝑎
⇒ 32𝑥 = 27
⇒ 32𝑥 = 33
⇒ 2𝑥 = 3
⇒ 𝑥2 = 3/2 𝐽𝑎𝑑𝑖 𝑥1 + 𝑥2 = 0 + 3/2 = 3/2
46. B
𝑎 = 𝐴𝐷𝐼 − 𝐴𝐷𝐸
𝑏 = 𝐴𝐷𝐼 + 5 − 𝐴𝐷𝐸 − 5 = 𝑎 + 10
47. C
Jelas. Volume bola = 4/3 phi r3
Makin besar jari2 makin besar volume bola
48. B
X = 1 ² + 2 ² + 3² + 4 ² + 5 ² + .............+ 9 ² =
=1+4+9+16+25+36+49+64+81
=(1+49)+(4+36)+(9,81)+(16+64)+25
=50+40+90+80+25
=285
Y= 1+2+3+4+5+ ....+29=

1/2N x (N+1)
Pada soal di atas N=29, maka
(29/2) x (29+1)
=29/2 x 30
=29 x 15
=435

49. B
Dapat dilihat langsung dari persamaan x/y = ¼, y = 4X
50. B
X= 2 + 3 + 5 + 7 + 11 + 13 + 17 + 19 + 23 + 29 + 31 + 37 + 41 + 43 + 47 = 328
Y =7 + 14 + 21 + 28 + 35 + 42 + 49 + 56 + 63 + 70 = 385

Marketplace untuk Staners. Platform online yang menyediakan seluruh kebutuhan yang berkualitas
untuk (calon) mahasiswa, alumni, dan orang-orang yang mempunyai ketertarikan terhadap PKN STAN.

www.stanersstore.com @stanersstore stanersstore


KUNCI TRY-OUT 4

51. B.
𝑥 + 10 2
=−
𝑦 + 45 9
9𝑥 − 90 = −2𝑦 − 90
9𝑥 = −2𝑦
2
𝑥=− 𝑦
9
𝑥 2
=−
𝑦 9
𝑥 2 2 2 4
= − =
𝑦 9 81
52. B
Diagonal ruang kubus 10 akar 3, berarti sisi kubus 10.
X= Volume kubus = 10 x 10 x 10 = 1000
Y= Volume limas = 1/3 x 15 x 15 x 7= 525
Jadi X >Y

53. A

54. B
𝑥 = 9, 𝑦 = 12
55. B
Y% x 80 = 32
Y/100 x 80 = 32
Y = 32 x 100 : 80
Y=4

X = 32 % x 80
X = 25.6

Marketplace untuk Staners. Platform online yang menyediakan seluruh kebutuhan yang berkualitas
untuk (calon) mahasiswa, alumni, dan orang-orang yang mempunyai ketertarikan terhadap PKN STAN.

www.stanersstore.com @stanersstore stanersstore


KUNCI TRY-OUT 4

30 𝑋 42 𝑋 13
56. C. = 75,83 Dibulatkan kebawah karena kalau keatas kapasitas kardus tidak
6 𝑋 6 𝑋6
akan cukup.
Satu kardus 75 buah. Dua kardus 150
𝑎+𝑏 + 𝑏+𝑐 + 𝑎+𝑐
57. C 𝑎 + 𝑏 + 𝑐 = = 19
2
58. C
250.000 × 70% = 175.000
175.000 × 140% = 245.000−→ 𝑢𝑛𝑡𝑢𝑛𝑔 2% 𝑑𝑎𝑟𝑖 250.000
59. C
 Misalkan x = selisih dari jumlah buah anak tangga dengan langkah kaki yang ikut
menapak saat eskalator seandainya berhenti.
 Dalam keadaan eskalator berjalan + 25 langkah Agus = lantai 2.
 Dalam keadaan eskalator berhenti:
25 langkah + x = lantai 2.
 Dalam keadaan eskalator berjalan + 9 langkah Jaka = lantai 2.
 Dalam keadaan eskalator berhenti:
9 langkah + 2x = lantai 2.
Jadi:
25 langkah + x = 9 langkah + 2x
25 – 9 = 2x – x
x = 16
Maka jumlah anak tangga eskalator tersebut adalah:
25 + x atau 9 + 2x
25 + 16 atau 9 + 2(16)
= 41 buah anak tangga

60. A. yaitu 10 angka 7 sebagai satuan ditambah 10 angka 7 sebagai puluhan


61. B. 2 + 3 + 5 + 7 + 11 + 13 + 17 + 19 + 23 + 29 + 31 + 37 + 41 = 238
Berarti bilangan prima terbesar yang dijumlahkan Helen adalah 41
62. B

x = 56 56 56 56 …

x2= 56 - 56 56 56 56 …

x2 = 56 – x
x2+ x -56 = 0
(x+8)(x-7)=0
X = -8 atau X= 7
Karena x bilangan positif maka x = 7, x2-1 = 48
63. C

Marketplace untuk Staners. Platform online yang menyediakan seluruh kebutuhan yang berkualitas
untuk (calon) mahasiswa, alumni, dan orang-orang yang mempunyai ketertarikan terhadap PKN STAN.

www.stanersstore.com @stanersstore stanersstore


KUNCI TRY-OUT 4

Perhatikan bahwa volum akuarium sebelum diisi balok kayu

= Luas alas × tinggi air semula

= 400cm2 × 25cm

= 10000 cm3

Volum akuarium setelah diisi balok kayu

= Luas alas × tinggi air setelah kenaikan

= 400cm2 × 30 cm

= 12000 cm3

Jadi selisih volum antara sesudah dan sebelum diisi balok kayu adalah 12000 cm3- 10000
cm3 = 2000 cm3 .

Berarti selisih volum ini terjadi karena akuarium dimasuki balok, dengan kata lain

volum balok sama dengan selisih volum yang terjadi, yakni 2000 cm3

Sehingga tinggi balok = volume/luas alas balok = 2000/100 = 20 cm

64. C
X+yz = 2...(1)
Y+xz = 2...(2)
Z+xy = 2...(3)

Sulap 1 dan 2
(X-y)(z-1) = 0
Didapat x=y atau z=1
Sulap (2) dan (3)
(Y-z)(x-1)=0
Y=z dan x=1 X=y=z
Subs X+x^2-2 = 0
(X+2)(x-1)=0
X=-2 atau x=1
Krn x=y=z jd solusinya:
(1,1,1),(-2,-2,-2)
65. E
Volume kerucut 1 : Volume kerucut 2
1/3 𝜋 r12t1 :1/3 𝜋 r22 t2
r12t1 : r22 t2

Marketplace untuk Staners. Platform online yang menyediakan seluruh kebutuhan yang berkualitas
untuk (calon) mahasiswa, alumni, dan orang-orang yang mempunyai ketertarikan terhadap PKN STAN.

www.stanersstore.com @stanersstore stanersstore


KUNCI TRY-OUT 4

r12t1 : (4 r1) 2 t1

1:96

66. D
Bilangan kuadrat empat digit yg diawali dengan 18 hanya satu yaitu 1849, brarti
umur morgan akar 1849=43, dia lahir tahun 1849-43=1806.
67. D
Perbandingan senilai (sebanding)
24 → 108
14 → 63
Jadi hasilnya 63
68. B
31000 mod 4 = ((1*4-1)1000)) mod 4 = 1 mod 4 =1
69. B
Diketahui :
a = 1, b = -p, c = p + 1.
x1 - x2 = (√D) / a
⇒ (x1 - x2) a = √D
⇒ (x1 - x2) a= √(b2 - 4.a.c)
⇒ 1(1) = √(p2 - 4.1.(p + 1))
⇒ 1 = √(p2 - 4p - 4)
⇒ 1 = p2 - 4p - 4
⇒ p2 - 4p - 5 = 0
⇒ (p - 5)(p + 1) =
0 ⇒ p = 5 atau p = -1.

70. B
Yang akan dilapisi adalah dinding kamar. Berarti luas yang dicari adalah 2x(pxt)+2x(lxt)
yaitu
2 x5,3 x 4 + 2 x 3,7 x 4 = 72 dalam satuan m2. Sedangkan harga per cm2 adalah 1450.
Maka biaya total yang harus dikeluarkan Asep adalah 72 x 100 x 1450 = 10.440.000

65 buku hal 58 no 14
66 Bilangan kuadrat empat digit yg diawali dengan 18 hanya satu yaitu 1849, brarti umur
morgan akar 1849=43, dia lahir tahun 1849-43=1806
71. B
1
Adi = 6 𝑙𝑢𝑠𝑖𝑛 = 78 𝑏𝑢𝑎𝑕
2
1
Budi = 𝑔𝑟𝑜𝑠𝑠 = 36 𝑏𝑢𝑎𝑕
4
2
Bayu = 3 𝑘𝑜𝑑𝑖 = 68 𝑏𝑢𝑎𝑕
5

Marketplace untuk Staners. Platform online yang menyediakan seluruh kebutuhan yang berkualitas
untuk (calon) mahasiswa, alumni, dan orang-orang yang mempunyai ketertarikan terhadap PKN STAN.

www.stanersstore.com @stanersstore stanersstore


KUNCI TRY-OUT 4

Aji = 58 buah
Total = 240
Dibagi ke 24 anak, maka masing – masing anak mendapat 10 potong.
72. B
1024 jam 2025menit dan 3249 detik= 32 jam 45menit dan 57 detik = 32 × 60 +
45 + 57/60 = 1966
73. B
Upah 48000/kg.
Jika Tulus ingin mendapatkan upah sebesar 240,000. Maka ia harus mengangkut apl
sebanyak 240000/48000 = 5 gerobak atau 5 x 40 kg = 200 kg
74. C
2.750.000 + 102.000 + 3.374.600 + 1.899.000 + x = 9.235.000
x = 1.109.400
75. A

Banyaknya Pesanan
Januari 720
Februari 1,570
Maret 1,550
April 2,300
Mei 1,005
Juni 2,370
Juli 1,120
Agustus 2,275
September 1,245
November 2,750
Desember 2,700
Jumlah 19605

𝐽𝑢𝑚𝑙𝑎𝑕 19605
𝑅𝑎𝑡𝑎 − 𝑅𝑎𝑡𝑎 = = = 1782
11 11
76. C
Februari-Januari= 1,570 − 720 = 850
Agustus – Juli 2,275 − 1,120 = 1155
850
Persentase = × 100% = 73,91%
1150
77. D
2700
= 3,75
720
78. C
Penjualan Ternak Persentase
Tahun 2015

Marketplace untuk Staners. Platform online yang menyediakan seluruh kebutuhan yang berkualitas
untuk (calon) mahasiswa, alumni, dan orang-orang yang mempunyai ketertarikan terhadap PKN STAN.

www.stanersstore.com @stanersstore stanersstore


KUNCI TRY-OUT 4

AYAM 3250 29
BURUNG DARA 2350 21
BEBEK 1250 11
SAPI 2750 24
KAMBING 1700 15
Total 11300 100%
Median 20,7 = 20

79. D
Sapi= 1700 × 0,35 = 595
Total penjualan sapi = 2750 – 595 = 2155
80. C
11300
Rata-rata = = 2260
5
Yang melebihi rata-rata Burung dara dan sapi dengan jumlah = 21% + 24% = 45%
81. Pembahasan
Pada soal ini, pola deret yang digunakan adalah lompatandengan bilangan ganjil: 1,
3, 5, 7, 9 dan bilangan prima 2,3,5, 7,11

Jawaban: C
82. Pembahasan
Pada soal ini, pola deret yang digunakan adalah angka kuadrat 1,4, 9,16.dan angka
berurut dari 0, 1, 2, 3
Jawaban: B
.
83. Pembahasan
Pola deret pada soal ini adalah pengurangan angka 3 dan perubahan tanda + - : 18, -
15, 12 ,-9. serta penambahan angka 2 : -19, -17, - 15,-13
. Jawaban:
B

84. Pembahasan
123 = 11 2+ 22
153 = 12 2+ 3 2
2, 3, 5, 7, 11, 13 …adalah bilangan prima

Maka
346 = 152+ 112
Jawaban: D
85. Pembahasan
3, 5, 8, 13, 21, 34,…
Pola yang diterapkan adalah penambahan dua digit didepan angka

Marketplace untuk Staners. Platform online yang menyediakan seluruh kebutuhan yang berkualitas
untuk (calon) mahasiswa, alumni, dan orang-orang yang mempunyai ketertarikan terhadap PKN STAN.

www.stanersstore.com @stanersstore stanersstore


KUNCI TRY-OUT 4

8 = 3 +5
13 = 5 + 8,

34 = 13 + 21
55 = 21 + 34
89 = 34 + 55
Jawaban: A
86. Pembahasan
Q, E, N, G, K, I, …, …
Untuk soal aritmatik huruf, penulis biasa menkonversinya menjadi sebuah angka.
17, 5, 14, 7, 11, 9,8, 11
Polayang diterapkan adalah pengurangan 3 angka: 17, 14, 11, 9 serta bilangan ganjil mulai
dari 5 : 5, 7, 9, 11
Bila sudah ditemukan konversi kembali menjadi huruf. Huruf yang dimaksud adalah H dan K
Jawaban: C
87. Pembahasan

A, Q, I, O, U, M, E, …,

Pola yang dimaksud adalah mundur dua huruf : Q, O, M, K

Dan huruf vocal : A, I, U, E, O

Jawaban: B

88. Pembahasan

A, B, C, E, H,

1,2,3,5,8,

Pola yang dimaksud adalah penambahan dua digit sebelumnya. 1,2,3,5,8, 13,21

Seperti pola no 85. Maka huruf yang dimaksud adalah M dan U

Jawaban: A

89. Pembahasan

Z, A, Y, E, X, I, W, M

26, 1, 25, 5, 24, 9, 23, 13

Pola yang dimaksud adalah mundur satu huruf : Z, Y, X, W, V

Serta penambahan angka 4 : 1, 5, 9, 13, 17 (Q)

Marketplace untuk Staners. Platform online yang menyediakan seluruh kebutuhan yang berkualitas
untuk (calon) mahasiswa, alumni, dan orang-orang yang mempunyai ketertarikan terhadap PKN STAN.

www.stanersstore.com @stanersstore stanersstore


KUNCI TRY-OUT 4

Jawaban: E

90. Pembahasan

B,E, D,G, G, I, L

2, 5, 4, 7, 7, 9, 12

Pola yang dimaksud adalah penambahan dua digit : 5, 7, 9, 11

Serta penambahan bilangan prima : 2, 4, 7, 12, 19

Maka huruf yang dimaksud adalah K dan S

Jawaban: E

91. B
Trik mengerjakan soal-soal seperti ini adalah dengan memahami kalimat. Kemudian,
sesuaikan dengan pilihan jawaban yang ada. Sekarang pukul 11, Es meleleh pada 7
derajat. Ingat tiap satu jam lemari es naik 1 derajat. Maka es akan mencair 3 jam lagi dari
sekarang, yaitu pukul 14.00

Jawaban: B

92. A
“Sebagian” merupakan himpunan dari “semua”, maka bila “sebagian” melakukan
sesuatu berarti “sebagian” tersebut melakukan yang dikerjakan “semua”.
Jawaban: A
93. B
Jika hewan hidup di darat, maka ia tidak bisa hidup di air. Jika hewan hidup di air,
maka ia tidak bisahidup di darat. Jadi tidak ada hewan yang bisa hidup baik di darat
maupun di air.
Kedua pernyataan tidak beririsan.

Jawaban: B
94. A
Semua siswa pandai berbahasa Inggris. Siswa yang pandai berbahasa Inggris gemar
berdiskusi.Siswa yang gemar berdiskusi tidak pandai berdebat.Jadi, kesimpulannya
siswa tidak pandai berdebat, tetapi pandai berbahasa Inggris
Pada silogisme kategorik tipe 4 apabila premis mayor dan minor adalah universal
makakesimpulannya pasti partikular menggunakan kata sebagian atau beberapa.
Jadi jawaban C,dan E salah
Term menengah, gemar berdiskusi, tidak akan muncul di kesimpulan. Jadi jawaban B
dan Djuga kurang tepat

Marketplace untuk Staners. Platform online yang menyediakan seluruh kebutuhan yang berkualitas
untuk (calon) mahasiswa, alumni, dan orang-orang yang mempunyai ketertarikan terhadap PKN STAN.

www.stanersstore.com @stanersstore stanersstore


KUNCI TRY-OUT 4

Jadi jawabannya adalah A.

Jawaban: A
95. B
Tipe soal yang digunakan adalah silogisme.
p q
q
p,
sehingga jawaban yang tepat Baron meyirami kebunnya..

Jawaban: B
96. E
Pahami premis yang ada.
Jawaban: E
97. E
Karena Leli tidak membeli susu, berarti Leli tidak mendonorkan darahnya bulan ini.
Jawaban: E
98. E
Karyawan adalah pegawai tetap. Sebagian karyawan tinggal di asrama, kecuali
Bonar. Jadi, adakaryawan yang tinggal di asrama, dan ada yang tidak tinggal di
asrama. Dan meskipun Bonar tidaktinggal di asrama, namun Bonar tetap karyawan
juga. Sehingga Bonar juga pegawai tetap. Jadi,Bonar karyawan yang pegawai tetap,
meskipun tidak tinggal di asrama.
Jawaban: E
99. D
Metode penanganan kanker semakin modern. Sehingga jumlah penderita kanker
yang ditanganitahun 1980-an lebih baik daripada tahun 1950-an.
Fokus pembicaraan pada kata modern dan jumlah penderita kanker.

Jawaban: D
100. D
Semua pakaian musim dingin berbulu.Sebagian pakaian yang berbulu harganya
mahal, sebagian yang lain tidak mahal.Sehingga, sebagian pakaian musim dingin ada
yang mahal ada yang tidak.Sebagian pakaian yang mahal tidak mudah rusak,
sebagian yang lain mudah rusak.Sehingga, sebagian pakaian musim dingin ada yang
mudah rusak ada yang nggak mudah rusak.Jadi, kesimpulan yang tepat adalah
sebagian pakaian musim dingin mudah rusak.
Jawaban: D
101. B
Premis (1) Jika hari panas, maka Ani memakai topi.
Premis (2) Ani tidak memakai topi atau ia memakai payung.

Marketplace untuk Staners. Platform online yang menyediakan seluruh kebutuhan yang berkualitas
untuk (calon) mahasiswa, alumni, dan orang-orang yang mempunyai ketertarikan terhadap PKN STAN.

www.stanersstore.com @stanersstore stanersstore


KUNCI TRY-OUT 4

Premis (3) Ani tidak memakai payung.

p : Hari panas
q : Ani memakai topi
r : Ani memakai payung

Selesaikan terlebih dahulu premis (1) dan (2) kemudian digabungkan dengan premis (3)

Dari premis (1) dan (2)


Premis (1) Jika hari panas, maka Ani memakai topi.
Premis (2) Ani tidak memakai topi atau ia memakai payung.

p→q
~q ∨ r

Ingat bentuk berikut:


~q ∨ r ekivalen dengan q → r

sehingga bentuk di atas menjadi :


p→q
q→r
_____
∴ p → r (Silogisme)

Dari sini gabungkan dengan premis ketiga:


p→ r
~r
_____
∴ ~p (Modus Tollens)

Kesimpulan akhirnya adalah ~p yaitu "Hari tidak panas"

Jawaban: B
102. B
Silogisme hipotesis bentuk khusus
Ada sekolah yang menerima dan tidak menerima BOS.Pencairan dana BOS harus disertai
laporan keuangan.Artinya, sekolah yang dapat BOS pasti menyertakan laporan keuangan
untuk mencairkan dana.Jadi, Sekolah yang tidak menyerahkan laporan, belum dapat
mencairkan dana BOS
Jawaban: B
103. D

Marketplace untuk Staners. Platform online yang menyediakan seluruh kebutuhan yang berkualitas
untuk (calon) mahasiswa, alumni, dan orang-orang yang mempunyai ketertarikan terhadap PKN STAN.

www.stanersstore.com @stanersstore stanersstore


KUNCI TRY-OUT 4

Syarat untuk disebut penipu, adalah pandai bicara dan ramah. Perhatikan tuan M,
meskipun pandaibicara, tetapi tidak ramah. Sehingga karena tidak ramah, maka jelas
tuan M bukan penipu.
Jawaban: D
104. B
Penggunaan sabuk pengaman menurunkan angka kematian akibat kecelakaan lalu
lintas di jalanraya. Istirahat 15 menit saat perjalaan di jalan tol menurunkan
sebanyak 30 persen kematian akibatkecelakaan mobil. Jadi, angka kematian akibat
kecelakaan lalu lintas dapat diturunkan denganpenggunaan sabuk pengaman dan
istirahat 15 menit.
Jawaban: B
105. E Futsal
Tabel awal:
SELASA KAMIS SABTU
basket voli badminton renang futsal Atletik
A E B A F
B F F C
C
D
Pada soal ini C dan F ingin berolahraga bersama-sama. Karena faisal tidak memilih
atletik, maka ia pasti memilih futsal. Oleh karena itu, jika Cinta dan Faisal infin
berolahraga bersama, mereka harus memilih olahraga futsal
106. AATLETIK menyambung soal sebelumnya, Eliza akan mengambil olahraga atletik.
107. B atletik, basket, dan renang
Amir dapat mengambil basket, renang, dan satu lagi antara futsal atau atletik

108. B P,Q,U,S,R,T,V

P QU S R T V
109. B Jarak R ke T adalah 3 km
PV = 52 ; PQ = 8 ; QS = 9; ST = 15
Maka TV = PV – PT = 52 – (8 + 9 + 15) = 23
110. A Jarak U ke R adalah 19km
UR = US + SR
SR = 15 – 3 = 12
UR = 7 + 12 = 19
111. C Mark
RC ; PS; HR; SG
1 2 3 4 5 6 7
M H R C P S G

112. B Kedua

Marketplace untuk Staners. Platform online yang menyediakan seluruh kebutuhan yang berkualitas
untuk (calon) mahasiswa, alumni, dan orang-orang yang mempunyai ketertarikan terhadap PKN STAN.

www.stanersstore.com @stanersstore stanersstore


KUNCI TRY-OUT 4

Lihat tabel diatas


113. B Kedua
Lihat tabel diatas

114. D Carlos, ricky


Peter harus menyalip Carlos, kemudian menyalip Ricky
115. C
Pola yang sama antar gambar adalah jumlah kaki atas = jumlah kaki bawah.
Sedangkan D mempunyai jumlah kaki atas lebih sedikit satu daripada kaki bawah.

Jawaban: C

116. B
Pola yang sama antar gambar adalah didalam tiap persegi terdapat garis yang
membentuk suatu sudut / lekukan.
Sedangkan B hanya berupa garis lurus (tidak terdapat lekukan)

Jawaban: B

117. E
Analisis satu-satu.
A
gambar kiri: Bila bintang dan titik secara berturut-turut adalah sisi depan dan atas
kubus, maka
Gambar kanan : sisi kanan tidak mungkin membentuk garis bawah karena terdapat
bagian putih (kiri tanda panah) akan menutupi bintang.
B
Gambar kiri : bila titik adalah sisi atas kubus dengan bagian putih menjadi sisi
belakang kubus (bukan sisi depan kubus karena sisi depan kubus harus bintang)
maka
Gambar kanan : sisi depan kubus bintang dan sisi kanan kubus panah menghadap ke
samping kanan
C
Gambar kiri : Bila bintang adalah sisi depan kubus dengan bagian putih mengikuti
menjadi sisi kiri atau bawah maka
Gambar kanan : sisi atas kubus titik dan sisi kanan kubus panah menghadap ke kanan
D
Gambar kiri : bila bintang menjadi sisi depan kubus, sisi atas kubus bukan titik begitu
pula sebaliknya

Marketplace untuk Staners. Platform online yang menyediakan seluruh kebutuhan yang berkualitas
untuk (calon) mahasiswa, alumni, dan orang-orang yang mempunyai ketertarikan terhadap PKN STAN.

www.stanersstore.com @stanersstore stanersstore


KUNCI TRY-OUT 4

E
Gambar kiri : bila bintang adalah sisi depan kubus dan titik adalah sisi atas dengan
kata lain bagian putih menjadi sisi belakang kubus maka
Gambar kanan: sisi kanan kubus panah menghadap ke bawah dengan bagian putih
mengikuti menjadi sisi bawah kubus

Jawaban: E

118. C
Bila diperhatikan garis pada sisi atas kubus dan sisi depan kubus bertemu. Maka
jawaban A, B salah.
Sedangkan D sisi kanan kubus berupa panah menghadap ke bawah bukan kesamping
depan. Maka jawaban yang tepat C

Jawaban: C

119. B
Analisis satu-satu
A
Bila diperhatikan dari jarring-jaring sisi dengan garis hitam yang tidak penuh tidak akan
menyentuh/ bertemu dengan garis silang (X) pada sisi sebelahnya
C
Asumsikan panah menghadap sisi yang benar, maka titik akan berada pada bagian kiri
bawah dari sisi bawah (bukan kanan bawah)
D
Asumsikan panah menghadap sisi yang benar, maka sisi atas kubus harusnya titik
E
Bila diperhatikan sisi dengan garis hitam yang tidak penuh tidak akan menyentuh/
bertemu dengan lingkaran pada sisi sebelahnya.
Jawaban: B

120. A
Karena semua pilihan jawaban mempunyai sisi depan (serong kiri) yang sama,
bayngkan anda memposisikan sisi dengan dua garis tebal tipis pada tempat yang
sesuai. Lalu lipat hingga bagian atas dapat menjadi sisi atas kubus, maka jawaban
yang tepat adalah A
Jawaban: A

Marketplace untuk Staners. Platform online yang menyediakan seluruh kebutuhan yang berkualitas
untuk (calon) mahasiswa, alumni, dan orang-orang yang mempunyai ketertarikan terhadap PKN STAN.

www.stanersstore.com @stanersstore stanersstore


KUNCI TRY-OUT 4

KUNCI JAWABAN TBI

1. A. kegiatan masa lampau yang seharusnya sudah selesai dilaksanakan


2. C. merupakan simple present
3. B. passive voice
4. C. conditional sentence yang akan terjadi
5. D simple present kegiatan yang akan dilakukan dengan kemungkinan yang belum pasti
6. C yang dibutuhkan adalah adjective
7. A kegiatan yang telah terjadi namun kegiatannya masih dirasakan saat ini.
Is instructed
8. A yang dibutuhkan adalah adjective
9. C yang dibutuhkan adalah adverb
10. D yang dibutuhkan adalah adverb
11. B. merupakan simple present tense karena subjek adalah singular maka verb yang tepat
adalah flows
12. B. The Apollo 11 adalah subject sehingga yang tepat adalah whom
13. A. kalimat conditional. Setelah were maka langsung diikuti subjek, ditulis dalam past
continuous
14. B. kalimat fakta ditulis dalam simple present tense
15. D. jika suatu kalimat menggunakan neither maka parallel words yg digunakan adalah
nor diikuti dengan word/phrase dengan pola sama
16. B. karena di kalimat setelah koma digunakan bentuk comparative, maka kalimat pertama
juga harus menggunakan bentuk ini
17. A. subjek kalimat berbentuk singular
18. C. subjek adalah plural maka pronoun yang tepat adalah their
19. D. objek kalimat ini adalah answered question (adj. + noun) maka harus ditambahkan
satu kata di depan adjective yang harus dalam bentuk adverb
20. B. bentuk kalimat past continuous
21. A. money adalah uncountable noun
22. B. keterangan tempat harus langsung diikuti dengan kata kerja subjek
23. A. verb harus dalam bentuk past principle karena merupakan passive form
24. A. kalimat conditional dengan would maka kalimat sebelumnya ditulis dalam bentuk
past tense
25. D. kalimat simple present dengan kata kerja are maka subjek harus plural
26. D. terdapat dua verb sehingga dibutuhkan kata penghubung
27. C. nowhere = negative expression. Maka negative expression + verb + subject
28. D. verb adalah kata have sehingga subjek harus berbentuk plural
29. B. bentuk comparative, struktur haruslah sama
30. C. jika kalimat setelah kata penghubung when menggunakan bentuk simple past maka
kalimat sebelumnya harus bentuk past perfect
31. A. frasa yang berada diantara dua koma hanya berguna sebagai keterangan. Subjek
adalah Henry Adams maka dibutuhkan verb
32. C. kalimat ini membutuhkan verb dari subjek plural
33. B. kalimat ini membutuhkan verb
34. B. frasa setelah kata penghubung harus dalam bentuk yang sama dengan sebelum kata
penghubung
35. B. berbentuk comparative

Marketplace untuk Staners. Platform online yang menyediakan seluruh kebutuhan yang berkualitas
untuk (calon) mahasiswa, alumni, dan orang-orang yang mempunyai ketertarikan terhadap PKN STAN.

www.stanersstore.com @stanersstore stanersstore


KUNCI TRY-OUT 4

36. D. kalimat passive form


37. C. subjek kalimat ini harus berbentuk noun
38. A. membutuhkan pronoun
39. B. kata kerja as harus diikuti oleh adverb
40. D. kata bottles harusnya didahului oleh adjective
41. D. Bentuk dari embedded question dalam pernyataan adalah :
question word + subject + verb
Seharusnya adalah what street was on
42. C. simple verb dipakai untuk beberapa kata berikut ini:demand, insist, require,
suggest, recommend. Subjuctive.
Seharusnya adalah repair
43. A. Article "an" digunakan sebelum kata yang diawali dengan vowel atau vowel
sound( bunyi hidup)
Seharusnya adalah a
44. D. Penggunaan kata yang di paralel-kan harus mempunyai bentuk gramatikal
yang sama. kata "fiercelly merupakan adverb sehingga tidak paralel dengan
kata dreamy dan brave
45. B. Penggunaan subject utamanya adalah "a body". seharusnya has been
46. A. Penggunaan Dangling modifier. Seharusnya seriously having burned in a
terrible car accident, ...
47. B. Penggunaan object dalam kalimat. Seharusnya him
48. C. Bab preposition in combination. Seharusnya of
49. C. Penggunaan bentuk passive voice dalam kalimat.
Form Bentuk pasif
Be + V-3 (Past Participle)
untuk kalimat dengan bentuk perfect tense, maka bentuk form-nya adalah S +
have + been + V-3 (past participle) seharusnya have been doubled
50. B. Bentuk negative not + to + V base. Seharusnya adalah not to feed
51. C. because this passage is discussing about newer fertilization technology after embryo
banks, so the most probable preceding paragraph is discussing about embryo bank.
52. B. clearly stated in line 1
53. D. according to words “Britain itself” in line 3
54. B. even though this passage mostly discuss about technology in medicine, but in
paragraph 2 we can see the pro contra that lead to a moral value. This kind of passage
usually appears as a headline in daily newspaper.
55. D. as stated in line 13
56. B. the passage is mainly discussed about The Mary Celeste
57. A. “leading up to” has the closest meaning to directing
58. D. as stated in paragraph 2
59. A. that can be seen from the departure date and the date the Mary Celeste found
60. C. as inferred in line 2

Marketplace untuk Staners. Platform online yang menyediakan seluruh kebutuhan yang berkualitas
untuk (calon) mahasiswa, alumni, dan orang-orang yang mempunyai ketertarikan terhadap PKN STAN.

www.stanersstore.com @stanersstore stanersstore


Paket 5
Perhatian!
1. Untuk semua soal, pilihlah satu jawaban yang paling tepat dari pilihan yang tersedia.
2. Jawaban benar bernilai 4 (empat); jawaban salah bernilai -1 (minus satu); tidak menjawab bernilai 0 (nol).
3. Nilai mati berlaku pada setiap bagian soal. Anda memperoleh nilai mati jika pada salah satu dari bagian soal
jawaban benar yang Anda peroleh kurang dari 1/3 jumlah soal pada bagian tersebut.

BAGIAN PERTAMA
TES POTENSI AKADEMIK
[ Nomor 1 s.d. 120]

Untuk bagian ini, jawaban benar kurang dari 1/3 dari jumlah soal (kurang dari 40) soal berarti nilai mati.

KOSAKATA & SINONIM A. Pembukaan


B. Penutup
1. RESIDU=…
C. Kebun Binatang
A. Sisa
D. Kandang Kuda
B. Hasil
8. LEUKIMIA
C. Polusi
A. Kanker Darah
D. Gas
B. Darah Rendah
2. APOLOGI=…
C. Kanker Otak
A. Pemujaan
D. Pendarahan Otak
B. Mengheningkan Cipta
ANTONIM
C. Penyucian
9. MAKAR >< …
D. Pembelaan
A. Muslihat
3. EFEKTIF=…
B. Jujur
A. Berhasil Guna
C. Menakjubkan
B. Tepat Guna
D. Mengeluh
C. Efisien
10. MONOLOGIS >< …
D. Tidak sia-sia
A. Sendirian
4. EBI=…
B. Bersama
A. Ayam Goreng
C. Egois
B. Sup Bebek
D. Kamu
C. Udang Kering
D. Cumi-Cumi
11. SPORADIS >< …
A. Kerap
5. SIE QUA NON=…
B. Jarang
A. Tandus
C. Kaku
B. Tiada Berair
D. Keras
C. Banjir
12. KOORDINATIF >< …
D. Gambut
A. Teratur
B. Amburadul
6. EFISIEN=…
C. Rapi
A. Berhasil Guna
D. Terarah
B. Tepat Guna
C. Efektif
13. SEKULER
D. Tidak sia-sia
A. Keagamaan
7. PREAMBUL=…
B. Duniawi

1
Paket 5
C. Serikat
D. Simpati 20. AUDISI : ARTIS
14. REGRESIF A. Ospek : Mahasiswa
A. Maju B. Training : Pegawai
B. Baik C. Wisuda : Mahasiswa
C. Cemerlang D. Wejangan : Santri
D. Parah 21. KIJANG : CEPAT
15. HIGIENIS A. Komputer : Kantor
A. Sakit B. Bola : Kaki
B. Sehat C. Siput : Lambat
C. Baik D. Topi : Kepala
D. Bersih 22. SEPAK BOLA : OFF SIDE
A. Pesta : Dugem
ANALOGI
B. Makan : Tersedak
16. SHAMPO : KEPALA C. Ekonomi : Resesi
A. Cat : Tembok D. Hidup : Pingsan
B. Tinta : Pulpen 23. HURUF : KAPITAL
C. Ilmu : Otak A. Bus : Eksekutif
D. Air : Mata B. Kereta : Bisnis
17. YOGYA : BUDAYA C. Atlet : Sumo
A. Perut : Usus D. Komputer : Intel
B. Jepang : Salju 24. ABONEMEN : TELEPON
C. Bogor : Hujan A. Tips : Turis
D. Kampus : Pegawai B. Pajak : Motor
18. CASTING : ARTIS C. Karcis : Peron
A. Belajar : Juara Kelas D. Upah : Buruh
B. Pacaran : Kekasih
C. Kualifikasi : Posisi Start 25. PROYEK : PROPOSAL
D. Wawancara : Karyawan A. Pergi : Izin
19. DATA : DISKET B. Sekolah : Biaya
A. Ikan : Kolam C. Makan : Lapar
B. Uang : Dompet D. Bertengkar : Emosi
C. Lagu : Pita
D. Mimpi : Tidur

Soal nomor 26-30 berdasarkan bacaan berikut.


Presiden Susilo Bambang Yudhoyono menyatakan tidak akan ada reshuffle kabinet dalam
100 hari pemerintahannya. Hal ini dijelaskan Juru Bicara Presiden, Julian Aldrin. Soal evaluasi kerja,
kata dia, sejauh ini masih belum dibicarakan dan tidak akan terjadi reshuffle pada 100 hari masa
kerja SBY. “Tidak ada karena ini baru 100 hari.”, ujarnya. Kendati demikian, Presiden SBY akan
mengejar para petinggi negara untuk meminta target yang telah dicapai dan tindak lanjutnya.
“Presiden meminta untuk departemen dan menteri menindaklanjuti target-target yang memang
akan dicapai.”, terang Julian. Dia menambahkan, pada pidato pembukaan acara penyerahan Daftar
Isian Pelaksanaan Anggaran (DIPA), Presiden juga sempat menyinggung soal keberhasilan program
yang tidak dapat dilihat dari 100 hari masa kerja tetapi yang sekarang dapat dilihat progresnya. “Ada
15 program unggulan sebagai prioritas yaitu seperti dibentuknya lembaga mafia hukum dan
penuntasan terorisme. Itu sebagai bukti konkret progres yang baik.”, ujar Julian.
(disadur dari okezone.com)
26. Yang dimaksud reshuffle dalam bacaan adalah
A. Penghentian sementara

2
Paket 5
B. Perubahan susunan
C. Pendaftaran kembali
D. Penggantian kabinet
27. Menurut bacaan, siapa Juru Bicara Presiden saat itu?
A. Andi Malarangeng
B. Andi Mattalata
C. Julian Aldrin
D. Aburizal Bakrie
28. Pernyataan berikut yang sesuai dengan bacaan adalah …
A. Sebenarnya ada rencana reshuffle kabinet dalam program 100 hari masa kerja SBY
B. Presiden SBY membentuk lembaga mafia hukum
C. Para menteri meminta Presiden untuk menindaklanjuti target yang akan dicapai
D. Presiden SBY belum sempat me-reshuffle cabinet karena belum terselesaikannya kasus Bank
Century.
29. Pernyataan berikut yang tidak sesuai dengan bacaan adalah …
A. Presiden SBY menyatakan tidak akan ada reshuffle kabinet dalam 100 hari masa kerjanya.
B. Presiden meminta target yang telah dicapai oleh petinggi negara
C. Penuntasan terorisme merupakan program yang tidak dapat dilihat dari 100 hari masa kerja
tetapi sekarang dapat dilihat progresnya
D. SBY telah melakukan evaluasi kerja seperti keberhasilannya dalam menumpas teroris
30. Pertanyaan yang jawabannya terdapat dalam bacaan adalah …
A. Mengapa tidak ada reshuffle kabinet dalam 100 hari pemerintahan Presiden SBY?
B. Apa saja 15 program unggulan Presiden SBY?
C. Apa saja hal-hal yang dilakukan SBY pada 100 hari pemerintahannya?
D. Bagaimana corak pemerintahan Presiden SBY dilihat dari 100 hari masa kerjanya?

Untuk Soal nomor 31-35 berdasarkan bacaan berikut.


Teleskop Keppler milik NASA menemukan lima eksoplanet baru. NASA menyebutkan, kelima
eksoplanet yang ditemukan tersebut kemudian diberi kode Keppler 4b, 5b, 6b, 7b, dan 8b. Menurut
NASA, ditemukannya planet-planet tersebut agaknya menjadi pembenaran bahwa teleskop Keppler
merupakan perangkat yang canggih untuk menemukan planet lain yang mirip Bumi. NASA pun
optimis teleskop tersebut bisa mencapai target yang ingin dicapai dalam bidang sains. Misi Keppler
NASA yang dimulai pada 6 Maret tahun lalu dirancang untuk meneliti lebih dari 150000 bintang guna
menemukan planet serupa Bumi. Dalam pernyataannya kepada public, seperti dikutip dari CNET,
NASA menjelaskan bahwa misi Keppler pada akhirnya memiliki tujuan untuk menentukan apakah
kita benar-benar “sendiri” di alam semesta yang luas ini? NASA sangat optimis, Keppler hanya
memerlukan waktu kurang dari tiga tahun untuk menemukan planet-planet seperti Bumi.
(disadur dari okezone.com)

31. Yang dimaksud eksoplanet dalam bacaan


adalah …
32. Pernyataan berikut yang sesuai dengan
A. Planet kecil yang tidak layak disebut
bacaan adalah …
sebagai planet
A. Teleskop Keppler mampu
B. Planet diluar Bumi yang mengorbit
menemukan 5 eksoplanet baru yang
matahari di dalam system tata surya
diberi kode Hubbler 4b, 5b, 6b, dan 7b
C. Planet yang mengorbit bintang selain
B. Teleskop keppler adalah perangkat
matahari diluar sistem tata surya
canggih yang digunakan untuk
D. Planet diluar sistem tata surya yang
mencari planet lain yang mirip bumi
mengorbit matahari

3
Paket 5
C. Teleskop Keppler telah mampu A. 175 C. 275
menemukan planet mirip Bumi. B. 225 D. 125
D. Teleskop Keppler mencari planet 40. Semua tempat duduk sebuah busway
dengan cara mengukur tingkat telah penuh dan ada 5 orang yang
kecerahan cahaya yang dipantulkan berdiri. Pada halte berikutnya ada 12
sebuah bintang orang yang turun, tetapi ada pula 5
33. Jika misi Keppler dimulai pada 6 Maret orang yang naik. Berapa jumlah
2009, maka NASA optimis menemukan tempat duduk yang kosong apabila
planet mirip Bumi pada tahun … semua orang telah duduk ?
A. 2012 A. 4 C. 2
B. 2013 B. 3 D. 1
C. 2014 41. Berapa lama waktu yang diperlukan
D. 2015 untuk mengisi penuh tangki yang
34. Pertanyaan yang jawabannya terdapat berkapasitas 3.750 cm3 jika air
dalam bacaan adalah … tersebut dipompakan masuk dengan
A. Kapan teleskop Keppler mulai dibuat? debit 800 cm3/minute dan dialirkan
B. Apakah kita benar-benar “sendiri” di keluar sebesar 300 cm3/minute ?
alam semesta yang luas ini? A. 7,5 menit C. 3,6 menit
C. Sampai saat ini sudah berapa planet B. 8 menit D. 6 menit
yang ditemukan Keppler? 42. 3 * 7 ^ 2 [1]
D. Apakah tujuan dari misi Keppler? 4 * 8 ^ 3 [2]
35. Pola kalimat pertama dalam bacaan N * 9 ^ 3 [6]
adalah … Berapa nilai N ?
A. SPOK A. 4 C. 6
B. SPK B. 5 D. 7
C. SPO 43. Jika diberikan -5 < p < 8 dan -1 < q < 5,
D. SPpel maka ?
A. 0 < p2 + q2 < 89
KEMAMPUAN KUANTITATIF
B. 0 < p2 + q2 < 89
36. Ada 3 perempatan jalan yang C. 26 < p2 + q2 < 89
mempunyai lampu merah. Lampu D. 25 < p2 + q2 < 89
tersebut menyala bersama pukul 44. Harga Laptop dinaikkan 25 % dari
08.15 WIB. Lampu pertama menyala harga sebelumnya. Persentase
tiap 12 menit, lampu kedua menyala penurunan harga laptop kembali
tiap 16 menit, dan lampu ketiga seperti sebelumnya adalah ?
menyala tiap 24 menit. Pada pukul A. 20 % C. 30 %
berapa ketiga lampu akan menyala B. 25 % D. 35 %
secara bersama-sama lagi ? 45. Yahyo meninggalkan kota A pukul
A. 09.03 WIB C. 10.03 WIB 06.20 . Dia tiba di kota B pukul 11.20 .
B. 09.43 WIB D. 10.13 WIB Jika Yahyo mengendarai motornya
dengan kecepatan 35 km/h dan
beristirahat selam 1 jam di perjalanan,
37. 35% dari X adalah 42. Dan, 5/6 dari Y tentukan jarak kota A dan B !
adalah 25. Berapa X + Y ? A. 150 km C. 140 km
A. 67 C. 90 B. 130 km D. 125 km
B. 145 D. 150
38. Jumlah bilangan kubik dan bilangan 46. Sebuah angka dikatakan sempurna
kuadrat antara 100 dan 200 adalah ? apabila sama dengan jumlah seluruh
A. 568 C. 634 faktor positifnya yang kurang dari
B. 630 D. 755 angka itu sendiri. Manakah dari
39. a + b = 30. Hasil kali maksimum a dan
b adalah ?

4
Paket 5
angka-angka berikut yang dapat B. X < Y D. Tidak ada
dikatakan sempurna ? 56. Berapakah bilangan bulat terbesar
A. 1 C. 6 yang merupakan penjumlahan dari
B. 4 D. 8 tiga bilangan prima berbeda yang
47. Bilangan yang dapat habis dibagi masing-masing besarnya tidak lebih
dengan 3 dan 88 adalah ? dari 53?
A. 888.888 C. 88.888.888 A. 143 C. 151
B. 88.888 D. 8.888.888 B. 147 D. 153
48. Diketahui x dan p masing-masing adalah 57. Sebuah bola karet dilepas dari
suatu bilangan bulat dan p terletak ketinggian 1 meter. Setelah
diantara 16 dan 35. Sedangkan x = menyentuh tanah, bola mamentul
(p+2)/8 . Banyaknya harga x yang kembali setinggi 7/8 kali tinggi awal.
memenuhi syarat adalah ? Jika pantulan selanjutnya memenuhi
A. 0 C. 2 7/8 dari tinggi pantulan sebelumnya,
B. 1 D. 3 berapa tinggi pantulan bola setelah
49. Tito membeli tabloid 4 hari sekali. Adi pantulan ke-5 ?
membeli tabloid 7 hari sekali. Kemarin A. 51,29 C. 45,3
Tito membeli tabloid. Adi akan B. 0,2 D. 55,3
membeli tabloid besok. Keduanya 58. Sebuah wadah berbentuk silinder diisi
paling cepat akan membeli tabloid air sebanyak 1/3 nya. Lalu wadah
ladi pada hari yang sama ... hari lagi. tersebut ditambah air 5 liter, sehingga
A. 28 C. 16 isinya menjadi 1.5 nya. Berapa
B. 39 D. 71 kapasitas wadah tersebut ?
50. 8 [16] 4 A. 18 liter C. 30 Liter
6 [3] 12 B. 12 Liter D. 24 Liter
P [27] 3 59. Berapa hektarkah 760000 m2 + 3000
Berapakah nilai dari P? dam2 +
A. 5 C. 9 400 are + 950000 m2 + 150000000 cm2
B. 7 D. 11 A. 220 hektar
51. Nilai terdekat dari 0,250/0,333 dibagi B. 206,5 hektar
0,125/0,167 adalah ? C. 20254 hektar
A. 5 C. 10 D. 20650 hektar
B. 1 D. 0,667 60. Ciblek dan Septian berangkat latihan
52. Jika (x-1)/(x+1) = 4/5, maka x = karate bersama-sama pada tanggal 17
A. 4 C. 12 Agustus. Ciblek latihan 4 hari sekali,
B. 9 D. 13 sedangkan Septian latihan 6 hari sekali.
53. Andi dapat mengisi kolam ikan dalam Tanggal berapa mereka berangkat
waktu 30 menit. Bedu dapat bersama untuk keempat kalinya?
melakukan hal yang sama dalam A.
waktu 45 menit, sedangkan catur 4 Oktober
hanya bisa menyelesaikan dalam B. 22 September
waktu 1,5 jam. Berapa menit yang C. 10 September
dibutuhkan bila mereka bekerja D. 28 Oktober
bersama-sama untuk mengisi kolam? 61. Robert mengendarai mobil barunya dari
A. 21 B. 23 C. 12 D 15 posisi A kemudian bergerak 600 m ke arah
54. 6,5 = berapa % dari 6 ? utara, 800 m ke arah timur, 300 m ke
A. 108,33 C. 8,33 selatan, 400 m ke barat. Berapa jarak
B. 107,67 D. 109,31 posisi akhir Robert diukur dari posisi
55. Jika -2<x<7 dan 4<y<9, maka awal?
hubungan x dan y adalah? A. 200 m
A. X > Y C. X = Y B. 300 m

5
Paket 5
C. 400 m 67. 27% + 5/4 – 10/32 x 64% - 27/36 =
D. 500 m A. 0,57
62. Jika z adalah 61% dari 37,2 dan y adalah B. 0,02
37,2% dari 61, maka C. 0,32
A. z > y D. 0,64
B. z < y 68. Di perusahaan Krasti Krep, 8 orang
C. z = y pekerja dapat membuat Krebi Peti raksasa
D. hubungan z dan y tidak dapat dalam waktu 7 hari. Bila pekerjanya
ditentukan bertambah 6 orang, lebih cepat berapa
63. Jika x + y + z = 24 dan x = z, y ≠ 0 maka harikah pekerjaan itu dapat diselesaikan?
A. x = 12 A. 10 hari
B. x < 12 B. 6 hari
C. x > 12 C. 4 hari
D. nilai x tidak dapat ditentukan D. 3 hari
64. Jika z = a2 + b2 – c2 dan y = b2 + (a + b) x (a 69. Di suatu kelas terdapat 30 anak. Nilai
– b) ujian Statistika kelas tersebut
+ (b – a) x (b + a) + (a – c) x (a + c) , maka dikelompokkan sebagai berikut. Terdapat
A. z > y 13 anak yang nilainya diantara 61 – 70.
B. z < y 10 anak yang nilainya 71 – 80. 5 anak yang
C. z = y nilainya 81 – 90. Dan sisanya 91 – 100.
D. hubungan z dan y tidak dapat Berapakah Median nilai ujian Statistika di
ditentukan kelas tersebut?
65. Dari 100 perusahaan di Kota Tahu, 40 A. 71
diantaranya menggunakan micro B. 71,5
komputer, sedangkan 30 perusahaan C. 72
menggunakan personal komputer. Dari D. 72,5
perusahaan tersebut 20 diantaranya 70. Afif menjual permata senilai
menggunakan keduanya. Berapa Rp472.500,00. Dengan harga itu dia
probabilitas bahwa pengambilan secara memperoleh mark up sebesar 35% dari
acak akan terambil sebuah perusahaan harga beli. Berapa harga beli permata
yang hanya menggunakan salah satu dari tersebut?
personal komputer saja atau micro A. Rp637.857,00
komputer saja? B. Rp165.375,00
A. 0,1 C. Rp350.000,00
B. 0,2 D. Rp375.165,00
C. 0,3
D. 0,4
71. Jika p*q = p-q2 maka nilai dari 2*(3*(5*1))
adalah
66. Sebuah tongkat sepanjang 6,5 meter A. 167
disandarkan pada sebuah dinding tembok. B. – 167
Bagian paling bawah tongkat berjarak 2,5 C. – 169
meter dari dinding. Jika bagian paling atas D. 169
tongkat digeser ke bawah sepanjang 0,8 72. Rata-rata nilai ujian Ekonomi kelas 1-Z
meter, berapa meterkah bagian paling yang berjumlah 28 anak adalah 67. Jika
bawah tongkat tersebut akan bergeser nilai 1 anak dari kelas 1-K dimasukkan,
dari posisi semula? maka rata-ratanya menjadi 68. Berapa
A. 0,4 nilai anak tersebut?
B. 0,8 A. 69
C. 0,2 B. 76
D. 0,6 C. 84

6
Paket 5
D. 96 geometri dengan rasio 3. Berapa jumlah
73. 84 + 84 + 84 + 84 = 4n . Berapakah nilai n? ketiga bilangan aritmatika tersebut?
A. 6 A. 26
B. 7 B. 28
C. 8 C. 30
D. 12 D. 32
74. Enam karyawan PT. NATS yang lulus masa 80. Suatu deret aritmatika: 3, 9, 15, 21, 27, …
percobaan, tiga diantaranya akan Suku ke-27 dalam deret tersebut adalah …
ditempatkan di bagian gudang. Berapa A. 5
kemungkinan susunan yang terjadi? B. 159
A. 120 C. 183
B. 60 D. 201
C. 40
KEMAMPUAN PENALARAN
D. 20
75. Dicky dapat menyelasaikan soal usm 81. Di zaman modern ini, peradaban
dalam waktu 120 menit. Sedangkan manusia semakin terdistorsi. Dugem
Zaenuri dapat menyelesaikan soal yang dan night club adalah salah satu dari
sama hanya dalam waktu 80 menit. peradaban modern.
Jika mereka bekerja sama berapa menit A. Dugem dan Night Club adalah
mereka dapat menyelesaikan 1/3 soal salah satu penyebab hancurnya
tersebut? moral.
A. 24 B. Dugem dan Night Club adalah
B. 26,7 ujian bagi orang beriman.
C. 33,3 C. Dugem dan Night Club baik untuk
D. 66,7 peradaban modern.
76. Berapa jumlah bilangan prima antara 0 – D. Di zaman modern ini, tidak sulit
20? untuk menemukan night club dan
A. 77 tempat dugem.
B. 78 82. Setiap karyawan seharusnya memiliki
C. 75 sikap profesionalisme yang tinggi.
D. 102 Masalah keluarga dan sakit menjadi
77. Berapa perbandingan antara volume faktor penghambat profesoinalisme
Kubus (yang mempunyai rusuk n cm) kerja.
dengan volume tabung yang ada di A.
dalamnya? Setiap pegawai tidak bisa
A. 1 : 4π menghindar dari sakit.
B. 4 : 1π B. Walaupun sedang sakit,
C. 1 : π profesionalisme dalam bekerja
D. π : 1 harus tetap dijaga.
78. Lynx membeli 3 rumah dan 5 mobil C. Broken home dan sakit dapat
seharga £15.375. Jika ia membeli 2 rumah menjadi penghambat kerja
dan 7 mobil, biaya yang dikeluarkannya seseorang.
adalah £15.475. Jika Lynx mempunyai D. Pegawai akan mengalami sakit
uang £17.575, berapa sisa uangnya bila ia yang lumrah.
hanya membeli 1 rumah dan 1 mobil?
A. £14.575 83. Waktu di New York 7 jam lebih cepat
B. £13.400 dari waktu di Jakarta. Jika perjalanan
C. £12.325 Puji memakan waktu 12 jam, dan Puji
D. £12.500 tiba di New York pukul 19.00, pukul
79. Tiga bilangan aritmatika jika suku berapa di Jakarta saat Puji berangkat?
tengahnya dikurangi 4 menjadi deret A. 00.00

7
Paket 5
B. 01.00 D. Austria & Swiss
C. 02.00
D. 03.00 89. Jika 1+2 = 0, 2 + 3 = 0, 3 + 4 = 0, 4 + 5
84. Pendidikan harus berorientasi kepada = 0, 5 + 6 = 1, 6 + 7 = 1, 7 + 8 = 2.
pencerdasan dan peluang kerja. Berapa 8 + 10 ?
Kuliah di Akademi “X” saat ini sedang A. 0 C. 2
lesu. B. 1 D. 3
A. Akademi X membuka peluang 90. Kristen lebih tua dari Islam. Taoisme
kerja. lebih muda dari Budha. Padahal
B. Kuliah di Akademi X tidak Kristen lebih muda dari Taoisme.
menjamin mendapatkan peluang Agama apa yang paling tua?
kerja. A. Islam
C. Pendidikan tidak selalu terkait B. Buddha
masalah pekerjaan. C. Kristen
D. Pendidikan menentukan D. Konghuchu
pekerjaan. 91. Setiap hari Senin, Jebe pergi ke pasar.
85. Semua hasil tambak bisa Hari ini Jebe tidak pergi ke pasar.
diperbaharui. Terasi adalah hasil A. Hari ini hari Senin
tambak. B. Hari ini, kaki Jebe sakit
A. Terasi tidak dapat diperbaharui. C. Jebe ada keperluan lain
B. Terasi dapat diperbaharui. D. Hari ini bukan hari senin
C. Terasi adalah budi daya tambak.
D. Terasi enek lho!! 92. Dicka tidak lebih Muda dari Dhiko.
Dhika tidak lebih muda dari Dicky. Jadi
86. Semua Mahasiswa STAN yang sudah ...
lulus langsung ditempatkan kerja. A. Dicky paling muda.
Geoniko adalah mahasiswa STAN. B. Dhika dan Dhiko saudara kembar
A. Geoniko langsung ditempatkan C. Dicka dan Dicky berusia sama
kerja. D. Mungkin keempatnya berusia
B. Geoniko sudah lulus. sama.
C. Geoniko belum ditempatkan
kerja.
D. Geoniko menganggur. Untuk mengarjakan soal no. 93-98
Perhatikan informasi berikut:
87. Dalam Euro 2004, Protugal Di suatu kota terdapat satu toko bangunan
mengalahkan Ceko 3-2. Sedangkan terkenal. Di sana ada 11 karyawan bernama
Perancis dikalahkan Yunani 1-4. Di
Adi, Beny, Ciblek, Dhika, Eko, Fuji, Gestald,
Babak final, Yunani mengalahkan
Portugal 3-2. Siapa yang menjadi Hendi, Ika, Jali, dan Kukuh. Satu karyawan
juara? hanya menjaga satu hari. Toko buka setiap
A. Yunani hari senin s.d. jumat. Hanya ada 1 hari yang
B. Portugal penjaganya tiga. Jadwal jaganya adalah :
C. Perancis - Ciblek hanya jaga hari Selasa
D. Wasit - Gestald tidak mau jaga bersama Jali
88. Pada Euro 2008, Rusia mengalahkan - Eko dan Adi tidak jaga pada hari Senin
Belanda, Spanyol mengalahkan Rusia. - Beny, Dhika dan Fuji jaga bersama
Dan Swiss mengalahkan Austria. Siapa - Hendi jaga hari jumat
tuan rumah Euro 2008? - Kukuh jaga bersama Adi
A. Rusia - Jali jaga pada hari Kamis
B. Portugal - Ika dan Eko tidak jaga hari Selasa
C. Italia

8
Paket 5
93. Siapa yang menjaga toko pada hari 100. Jika sisi Hijau berada di bawah, sisi
Rabu? apakah yang berada di atas?
A. Beny A. Biru
B. Kukuh B. Merah
C. Ika C. Oranye
D. Gestald D. Kuning
94. Hari apakah yang diisi tiga penjaga? 101. Jika sisi Hijau ditukar dengan sisi Putih
A. Jumat dan sisi Biru ditukar dengan sisi Oranye.
B. Rabu Pernyataan manakah yang salah?
C. Selasa A. Sisi Putih di sebelah sisi Kuning
D. Senin B. Sisi Merah di sebelah sisi Hijau
95. Hari apakah Adi jaga toko? C. Sisi Biru di sebelah sisi Hijau
A. Rabu D. Sisi Merah berhadapan dengan sisi
B. Kamis Oranye
C. Jumat 102. Jika sisi Putih ditukar dengan sisi Kuning,
D. Senin sisi Oranye ditukar dengan sisi Hijau,
96. Hari apakah Eko jaga toko jika Ika dan sisi Merah ditukar dengan sisi Hijau.
jaga pada hari Kamis? Pernyataan manakah yang benar?
A. Selasa A. Sisi Putih di sebelah sisi Kuning
B. Rabu B. Sisi Merah berhadapan dengan
C. Kamis sisi Hijau
D. Jumat C. Sisi Biru di sebelah sisi Hijau
97. Siapa yang menjaga toko bersama D. Sisi Oranye di sebelah sisi Hijau
Ciblek?
A. Adi
B. Gestald
C. Kukuh
D. Eko
98. Siapa yang menemani Jali apabila
Hendi jaga bersama Ika? Untuk mengarjakan soal no. 103 - 105
A. Eko Perhatikan informasi berikut:
B. Adi
C. Gestald Ada suatu perlombaan lari yang diikuti oleh
D. Kukuh lima orang, diantaranya Ittaqih, Jamal, Randy,
Risky, dan Yoga.
Untuk mengerjakan soal no. 99 - 102 Piala akan diberikan kepada juara satu, dua,
Perhatikan informasi berikut: tiga, dan empat. Kondisi pelombaan itu adalah
sebagai berikut
Suatu kubus mempunyai sisi berwarna putih,
- Randy berhasil menyalip Risky
biru, kuning, hijau, oranye, dan merah.
- Ittaqih tidak dapat menyalip Jamal
- Sisi Merah di sebelah sisi Hijau - Jamal berada di urutan kedua sebelum
- Sisi Putih berhadapan dengan sisi Kuning akhirnya dapat disalip oleh Yoga
- Sisi Biru di sebelah sisi Putih - Risky disalip oleh Ittaqih
- Sisi Biru di atas
- Sisi Oranye di sebelah sisi Merah
103. Berada di posisi manakah Yoga?
99. Sisi apakah yang berada di bawah? A. Pertama
A. Biru B. Kedua
B. Merah C. Ketiga
C. Oranye D. Keempat
D. Hijau

9
Paket 5
104. Siapakah juara 1 dalam lomba B. 67
tersebut? C. 68
A. Yoga D. 69
B. Jamal
C. Randy
D. Risky Pilihlah 1 diantara 4 gambar yang disediakan
105. Siapakah yang tidak mendapatkan sesuai dengan perintahnya.
piala?
A. Ittaqih
B. Jamal 114.
C. Risky
D. Randy
106. V, Y, M, R, Y, Q, N, Y, U, …
A. Y
B. J
C. O
D. K
107. 36, 6, 30, 12, 60, 18, 54, …, …
115.
A. 24, 108
B. 36, 120
C. 48, 360
D. 60, 90
108. A, A, B, C, E, H, M, …
A. T
B. M
C. R
D. U

109. 1, 1/4 , 16, 1/64 , … 116. Bayangan dari gambar paling kiri
A. 512 adalah …
B. 128
C. 256
D. 1024
110. 27, 18, 36, 0, …
A. 9
B. 45
C. 72
D. 0
111. I, J, K, N, M, L, O, P, Q, T, S, R, … , …
A. U, V
B. Q, P
C. W, V 117. Manakah yang berbeda?
D. V, W
112. 0, 6, 6, 20, 20, …
A. 34
B. 44
C. 76
D. 66
113. 15, 23, 31, 40, 49, 59, …
A. 66

10
Paket 5

118. Manakah gambar selanjutnya?

120. Bayangan dari gambar paling kiri


adalah …

119. Manakah gambar selanjutnya?

11
Paket 5
BAGIAN KEDUA
BAHASA INGGRIS
[ Nomor 121 s.d. 180]

Untuk bagian ini, jawaban benar kurang dari 1/3 dari jumlah soal (kurang dari 20) soal berarti nilai mati.

I. STRUCTURE b. must kicks the ball


c. may kick them ball
121. The Eiffel Tower ___________ Paris,
d. must kick them balls
France.
128. The observation deck at the Sears Tower
a. landmarks
_________ in Chicago.
b. is landmarked in
a. is highest than any other one
c. is a landmark in
b. is highest than any other one
d. is in a landmark
c. is higher than any other one
122. Young deer _________.
d. is higher that any other one
a. are called fawns
129. 129. If it _________ so cloudy, we would
b. be fawns
plan on having the fair outside.
c. is fawns
a. was
d. are fawns called
b. was not
123. Not until a dog is several months old does
c. weren’t
it begin to exhibit signs of
d. had not
independence ___________.
130. At the 1984 Democratic National
a. its mother from
Convention in San Francisco, Geraldine
b. from mother
Ferraronbecame the first woman
c. to mother
_________ for the vice presidency.
d. from its mother
a. to being nominated
124. The flexibility of film allows the artist
b. to has been nominated
__________ unbridled imagination to the
c. to have been nominated
animation of cartoon characters.
d. to will be nominated
a. to bring
131. The knee is the joint __________the thigh
b. bringing
bone meets the large bone of the lower
c. is brought
leg.
d. brings
a. when
125. In 1964__________of Henry Ossawa
b. where
Tanner’s paintings was shown at the
c. why
Smithsonian Institution.
d. which
a. was a major collection
b. that a major collection
132. Sometimes __________to place physics
c. a collection was major
and chemistry into separate categories.
d. a major collection
a. difficult
b. is difficult
126. Great numbers of tiny shelled
c. it is difficult
animal__________on the ocean floor.
d. that it is difficult
a. Live
133. _______ chocolate will give you a tummy
b. Living
ache.
c. They will live
a. Eat too much
d. If they lived
b. Eating to much
c. Eating too much
127. To score a goal in soccer you ________.
d. Eating too many
a. must kick the ball

12
Paket 5
134. If she ____________ to advance her clock 141. ____________ a tree can be grown from
one hour, she wouldn’t have been late for a seedling.
work. a. That is generally believed
a. should have remembered b. Believed generally is
b. could remembered c. Generally believed it is
c. remembered d. It is generally believed that
d. would have remembered 142. __________ratchet is a wheel or bar that
135. All gases and most liquids and solids can move in only one direction.
expand --- heated. a. A
a. in b. It is a
b. how c. Although a
c. when d. There is a
d. about 143. Most substances contract when they
freeze so that the density of a substance’s
136. 测测点:省略句 solid is _________of its liquid.
136. A dream about falling _________. a. than the higher density
a. scary is b. higher than the density
b. is scary c. the density is higher than that
c. are scary d. the higher the density
d. very scary is
144. that is much sweeter _________.
137. George Washington _________first U.S. a. than cane sugar does
President. b. does cane sugar
a. was the c. cane sugar
b. became d. than cane sugar
c. were the
d. are the 145. The poverty level in the United State is
138. Amelia Earhart was _______________ to currently set __________________.
pilot her plane across the Atlantic a. at 12,000 dollars or less.
Ocean. b. as 12,000 dollars or less.
a. the first and a woman c. at 12,000 dollars as less
b. the first woman d. at 12,000 dollar or less.
c. who the first woman
d. the woman who first 146. _____ Henry Ford first sought financial
backing for making cars, the very notion of
139. 139. Long before children are able to farmers and clerks owning automobiles
speak or understand a language, was considered ridiculous.
__________communicate through facial a. How
expressions and by making noises. b. Even
a. however c. When
b. they d. Despite
c. furthermore
d. who 147. The first president of Cornell University,
Andrew White _____ the concept of a
140. The University of Georgia, __________ in university unaffiliated with any religious
1785, was the first state supported sect or political party.
university in the United States. a. develop
a. chartered b. developing
b. was chartered c. develops
c. it was chartered d. developed
d. to be chartered

13
Paket 5
148. In order for information to be easily writing: the conflict d.between
communicated, _____ must be organized commitment to a social ideal and
in an understandable way. commitment to art.
a. there
b. and
c. it 154. Montessori preschools a.differ than public
d. how elementary schools b.in that the activities
149. 答案:C c.focus on the child’s individual abilities
149. Because of record snowfalls in the and interests rather d.than academic ones.
mountains surrounding Utah’s Great Salt Lake,
there is more water in the lake and its salt
155. Josh Billings roamed the country as a
content is _____ it once was.
a.laborer when he was a young man, but
a. least as
b.settled down in his c.later life to become
b. much less than
a humorist and d.lecturing.
c. the least what
d. less 156.
Data a.received from two spacecraft
150. Home movies began to become popular as indicate that there is b.many evidence that
a hobby in the United States during the huge thunderstorms c.are now occurring
1920’s, _____ of low-cost film. around the d.equator of the planet Saturn.
a. the invention followed
b. the invention to follow
c. following the invention 157. Every individual cell, whether a.its exists as
d. invention the following b.an independent microorganism or is part
of a c.complex creature, has its own d.life
II. FINDING ERROR cycle.

151. Using a.their bills as needles, tailorbirds


sew b.large leaves together c.with plant 158. a.Because aluminum is nonmagnetic, it is
fiber d.to forming their nests. b.value for protecting c.electrical
equipment d.from magnetic interference.

152. a.Columns may be b.circular or polygonal 159.


in cross section, and are generally c.at Nitrogen and a.oxygen are too
least four times d.more taller than they b.important that c.most living organisms
are wide. d.cannot survive without these elements.

153.
The a.poetry of Gwendolyn Brooks 160.a. Coal and petroleum resulted b.when
b.demonstrates a major plants c.become buried d.in swamps and
c.characteristically of twentieth –century decayed.

III. READING
Leonardo da Vinci was born on April 15, 1452 in Vinci, Italy. He was the illegitimate son of
Ser Piero, a Florentine notary and landlord, but lived on the estate andwas treated as a legitimate
son.
In 1483, Leonardo da Vinci drew the first model of a helicopter. It did not look very much like
our modern day “copter,” but the idea of what it could do was about the same.
Leonardo was an artist and sculptor. He was very interested in motion and movement and
tried to show it in his art. In order to show movement, he found it helpful to study the way things
moved. One subject he liked to study was birds and how they flew. He spent many hours watching

14
Paket 5
the birds and examining the structure of their wings. He noticed how they cupped air with their
wings and how the feathers helped hold the air. Through these studies, Leonardo began to
understand how birds were able to fly.
Like many other men, Leonardo began to dream of the day when people would be able to
fly. He designed a machine that used all the things he had learned about flight, and thus became the
first model of a helicopter.
Poor Leonardo had only one problem, however. He had no way to give the necessary speed
to his invention. You see, motors had not yet been invented and speed 19 was an important part of
the flying process. It would be another four hundred years before the engine was invented and
another fifty years before it was put to the test in an airplane. Leonardo’s dream of a helicopter
finally came to pass in 1936.
The Italian painter, sculptor, architect, engineer, and scientist, Leonardo died on May 2,
1519, and was buried in the cloister of San Fiorentino in Amboise.

161. What is the author’s main point? a. paragraph 3


a. The invention of the helicopter. b. paragraph 4
b. Birds cup air with their wings and c. paragraph 5
use feathers to help hold the air. d. paragraph 2
c. An overview of one of Leonardo da 167. What was the main problem with
Vinci’s many skills. Leonardo’s invention?
d. Leonardo da Vinci was born in 1452 a. motors were not yet invented
and died in 1519. b. the birds lost their feathers
162. The word problem in paragraph five c. he was illegitimate
could best be replaced by the word: d. he couldn’t draw
a. dilemma 168. The word they in the third paragraph
b. mistake refers to:
c. danger a. the feathers
d. pain b. the birds
163. The word it in paragraph two refers to: c. the studies
a. Leonardo da Vinci d. the wings
b. The first model helicopter 169. In what year was the first helicopter
c. 1483 flown
d. motion and movement a. 1483
164. Which paragraph explains why b. 1452
Leonardo’s helicopter was not successful c. 1519
in his lifetime: d. 1936
a. paragraph 1 170. What two things did birds have that
b. paragraph 2 Leonardo da Vinci noticed helped them to
c. paragraph 4 fly?
d. paragraph 5 a. wings and beaks
165. The word illegitimate in paragraph one is b. feathers and talons
closest in meaning to: c. wings and feathers
a. against the law or illegal d. cups and feathers
b. not in correct usage 171. The word thus in the fourth paragraph
c. incorrectly deduced; illogical could best be replaced by:
d. born out of wedlock a. Hence
166. The following sentence would best b. After
complete which paragraph? “Since then c. Unsuitably
people have been living out Leonardo’s dream d. Inappropriately
of flying.”
Pertanyaan nomor 172 – 180 berdasarkan teks berikut:

15
Paket 5
Glass fibers are extremely strong; for their weight, they are stronger than steel. They are
made by forcing molten glass through tiny holes called spinnerets. As many as four hundred
spinnerets are placed together, and threads of glass much thinner than human hairs are drawn off at
great speed-miles of thread per minute. As they speed along, the threads are coated thinly with a
type of glue and twisted into a yarn.The glass fibers are used with plastics to make boats and car
bodies. They are also woven into heavy cloth for window draperies and into strong belts for making
tires stronger.
A special kind of glass fiber is causing a revolution in communications. A signal of light can
be made to travel along the fiber for very long distances. By changing the quality of the light, many
messages can be sent at once along one strand of glass. New office buildings are being “wired” with
glass fibers as they are built. The glass fibers will be used to connect telephones and computers in
ways that not long ago were either impossible or too expensive.
Glass wool traps air in a thick, light blanket of fibers. This blanket is then put into walls and
ceilings to keep warm air in during the winter and cool air in during the summer.
To make glass wool, molten glass is fed into a spinning drum with many holes in it. As the
glass threads stream out of the holes, they are forced downward by a blast of hot air and through a
spray of glues. The threads are then further blown about to mix them up as they fall in a thick mat
on a moving belt. The glass we see through and drink out of has many, many other uses besides the
ones described here.
172.
What was the author’s main purpose in d. Doing
writing the article? 176. What are glass fibers woven into cloth
a. To inform you how special kinds of for?
glass are made and used a. Draperies
b. To persuade you to investigate the b. Cars and boats
many uses of glass beyond those c. Glasses
mentioned in the article d. Glue
c. To inform you about the strength of 177. The word fed in the fourth paragraph
glass fibers means:
d. To inform you that glue is used to a. To give food to
hold strands of glass together b. To minister to
173. The word special in the second c. To support
paragraph is closets in meaning to: d. To supply
a. Distinct among others of a kind 178. The word they in the second sentence of
b. Additional the first paragraph refers to:
c. Common a. Human hair c. Glass fibers
d. Species b. Weight d. Yarn
179. The word it in the fourth paragraph
174. Glass fibers are made by forcing molten refers to:
glass through: a. Molten glass c. Spinning drum
a. Spinners c. Spinnerets b. Glass wool d. Holes
b. Spiderets d. Spinets
175. The word changing in the second 180. The following sentence would best
paragraph could best be replaced by the complete which paragraph? “This
word: improvement in technology is expected to
a. Altering continue.”
b. Boring a. Paragraph 1 c. Paragraph 3
c. Bringing b. Paragraph 2 d. Paragraph 4

16
PEMBAHASAN TES POTENSI AKADEMIK

SINONIM Terdapat Hubungan “Ukuran Merupakan deret aritmatik, tetapi


Jumbo” karena perhitungannya lama,
1. Residu adalah sisa (A). 24. Jawaban = maka digunakan penjumlahan
2. Apologi adalah pembelaan (D). Upah : Buruh (D). manual. ^^
3. Efektif adalah berhasil guna (A). Terdapat Hubungan “Yang 121 + 144 + 169 + 196 = 630
4. Ebi adalah Udang Kering (C). dibayarkan kepada sesuatu” 39. Jawaban = (B).
5. Sie Qua Non adalah tiada berair 25. Jawaban = Perkalian maksimum= 15 x 15 =
(B). Pergi : Izin (A). 225.
6. Efisien adalah tepat guna (B). Terdapat Hubungan “Persyaratan” 40. Jawaban = (B).
7. Preambule adalah Penutup (B). 12-5-5 = 2
8. Leukimia adalah kanker darah Pembahasan Bacaan
(A). 41. Jawaban = (A).
26. Jawaban: B
Volum (V) = Debit (Q) x Waktu (t)
ANTONIM Reshuffle = Perubahan
3750 = (800-300) x t
susunan
t = 3750/500 = 7,5 menit
9. Makar >< muslihat (A). 27. Jawaban: C
Sudah ada dalam bacaan 42. Jawaban = (A)
10. Monologis >< Bersama (B).
28. Jawaban: B Pola Hitungan :
11. Sporadis >< Kerap (A).
Ada dalam bacaan 3.7 = 21 , crop digit terakhir [1]
12. Koordinatif >< Amburadul (B).
29. Jawaban: D 4.8 = 32, crop digit terakhir [2]
13. Sekuler >< Keagamaan (A).
Sudah jelas Yang di crop 6, jadi angka hasil
14. Regresif >< Maju (A).
30. Jawaban: A perkalian adalah 36, N = 36 / 9 = 4
15. Higienis >< Sakit (A).
T : Mengapa tidak ada 43. Jawaban = (C)
2
ANALOGI reshuffle kabinet dalam 100 -5 < p < 8 25 < p < 64
2
hari pemerintahan Presiden SBY? -1 < q < 5 1 < q < 25
2 2
16. Jawaban = J : Karena ini baru 100 hari Maka, 26 < p + q < 89
Cat : Tembok (A). 31. Jawaban: C 44. Jawaban = (A).
Terdapat hubungan “Tempat Eksoplanet = Planet yang Misal harga awal laptop = 100
Pemakaian” mengorbit bintang selain Harga laptop setelah dinaikkan =
17. Jawaban = matahari diluar sistem tata surya 125 % x 100 = 125.
Bogor : Hujan (C). 32. Jawaban: B Penurunan harga laptop agar
Terdapat hubungan “Julukan Terdapat dalam bacaan kembali ke harga awal adalah 125-
Kota” 33. Jawaban: A 100 = 25. Persentase penurunan
Tiga tahun dari 2009 = 2012
18. Jawaban = harga 25/125 x 100 % = 20 %
34. Jawaban: D
Belajar : Juara Kelas (A). 45. Jawaban = (C).
Cukup jelas
Terdapat hubungan “Yang Harus Lama perjalanan tanpa istirahat
35. Jawaban:
dilakukan untuk menjadi...” Teleskop Keppler milik NASA adalah 4 jam. Jarak (S) =
19. Jawaban = menemukan Kecepatan (V) x Waktu (t) , S = 35
Lagu : Pita (C). S km/h x 4 h = 140 km
Terdapat Hubungan “Tempat P 46. Jawaban = (C)
Penyimpanan” lima eksoplanet baru. Faktor positif yang kurang dari
20. Jawaban = O bilangan itu sendiri :
Ospek : Mahasiswa (A). 1 -> -
Terdapat Hubungan “Yang Harus KUANTITATIF 4 -> 1,2
dilakukan untuk menjadi...” 6 -> 1,2,3
21. Jawaban = 36. Jawaban = (A). 8 -> 1,2,4
Siput : Lambat (C). KPK dari 12,16, dan 24 adalah 48. 47. Jawaban = (A).
Terdapat Hubungan “Kecepatan” Maka, ketiga lampu akan Selamat Mencoba ^_^ !
22. Jawaban = menyala bersama-sama lagi pada 48. Jawaban = (C).
Makan : Tersedak (B). pukul 08.15 + 48 menit = 09.03 x = (p=2)/8, karena x adalah
Terdapat Hubungan “Yang terjadi 37. Jawaban = (D). bilangan bulat, maka (p+2) harus
apabila terlalu berlebih” 35 % . X = 42, X = 42 / 0,35 = 120 habis dibagi 8 Oleh karena
23. Jawaban = 5/6 . Y =25, Y = 30. 16<p<35, maka yang memenuhi
Atlet : Sumo (C) X + Y = 120 + 30 = 150 adalah 24 dan 32
38. Jawaban = (D). 49. Jawaban = (D).

1
Merupakan kelipatan 7 ,yaitu 14 = 10 Jadi, pekerjaan selesai 4 hari
dan 28. Karen Tito membeli September 3 hari lebih cepat dari semula
kemarin, berarti dalam 14 hari lagi Keempat kalinya = 10 September +69.
12 Jawaban: D
Tito membeli 3 tabloid lagi hari = 22 September Kelas yg mengandung median =

30
sedangkan Adi membeli 2 tabloid 61. Jawaban: D 30 : 2 = 15 > 71 - 80
− 
 =  +   2 
lagi. Jarak pengendara dari posisi awal
50. Jawaban = (C)
=[ 
15 − 13
2
8 [16] 4 = 8 /16 = 4
= 70,5 + 10  
( 600 − 300 ) 2 + ( 800 − 400 ) 2
10
2
6 [3] 12 = 6 /3 = 12
1/2

= 70,5 + 2
2
P [27] 3 = P /27 = 3, P = 9 ]

= 72,5
51. Jawaban = (B) 2 2 1/2
= (300 + 400 )
0,75/0,748 = 1
70. Jawaban: C
Harga beli = × 472500 =
52. Jawaban = (B) 1/2 

= 250000 = 500 m
350000
5(x+1) = 4(x-1)
X= 9 62. Jawaban: C
53. Jawaban = (D) Asosiatif, 61/100 x 37,2 =
Per menitnya: A = 1/30, B = 1/45, 37,2/100 x 61 71. Jawaban: B
2
p*q = p-q
C = 1/90. A+B+C = 6/90 bagian / 63. Jawaban: D
Bisa x < y, bisa juga x > y 2*(3*(5*1)) = 2 – (3 – (5 –
menit. Waktu yang dibutuhkan 2 2 2
64. Jawaban: C (1) ) )
1/(6/90) = 15 menit. 2 2 2
z=a +b –c = 2 – (3 – (5 –
54. Jawaban = (A). 2 2 2
y = b + (a + b) x (a – b) + (b – a) x 1) )
6,5/6 x 100 % = 108,33% 2 2
(b + a) = 2 – (3 – (4) )
55. Jawaban = (D). 2
+ (a – c) x (a + c) = 2 – (-13)
Tidak ada atau hubungan tidak 2 2 2 2
=b +a –b +b –a +a –c
2 2 2 = 2 – 169 = - 167
dapat ditentukan.
67 × 28 + 
2 2 2 72. Jawaban: D
=a +b –c
= 68
56. Jawaban = (C).
29
Bilangan prima yang mendekati 53 65. Jawaban: C
1876 +  = 1972
66. Jawaban: A
= 47,51,53 . Hasil penjumlahannya
adalah 151. sebelum digeser  = 96
57. Jawaban = (A). 73. Jawaban: B
6-1
U6 = 1(7/8) = 51,29 t = 6,5 2 − 2,5 2 = 36 = 6 m 4 4
8 +8 +8 +8 =4
4 4 n

4 n
58. Jawaban = (C) 1 4.8 =4
L= × 2 ,5 × 6 = 7 ,5 m 2 2 12
2 .2 =2
2n
1/3 x + 5 = ½ x . 2
14
1/6 x = 5 setelah digeser 2 =
2
2n Kelas Jumlah
X = 30
1 61 – 70 13
7,5 = × alas × 5,2 n=7
Pembahasan Kuantitatif 2 71 – 80 10
81 – 90 5
7,5
59. Jawaban: B alas = = 2,9 91 – 100 2
2,6
6! 6×5×4
hektar = hecto are = ha 74. Jawaban: D

! = =
"
pergeseran = 2,9 − 2,5 = 0,4
3! $6 − 3%! 3 × 2 × 1
2
1 dam = 1 are

= 20
Jadi,
2 2
760000 m + 3000 dam + 400 67. Jawaban: A
1 1 1
are 75. Jawaban: A
= +
27% + 5/4 – 10/32 x 64% - 27/36
& 120 80
2 2
+ 950000 m + 150000000 cm = 27/100 + 5/4 – 10/32 x 64/100 –
5
=
= 76 ha + 30 ha + 4 ha + 95 ha + 3/4
1,5 ha 360
& =72
= 27/100 + 5/4 – 20/100 – 3/4

1 1
= 206,5 ha = 7/100 + 2/4
& = × 72 = 24 '(
60. Jawaban: B
3 3
= 57/100 = 0,57
KPK dari 4 & 6 = 12
Kedua kalinya = 17 Agustus + 76. Jawaban: A
68. Jawaban: D
12 hari 1 + 2 + 3 + 5 + 7 + 11 + 13 + 17 +
8 x 7 = 14 x n
= 29 Agustus 19 = 77
56 = 14 x n
Ketiga kalinya = 29 Agustus + 77. Jawaban: B
n = 56/14 2
12 hari Vkubus : Vtabung = n x n x n : π r
n=4
t

2
3
= n : π berangkat, waktu di Jakarta adalah 96. Jawaban: D
2
(1/2n) n 07.00 – 7 = 00.00 (A). Ika > Kamis
3 Eko > Jumat
= n : π 1/4 84. Jawaban = (B). Sudah Jelas.
3 97. Jawaban: B
n 85. Jawaban = (B). Sudah Jelas.
=4:π 86. Jawaban = (C). Pembahasan : Selasa > Ciblek + Gestald
98. Jawaban: A
3) + 5* = 15375|× 2|6) + 10*
78. Jawaban: B Karena masih berstatus mahasiswa,
Jumat > Hendi + Ika
= 30750
maka geoniko belum lulus. Jadi,
Kamis > Jali + Eko
2) + 7* = 15475|× 3|6) + 21*
dia belum ditempatkan.

= 46425
87. Jawaban = (A). Sudah Jelas.
Gambar no. 7, 8 & 10
−11* = 89. Jawaban = (D). Pembahasan :
88. Jawaban = (D). Sudah Jelas.
Gambar no. 9
−15675
*
Jawaban soal ini berorientasi pada

= 1425
banyaknya jumlah lingkaran di
K K
3) + 5$1425% = 15375
suatu angka. Di angka 8, ada dua

3) = 15375 − 7125
lingkaran, di angka 10 ada satu

) = 2750
lingkaran. Jadi, 2 + 1 = 3. B O
90. Jawaban = (B). Sudah Jelas.
H P O P H B
91. Jawaban = (D).
17575 – 2750 – 1425 = 13400
92. Jawaban = (D). M M
79. Jawaban: C
Arit : n-b, n, n+b
Pembahasan Penalaran 99. Jawaban: B
Geo: n-b, n-4, n+b
Atas > Biru
r=3
Bawah > Merah
(n-b)3=n-4
100. Jawaban: C
(n-4)3=n+b Se Sela Rab Ka Ju Bawah > Hijau
3n-3b=n-4 nin sa u mis ma
Atas > Oranye
3n-12=n+b t
Be Cibl Kuk Jali Hen 101. Jawaban: C
2n=3b-4 Biru di sebelah Hijau
ny ek uh di
2n=b+12 102. Jawaban: C
Dhi Gest Adi
ka ald Biru di sebelah Hijau
3b-4=b+12 Fuji 103. Jawaban: B
Urutan pelari dari yang
2b=16 - Ciblek > Selasa pertama:
- Jali > Kamis Randy, Yoga, Jamal, Ittaqih,
b= 8 ; n=10 - Hendi > Jumat Risky
deret arit: 2, 10, 18 - Kukuh + Adi tidak Senin, space 104. Jawaban: C
∑ = 30 yg kosong hanya hari Senin & Juara satu = Randy
Rabu, jadi, Kukuh + Adi >Rabu 105. Jawaban: C
80. Jawaban: B
- Space kosong hanya hari Tidak dapat piala (juara lima) =
a=3
Senin, jadi, Beny + Dhika + Fuji
b=6 Risky
> Senin
U27 = a + (n-1)b - Yang tersisa Eko, Gestald, & 106. Jawaban: B.
= 3 + (27-1)6 Ika V Y M R Y Q N Y U JY Y
= 3 + 156 - Ika tidak Selasa V > R = -4
= 159 - Eko tidak Senin & Selasa Y=Y
- Gestald > Selasa M > Q = +4
PENALARAN - Eko & Ika dependen 107. Jawaban: A
(Kamis/Jumat) 36 _ 30 _ 60 _ 54 ( pola: 36-
81. Jawaban = (A). Sudah Jelas. Jika Eko Kamis, maka Ika Jumat vice
6=30; 30x2=60;
82. Jawaban = (C). Sudah Jelas. versa
60-6=54; 54x2= [108] )
83. Jawaban = (A). Pembahasan : 6 _ 12 _ 18 ( pola: 6+6=12;
93. Jawaban: B
(i) NY = Jakarta + 7 jam Rabu > Kukuh + Adi 12+6=18; 18+6= [24])
(ii) Perjalanan 12 jam 94. Jawaban: D 108. Jawaban: D
(iii) Ketika sampai di NY, pukul 19.00 . Beny + Dhika + Fuji > Senin Fibonacci
Berarti, saat berangkat, waktu di 95. Jawaban: A 109. Jawaban: C
1 1
NY pukul 07.00 . Jadi, saat Adi + Kukuh > Rabu 1, /4 , 16, /64 , …
0 -1 2 -3 n
Pola: 4 , 4 , 4 , 4 , 4

3
4
n=4, jadi 4 = 256 0 , 6 , 6 , 20 , 20 , n 115. Jawaban: A
+6_+6_+14 +14 +22 Perhatikan arahnya
+0 +8 +0 +8 116. Jawaban: A
110. Jawaban: C 113. Jawaban: D Dibalik
27 , 18 , 36 , 0 , n 15 , 23 , 31 , 40 , 49 , 59 , n 117. Jawaban: D
-9 +18 -36 +72 +8 +8 +9 +9 +10 +10 Sudah jelas
111. Jawaban: A 114. Jawaban: 118. Jawaban: A
IJK NML OPQ TSR UV Cukup jelas
W 119. Jawaban: D
Sudah jelas
Urutan huruf, yg digarisbawahi
120. Jawaban: A
dibalik
Pembahasan Gambar Cukup jelas
112. Jawaban: B

KUNCI BAHASA INGGRIS 141.D 161.A

121.C 142.A 162.A

122.A 143.B 163.B

123.D 144.D 164.D

124.A 145.A 165.D

125.D 166.C

126.A 146.A 167.A

127.A 147.D 168.B

128.C 148.C 169.D

129.B 149.B 170.C

130.C 150.C 171.A

131.B 151.D

132.C 152.D 172.A

133.C 153.C 173.A

134.D 154.A 174.C

135.C 155.D 175.A

136.B 156.B 176.A

137.C 157.A 177.D

138.B 158.B 178.C

139.B 159. 179.C

140.A 160.C 180.B

4
r r r t
r r r r r r ..

lfjf
+
I

t + + .. ..

STAN RS�i(l]i)�

I- .. I- .. +
I- .. + I- ..
� .. + t ..
t .. + + + .. .. + +
I- t
+ + + .. ..
I

SOAL Tr'� (])lit 6


I

t .. .... + ""
� + I- ..
+ t ... .. +

FREEMIUM GROUP FACEBOOK


SOAL SIMULASI
UJIAN SARINGAN MASUK
POLITEKNIK KEUANGAN NEGARA STAN
SOAL SIMULASI
UJIAN SARINGAN MASUK
POLITEKNIK KEUANGAN NEGARA STAN
SOAL SIMULASI
UJIAN SARINGAN MASUK
POLITEKNIK KEUANGAN NEGARA STAN

Untuk soal nomor 1 – 120, pilihlah satu di antara alternatif jawaban yang disediakan, yang Anda anggap merupakan
jawaban yang paling tepat dengan memperhatikan setiap petunjuk yang ada. Jika dibutuhkan Anda dapat menggunakan
bagian yang kosong pada naskah soal ini sebagai buram.

BAGIAN PERTAMA
TES POTENSI AKADEMIK
(NOMOR 1 s.d. 120

Untuk soal nomor 1 - 6, pilihlah kata atau frasa yang bertanda A, B, C, atau D yang mempunyai arti sama atau arti paling
dekat dengan kata yang dicetak dengan huruf kapital (huruf besar) yang terdapat di atas pilihan-pilihan tersebut.

1. ALMANAK 2. KURTASE
A. kalender A. upah makelar
B. hari B. peredaran tahun
C. penanggalan C. peleburan dua masalah
D. bulan D. penolakan sepihak
E. ahad E. kertas putih

3. PAKTA 4. INDOLEN
A. keterikatan A. cepat
B. kenyataan B. makanan
C. perjanjian internasional C. suaka
D. perkumpulan negara-negara D. lamban
E. sepuluh E. minuman

5. PADUSI 6. ITERASI
A. surga A. penambahan
B. perempuan B. satuan
C. pasangan C. perulangan
D. pria D. penampakan
E. jantan E. atraktif

Untuk soal nomor 7 - 11, pilihlah kata-kata atau frasa yang bertanda A, B, C, atau D yang merupakan padanan kata atau
padanan pengertian yang paling dekat dengan kata yang dicetak dengan huruf kapital yang terdapat di atas pilihan-pilihan
tersebut.

7. KONSENSUS 8. MILITAN
A. mufakat A. berani
B. keputusan B. gagah
C. penghitungan C. militeristis
D. menimbang D. agresif
E. kependudukan E. pasukan

9. PEDAGOGI 10. RONA


A. pengajaran A. cerah
B. ilmu kesehatan B. warna
C. sillabus C. terang
D. dokter gigi D. kilau
E. arkeologi E. guru

11. TUKAK
A. sakit
B. parah
C. borok
D. tuli
E. buta

Untuk soal nomor 12 - 16, pilihlah kata-kata atau frasa yang bertanda A, B, C, atau D yang merupakan lawan kata atau
lawan pengertian yang paling dekat dengan kata yang dicetak dengan huruf capital.

12. SURAI 13. MARGINAL


A. berhimpun A. sangat cepat
B. bertemu B. cepat berubah
C. akhir C. lambat berkembang
D. mengalah D. sangat menguntungkan
E. sungai E. perlahan

14. OFENSIF 15. PREAMBUL


A. menyerang A. dialog
B. menarik B. prolog
C. bertahan C. epilog
D. mengikat D. monolog
E. mendukung E. pasal

[Type here]
16. DISPARITAS
A. persamaan
B. penyatuan
C. persatuan
D. perkumpulan
E. seksualitas

Untuk soal nomor 17 - 24, pilihlah padanan yang bertanda A, B, C, atau D yang merupakan padanan hubungan atau
hubungan yang paling serupa dengan hubungan kata-kata yang dicetak dengan huruf kapital yang terdapat di atas pilihan-
pilihan tersebut.

17. ALIS : MATA 18. GEMETAR : … = … : LUCU


A. satu : sua A. lapar – geli
B. kepala : rambut B. sakit – lelucon
C. hidung : bibir C. gugup – parodi
D. gigi : mulut D. takut – tertawa
E. mulut : gigi E. geli : takut

19. SUKA : DUKA 20. … :LUKISAN = PENULIS : …


A. gembira : ria A. abstrak – kertas
B. tangis : tawa B. seni – penerbit
C. bangga : sombong C. pelukis – buku
D. hijau : daun D. realis – tulisan
E. klorofil : lukisan E. pelukis : penerbit

21. SURUH : KERJA 22. MATI: … = PERSALINAN : …


A. panggil : datang A. kehidupan – kehamilan
B. rayu :bujuk B. sakit – janin
C. nyanyian : merdu C. penyakit – bayi
D. minum : air D. nyawa – dokter
E. merdu : panggil E. janin : ibu

23. … : KERIPUT = KEROPOS : … 24. JALAN RAYA: … = … : GULING


A. tua – rusak A. polisi lalu lintas – selimut
B. layu – celah B. rambu lalu lintas – bantal
C. penuaan – lubang C. keamanan lalu lintas – seprai
D. pikun – hancur D. lampu lalu lintas – kasur
E. tua : lubang E. gang : pohon

Untuk soal nomor 25 – 35, bacalah tiap kutipan dengan seksama, kemudian jawablah pertanyaan yang berkaitan
dengan kutipan tersebut dengan memilih jawaban A, B, C, D, atau E.

Bacaan 1 berikut ini untuk soal nomor 25 -27


Asam lemak yang ditemukan dalam suplemen minyak ikan diketahui dapat mengganggu kerja kemoterapi dalam
melawan tumor.Oleh karena itu, pasien kanker disarankan untuk berkonsultasi dengan dokter mengenai penggunaan
suplemen tersebut. Suplemen minyak ikan diketahui kaya akan kandungan asam lemak omega-3 dan omega-6 serta yang
sangat baik akan kesehatan jantung. Namun para ilmuwan dari Belanda mengatakan, suplemen minyak ikan berpotensi
mengurangi efektivitas kemoterapi.
Dalam riset yang dipimpin Voest, salah satu tipe kemoterapi yang disebut cisplastin dan kerap digunakan untuk
mengatasi kanker paru, ovarian, testis dan kandung kemih, tidak bisa bekerja akibat pengaruh platinum-induced fatty acids
(PIFA). Asam lemak ini dihasilkan oleh sel punca dalam darah dan juga ditemukan dalam suplemen minyak ikan.
Penelitian lain pada mencit yang menderita tumor di bawah kulitnya menunjukkan hewan yang disuntikkan asam
lemak dan disebut ‘dalam kadar alamiah’ menjadi tidak sensitif terhadap kemoterapi. Sel-sel menghasilkan dua jenis asam
lemak PIFA yang disebut KHT dan 16:4(n-3).Asam lemak ini memicu reaksi kimia yang kemudian membuat sel-sel kanker
menjadi resisten terhadap kemoterapi.
Resistensi pada kemoterapi ini adalah hal yang serius.Kini kita sudah melihat bahwa tubuh sendiri menghasilkan
substansi protektif dalam darah yang kuat untuk menghadang efek kemoterapi.Substansi tersebut bisa ditemukan dalam
minyak ikan. Ahli onkologi dari The Netherlands University Medical Center Utrecht dalam jurnal Cancer Cell memublikasikan
bahwa mereka tidak merekomendasikan konsumsi suplemen minyak ikan pada pasien yang akan menjalani kemoterapi, sikap
ini diambil sambil menunggu riset lebih lanjut.

25. Berikut adalah hal yang tidak dijelaskan dalam bacaan di atas…
A. jenis-jenis asam lemak
B. tujuan dilakukannya kemoterapi
C. kadar asam lemak yang membahayakan kesehatan
D. dampak konsumsi asam lemak pada tubuh manusia
E. asam lemak dapat melawan tumor

26. Yang dimaksud resistensi pada paragraf 4 bacaan di atas adalah …


A. penolakan sel kanker terhadap asam lemak kht
B. meningkatnya kekebalan sel kanker terhadap kemoterapi
C. meningkatnya kekebalan tubuh manusia terhadap kanker
D. penolakan dalam tubuh manusia terhadap senyawa dalam minyak ikan
E. penolakan terhadap bahaya
27. Pernyataan yang sesuai dengan bacaan di atas adalah …
A. cisplatin dapat mengatasi semua jenis kanker
B. asam lemak yang tinggi pada manusia dapat mengurangi kanker
C. tubuh manusia memiliki zat yang dapat menghadang efek kemoterapi
D. tumor yang diderita mencit disebabkan oleh kadar asam lemak yang tinggi
E. asam lemak merupakan asam kuat

Bacaan 2 berikut ini untuk soal nomor 28 -31


Pernahkah Anda bertanya mengapa susunan huruf dalam keyboard mesin ketik, komputer, hingga ponsel berupa
QWERTY?Mengapa tidak dibuat berurutan seperti ABCDEFGHIJ? QWERTY sebenarnya punya banyak kelemahan seperti
membuat tangan kiri Anda overload terutama ketika menulis dalam bahasa Inggris (hal serupa dirasakan ketika menulis dalam
bahasa Indonesia). QWERTY juga membuat kelingking Anda overload.
Penelitian menunjukkan bahwa distribusi huruf tidak merata sehingga jari Anda harus menyeberang dari baris ke
baris. Bila dihitung jari tukang ketik tipikal akan berjalan lebih dari 20 mil per hari dibandingkan dengan DVORAK yang hanya1
mil. Sayangnya, orang telanjur ogah berpaling dari desain QWERTY kendati desain tersebut bukan merupakan desain yang
terbaik. Sekalipun teknologi sudah bisa mengatasi problem tombol yang mengalami jammed, orang tetap bertahan dengan
desain QWERTY. Bukannya beralih ke desain lain yang lebih superior. Bahkan QWERTY dinobatkan menjadi standar
internasional di tahun 1966.
Hal yang sama juga terjadi di Microsoft Windows. Kita tentu tahu bahwa Windows bukanlah sistem operasi terbaik,
entah itu dari segi keamanan, kemudahan, kinerja, sampai soal keindahan.Namun, karena penetrasi pasar Windows sudah
begitu deras, orang mulai terbiasa menggunakan Windows dan sistem operasi tersebut menjadi terstandardisasi.
Apakah tidak ada yang lebih baik dari Windows?Tentu saja ada.Namun orang perlu berpikir beberapa kali sebelum
berpaling dari standar tersebut. Mereka harus menghadapi barrier seperti faktor biaya, isu kompatibilitas, proses
pembelajaran, faktor waktu, dan masih banyak lagi. Akibatnya jumlah mereka yang setia jauh lebih besar daripada yang
murtad.Inilah yang menjadikan Windows atau QWERTY kemudian menjadi standar, kendati mereka bukanlah yang terbaik.
Dalam dunia ilmiah, fenomena in dijelaskan sebagai konsep path dependency dan network externality. Intinya,
inovasi tidak menghasilkan outcome yang out of the blue, tetapi merupakan perkembangan yang bisa diprediksi dari yang
sudah-sudah. Selain itu, value dari inovasi tersebut akan makin tinggi bila digunakan oleh makin banyak orang. Pada tahap
tertentu, inovasi tersebut akan menjadi standar yang digunakan oleh umum.

28. Banyak orang menggunakan keyboard QWERTY karena ….


A. sudah terbiasa
B. desainnya lebih baik
C. harganya lebih murah
D. lebih mudah digunakan
E. perkembangan zaman

29. Mengapa QWERTY dijadikan standar internasional untuk keyboard?


A. QWERTY memiliki desain terbaik.
B. Jumlah penggunanya banyak.
C. Bisa mengatasi tombol jammed.
D. Faktor biaya produksi.
E. karena digunakan di Negara maju

30. Pernyataan yang sesuai dengan bacaan di atas adalah ….


A. keyboard DVORAK rentan mengalami jammed
B. keyboard QWERTY dibuat sesuai dengan standar internasional yang berlaku
C. sebuah inovasi lebih bernilai apabila makin banyak orang yang menggunakannya
D. dengan menggunakan keyboard QWERTY dapat meningkatkan kemampuan mengetik
E. keyboard QWERTY sama efisiennya dengan keyboard biasa.

31. Fakta yang terdapat pada bacaan di atas adalah ….


A. distribusi huruf pada keyboard DVORAK lebih merata
B. keyboard QWERTY dan Windows merupakan produk terbaik
C. jari kelingking lebih sering digunakan pada keyboard QWERTY
D. jumlah pengguna keyboard QWERTY lebih banyak daripada pengguna Windows
E. keyboard QWERTY sama efisiennya dengan keyboard biasa.

Bacaan 3berikut ini untuk soal nomor 32 -35


Banyaknya pengaduan masyarakat tentang penggunaan permen sebagai alat pengembalian transaksi
perdagangan ritel membuat pemerintah gerah.Departemen Perdagangan (Depdag) minta para pedagang eceran tak lagi
menjadikan permen sebagai ganti uang kembalian kepada konsumen.Direktur Perlindungan Konsumen Depdag Radu Malam
Sembiring menegaskan, aturan pengembalian dalam transaksi ritel tertuang jelas dalam UU Nomor 23/1999 tentang Bank
Indonesia.
UU BI menetapkan, seberapa pun kecil nilai kembalian dalam setiap transaksi, tetap harus menggunakan alat
pembayaran yang sah.”Kami masih memberikan waktu bagi peritel untuk membenahi.Setelah ini kami akan mengambil
tindakan tegas,” kata Radu, kemarin.Jika peritel tetap membandel, Radu menilai, mereka telah melanggar UU BI dan UU No
8/1999 tentang Perlindungan Konsumen sehingga perlu terkena tindakan tegas.
Namun, sebelum mengambil tindakan tegas yang tak dia sebutkan dalam bentuk apa, Radu mengatakan, masalah
transaksi ini lebih dulu diselesaikan langsung dengan instansi terkait, yaitu BI. Sebab, dalih pengusaha, mereka terpaksa
memberikan permen karena tak mudah lagi bagi mereka mendapatkan uang receh untuk kembalian transaksi.”Kami kesulitan
untuk mendapatkan uang receh,” kata HeriSumantri, Ketua Bidang Kemitraan dan Departemen UKM Asosiasi Pengusaha
Ritel Indonesia.
Namun, BI tak bisa begitu saja menerima dalih dari para pedagang tersebut.”Perusahaan ritel bisa menukar ke BI
dan kami sudah menyediakan,” kata Ery Setiawan, Kepala Bagian Pengelolaan Uang Keluar BI. (Disadur dari kompas.com)

[Type here]
32. Ide pokok bacaan di atas adalah …
A. Susahya mencari uang recehan sebagai alat pengembalian.
B. Adanya aturan yang mengatur pengembalian dalam transaksi ritel.
C. Pengaduan masyarakat tentang penggunaan permen sebagai alat pengembalian transaksi ritel.
D. Hak-hak konsumen harus dilindungi.
E. hak produsen harus dilindungi

33. Tindakan tegas terhadap pelanggaran UU No 8/1999 tentang Perlindungan Konsumen dilakukan oleh …
A. Departemen Perdagangan
B. Bank Indonesia
C. Konsumen
D. Pengusaha ritel
E. pedagang kaki lima

34. Yang tidak termasuk perusahaan ritel adalah …


A. Alfamart
B. Pedagang eceran
C. Toserba Yogya
D. indomaret
E. Tidak ada jawaban yang benar

35. Peran Bank Indonesia sebagai bank sentral terdapat pada paragraf …
A. 1
B. 2
C. 3
D. 4
E. tidak ada jawaban yang benar

Untuk soal nomor 36-80, pilihlah satu di antara alternatif jawaban yang disediakan yang Anda anggap merupakan jawaban
yang paling tepat.Jika dibutuhkan Anda dapat menggunakan bagian yang kosong pada buku soal ini sebagai buram.
−3 3
2 −4 √(0,008)5
𝑥 3 ∙𝑦 3
4 37.
(0,2)4
=⋯
36. ( 2 ) =⋯
𝑦 3 ∙𝑥 2 A. 0,1
B. 0,01
A. √𝑥𝑦 2
C. 0,2
B. 𝑥 √𝑦 D. 0,02
C. √𝑥 2 𝑦 E. 0,001
D. 𝑥𝑦√𝑦
E. tidak dapat ditentukan

38. Jika barisan x1, x2, x3 . . . memenuhi x1 + x2 + x3 + …+ 8𝑝


xn = n3 untuk semua n bilangan asli. Maka nilai X100 = .
39. Jika = 4, p dan q adalah bilangan riil, maka …
2𝑞+3
...? A. p<q
A. 29.701 B. p>q
B. 27.901 C. p=q
C. 36.971 D. p + q = 100
D. 37.961 E. Hubungan p dan q tidak dapat ditentukan
E. tidak dapat ditentukan

282 −142 1 1 2 2 3
40. Jika 𝑥 = × dan 𝑦 = √42,5 ∶ 6,80 maka …. 41. Diketahui matriks A = ( ) dan B = ( ). Jika 𝑎 =
28 4 3 4 0 1
A. 𝑥<𝑦 𝑑𝑒𝑡 (A + B) dan 𝑏 = 𝑑𝑒𝑡 (A – B) maka …
2 2

B. 𝑥>𝑦 A. 𝑎>𝑏
C. 𝑥=𝑦 B. 𝑎<𝑏
D. 𝑥 + 𝑦 = 10 C. 𝑎=𝑏
E. Hubungan 𝑥 dan 𝑦 tidak dapat ditentukkan D. 𝑎+𝑏 =5
E. Hubungan 𝑎 dan 𝑏 tidak dapat ditentukan

1
42. Saya adalah bilangan pecahan. Saya lebih besar dari √𝑥−𝑦 5
5 43. Jika
2√𝑥
= 8, x dan y bilangan asli, maka ….
1
dan kurang dari , pembilang saya adalah 2. Pecahan A. x>y
4
manakah saya? B. x<y
A. 2⁄3 C. x=y
B. 2⁄7 D. x + y = 10
E. hubungan antara x dan y tidak dapat ditentukan
C. 2⁄11
D. 2⁄9
E. tidak dapat ditentukan
2
44. Jika (x-2)(x-3)(x-4) > 0 , maka . . . . 45. Nilai x yang memenuhi persamaan 1000(𝑥 −3𝑥−4) =
A. x > 2 atau x > 3 2
B. x > 3 10(𝑥 −2𝑥−3) adalah . . .
C. 3 < x < 4 atau x < 2 A. x = 1 atau x= 9/2
D. 2 < x < 3 atau x > 4 B. x= -1 atau x= 9/2
E. tidak dapat ditentukan C. x= 1 atau x= -9/2
D. x= -1 atau x= -9/2
E. tidak terdefenisi
46. X0 adalah rata – rata hitung dari X1, X2, X3, . . ., X10. 47. Himpunan bilangan real X yang memenuhi
𝑥1 𝑥2 𝑥3
Jika data berubah mengikuti pola + 2, + 4, + pertidaksamaan 22𝑥 − 2𝑥+1 > 8 adalah . . .
2 2 2
A. X > 2
6, … .. dan seterusnya, maka nilai rata – rata akan
B. X > 3
menjadi . .
C. X > 4
A. X0 + 11
D. X > 6
B. X0 + 12
E. X > 8
C. ½ X0 + 11
D. ½ X0 + 12
E. ½ X0 + 13

48. Sepuluh tahun yang lalu umur A dua kali umur B, lima 49. Ada dua buah kubus yang selisih rusuknya 4cm dan
tahun kemudiannya umur A menjadi 1,5 kali umur B. selisih volumenya 784 cm3. Rusuk salah satu kubus
sekarang umur A adalah. . . tersebut. . . .
A. 20 tahun A. 10
B. 25 tahun B. 11
C. 30 tahun C. 12
D. 35 tahun D. 14
E. 40 tahun E. 16

50. Persamaan kuadrat 𝑥 2 − 3𝑥 − 3 − 𝑎 = 0 mempunyai (1+2+3+⋯+10)3


akar-akar 𝑥1 dan 𝑥2 .Jika 𝑥1 + 2 𝑥2 = 2 ,maka nilai a 51. =⋯
13 +23 +33 +⋯+103
haruslah. . . . A. 2750
A. -2 B. 3025
B. -1 C. 505
C. 1 D. 55
D. 2 E. 110
E. 0

52. Jika 93 + 93 + 93 = 3X maka nilai X adalah . . . 53. Banyaknya bilangan asli antara 1 dan 110 yang habis
A. 6 dibagi 6 dan 9 adalah . . .
B. 7 A. 2
C. 8 B. 4
D. 9 C. 6
E. 10 D. 8
E. 10

54. 67,172 – 32,832 = . . . . 55. Tiga ekor kambing dapat menghabiskan rumput di 3
A. 4343 lapangan bola dalam waktu 3 minggu. Berapa waktu
B. 3434 yang dibutuhkan 5 ekor kambing untuk menghabiskan
C. 3344 rumput di 5 lapangan bola ?
D. 3343 A. 14 hari
E. 4433 B. 21 hari
C. 28 hari
D. 35 hari
E. 42 hari

56. Gaji rata – rata 13 orang karyawan Rp 160.000, jika gaji 57. Semula harga satu porsi mie ayam dan harga segelas
seorang direktur digabungkan maka rata – rata nya jus jeruk masing – masing Rp.8.000,00. Setelah
menjadi RP 175.000. besar gaji direktur tersebut adalah kenaikan harga minyak tanah, semangkuk mie ayam
.. harganya naik 14% dan harga segelas jus jeruk naik
A. Rp 370.000 16%. Kenaikan harga semangkuk mie ayam dan
B. Rp 245.000 segelas jus jeruk adalah . . . .%
C. Rp 225.000 A. 45
D. Rp 175.000 B. 32
E. Rp 150.000 C. 30
D. 15
E. 12

58. Tiga orang mahasiswa masing – masing memiliki 59. Sebuah bola diletakkan di dalam sebuah kubus
sejumlah uang. Jika uang yang dimiliki oleh setiap dua sedemikian sehingga kulit bola menyentuh dinding
orang dari mereka dijumlahkan, besarnya adalah (dalam kubus. Jika panjang rusuk kubus adalah x, berapakah
ribuan) 241, 249, 258. Rata – rata uang ketiga luas permukaan bola tersebut ?
mahasiswa tersebut adalah . . . A. 𝜋𝑥 2
2 B. 3𝜋𝑥 2
A. 120
3
2
C. 4𝜋𝑥 2
B. 121 D. 12𝜋𝑥 2
3
2 E. tidak dapat ditentukan
C. 123
3
2
D. 124
3
E. tidak dapat ditentukan

[Type here]
60. Antara bilangan 23 dan 135 disisipkan 7 buah bilangan. 1 1 1 1
Bilangan ini bersama dua bilangan semula membentuk
61.
1
+ 1+2 + 1+2+3 + ⋯ + 1+2+3+⋯+100 =
deret hitung. Jumlah deret hitung itu adalah . . . A. 200/101
A. 956 B. 157/65
B. 889 C. 2,5
C. 871 D. 1,243
D. 711 E. 100/101
E. 667

62. Jika bangun ABCD.EFGH adalah kubus dengan 63. 4𝑥 2 + 5𝑥 − 13 = 0 memiliki akar-akar 𝑥1 𝑑𝑎𝑛 𝑥2 ,
panjang sisinya masing 4cm. Berapa kah panjang AO 1 1
tentukanlah nilai dari + !
jika diketahui titik O adalah titik potong garis EG dengan 𝑥1 𝑥2
garis FH? A. 1/2
A. 2 − √3 B. 2/7
C. 10/26
B. 2 + √3
D. 3/8
C. 4 − √6 E. 1
D. 2√6
E. 2,5

64. Manchester United adalah sebuah tim sepakbola papan 65. Seorang anak akan mengikuti test USM STAN dengan
atas yang terdiri dari 25 pemain (pemain inti dan pemain komposisi soal: 10 soal pilihan ganda TPA dan 15 soal
cadangan) dengan masing-masing posisinya: 3 penjaga pilihan ganda TBI. Berapakah peluang anak tersebut
gawang, 10 pemain bertahan, 8 pemain tengah, dan diterima di STAN, jika syarat masuk STAN adalah lolos
sisanya penyerang. Jika pelatih mereka, Louis van Gaal, nilai mati USM STAN, dengan komposisi 70% TPA dan
2
menginginkan Manchester United bermain dengan 66 % TBI?
3
posisi 4-2-4, berapakah banyak susunan lineup yang
A. 3/22
mungkin?
B. 1/2
A. 17.640
C. 3/10
B. 14.400
D. 7/16
C. 15.600
E. 1
D. 3.600
E. 10

66. Putri dan Rima ingin mengecat pagar. Putri dapat 𝟏 𝟏 𝟏 𝟏 𝟏 𝟏


67. + + ⋯+ + + ⋯+ + =
menyelesaikan pengecatan pagar oleh dirinya sendiri 𝟓𝟓 +𝟏 𝟓𝟒 +𝟏 𝟓𝟏 +𝟏 𝟓𝟎 +𝟏 𝟓−𝟒 +𝟏 𝟓−𝟓 +𝟏
dalam waktu 3 jam, sedangkan Rima dapat A. 4
menyelesaikannya dalam 4 jam. Pada pukul 12:00 siang B. 4,5
mereka mulai mengecat pagar, kemudian Mereka C. 5
bertengkar selama 10 menit dan dalam masa itu tidak D. 5,5
satupun yang melakukan pengecatan. Setelah E. 6
pertengkaran tersebut Rima pergi dan Putri
menyelesaikan pengecatan pagar sendirian. Jika Putri
menyelesaikan pengecatan pada pukul 14:25, pada
pukul berapakah pertengkaran dimulai?
A. 12.45
B. 13.00
C. 13.15
D. 13.30
E. 14.00

68. Nilai rata-rata kelas A adalah 73, sedangkan nilai rata- 69. Sebuah kantong berisi 15 bola merah, 12 bola biru, dan
rata kelas b adalah 88. Jika jumlah siswa kedua kelas 3 bola hijau. Diambil sebuah bola secara acak sebanyak
tersebut adalah 75 dan nilai rata-rata nilai kedua kelas 2 kali tanpa pengembalian. Peluang bola yang terambil
adalah 80, maka banyak siswa kelas A adalah … orang. merah pada pengambilan pertama dan hijau pada
A. 35 pengambilan kedua adalah …
B. 38 A. 1/20
C. 40 B. 3/58
D. 42 C. 1/5
E. 45 D. 3/29
E. 6/29

70. Lima orang akan naik mobil dengan kapasitas enam 71. Diketahui sekelompok data memiliki sifat-sifat sebagai
tempat duduk, yakni 2 di depan termasuk pengemudi, berikut:
dua di tengah, dan dua di belakang. Jika hanya ada dua (i) Terdiri dari 5 data bilangan bulat positif dengan
orang yang bisa mengemudi, banyak cara mengatur rataan = 7
tempat duduk mereka adalah … (ii) Median = Modus = 9
A. 120 Jika jangkauan didefinisikan sebagai selisih data
B. 200 terbesar dengan data terkecil, maka jangkauan terbesar
C. 220 yang mungkin adalah …
D. 240 A. 11
E. 280 B. 12
C. 13
D. 14
E. 15
72. Sebuah survey mengenai bagaimana warga 73. Seorang siswa telah mengikuti tes sebanyak n kali dan
memperoleh berita berhasil memperoleh data sebagai memperoleh nilai rata-rata 80. Berapakah nilai yang
berikut: 65% warga menonton TV, 40% membaca harus diperoleh siswa tersebut pada tes selanjutnya
Koran, dan25% membaca Koran dan menonton TV. supaya rata-ratanya menjadi 82?
Berapakah warga yang tidak membaca Koran dan A. 2n – 82
menonton TV? B. n + 82
A. 5% C. 2n + 82
B. 10% D. 2n + 80
C. 15% E. 2n – 80
D. 20%
E. 25%

74. Jam tangan Agung yang setiap jamnya selalu lebih 75. Jika a, b, c, dan d adalah ekspresi bilangan bulat,
cepat 3 menit dicocokkan dengan jam tangan Bayu yang masing-masing persamaan berikut ini memiliki nilai yang
setiap jamnya terlambat 4 menit, tepat pada tengah sama, kecuali:
malam. Berapa jam perbedaan antara kedua jam itu bila A. ad + bd + cd
jam yang tepat menunjukkan tepat jam 12 siang? B. 3d + a + b + c
A. 2 jam 14 menit C. (a + b + c)d
B. 1 jam 24 menit D. ad + a(b + c)
C. 1 jam 4 menit E. d(a + b) + cd
D. 44 menit
E. 0 menit

76. Ade Wijaya diminta ibunya untuk menempel lapisan 77. Huruf-huruf A, G, E, T, W, O, N masing-masing mewakili
kertas bermotif pada sebuah kotak karton yang memiliki sebuah angka antara 1 sampai dengan 9 secara unik.
dimensi panjang 16cm, lebar 6cm, dan memiliki AGE, TWO, NOT, dan TO masing-masing merupakan
ketebalan 12 cm. berapa cm 2 kah kertas bermotif yang bilangan kuadrat dari bilangan bulat, apakah hasil
dia butuhkan untuk dapat melapisi kotak tersebut pada TWO+TO+TOO ?
semua sisinya? A. NET
A. 192 B. NAG
B. 360 C. TON
C. 720 D. TEN
D. 900 E. ONE
E. 1440

78. Pak Jay Kumar mengisi bak penampungan air yang 1 2 4 3


79. Ditentukan matriks-matriks A = ( ) dan B = ( ),
memiliki kapasitas 3750 meter kubik. Berapa lama waktu 3 4 2 1
yang ia butuhkan untuk menyelesaikan pekerjaannya maka tentukan nilai dari det (𝐴 + 𝐵) !
tersebut jika dia mengisi dengan menggunakan pompa A. 1,5
air yang memiliki kapasitas 800 meter kubik per menit, B. 1
dan tanpa sepengetahuannya ternyata pada bak C. 0
penampungan air tersebut terdapat kebocoran yang D. -1
cukup besar yang dapat mengakibatkan sejumlah 300 E. -1,5
meter kubik permenit air terbuang sia-sia?
A. 3 menit, 36 detik
B. 6 menit
C. 8 menit
D. 1875 detik
E. 7 menit, 30 detik

80. x,y,z memenuhi sistem persamaan:


2x+y = 10
2y+z = 20
2z+x = 30
Tentukan nilai 2x !
A. 20√15
B. √15
2
C.
3
√15
D. 2√15
E. 5

Untuk soal nomor 81-90, masing-masing soal merupakan deret yang belum selesai. Selesaikanlah deret-deret tersebut
dengan memilih salah satu alternatif yang disediakan, yang Anda anggap paling tepat.

81. 1, 1, 3, 9, 21, 41, 71, …, 82. 3, 8, 24, 48, 120, …


A. 113 A. 160
B. 121 B. 156
C. 111 C. 168
D. 101 D. 144
E. 143 E. 143

83. 3, 5, 7, 4, 12, 24, 5, 13, … 84. 1, 1, 2, 3, 3, 5, 4, …, …


A. 20 A. 7, 5
B. 21 B. 7, 6
C. 23 C. 7, 7
D. 24 D. 5,7
E. 25 E. 5, 6

[Type here]
85. 3, 8, 9, 16, 27, 24, …, … 86. B, C, D, P, C, D, E, Q, D, E, F, R, …, …, …, …
A. 81, 32 A. F, G, H, S
B. 81, 34 B. F, G, H, T
C. 32, 81 C. E, C, G, H
D. 32, 34 D. E, F, G, S
E. 81, 128 E. E, F, G, T

87. A, B, C, D, E, H, …, … 88. A, C, F, H, K, …, …
A. G, P A. M, P
B. I, J B. M, N
C. K, L C. P, R
D. J, L D. R, S
E. P, R E. M, O

89. B, B, C, C, D, E, E, G, …, … 90. S, R, T, V, U, W, Y, X, …
A. F, K A. A
B. F, H B. Q
C. F, G C. P
D. F, I D. Z
E. F, J E. B

Untuk soal nomor 91-104 pilihlah simpulan yang Anda anggap merupakan jawaban yang paling tepat.

91. Tidak semua orang pergi ke rumah sakit karena sakit. 92. Beberapa siswa SMAN 1 Jember bergabung dalam tim
Waris pergi ke rumah sakit. sepakbola. Tim Sepakbola tidak ada yang menjadi
A. Hari ini Waris sakit. pemain tenis.
B. Hari ini Waris tidak sakit. A. Ada beberapa siswa SMA N 1 Jember yang
C. Waris adalah seorang dokter. menjadi pemain tenis.
D. Rumah sakit adalah tempat orang sakit. B. Beberapa siswa SMAN 1 Jember bukan pemain
E. Waris belum tentu sakit tenis.
C. Tidak ada siswa SMAN 1 Jember yang menjadi
pemain tenis.
D. Beberapa pemain tenis bukanlah merupakan tim
sepakbola.
E. Tidak ada kesimpulan yang tepat

93. Beberapa hewan berkaki empat. Singa hewan karnivora. 94. Peserta SBMPTN 2014 mengikuti Test Potensi
A. Hewan berkaki empat adalah karnivora Akademik (TPA). Thomas gagal lolos SBMPTN 2014.
B. Singa dan kuda adalah karnivora Simpulan yang tepat tentang Thomas adalah …
C. Singa adalah hewan pemakan daging A. Tidak mengikuti TPA dalam SBMPTN 2014
D. Kaki harimau sama dengan kaki kuda B. Telah mengikuti TPA dalam SBMPTN 2014
E. Tidak ada kesimpulan yang tepat C. Bukan peserta SBMPTN 2014 yang mengikuti TPA
D. Peserta SBMPTN 2014 yang mengikuti tes bukan
TPA
E. Tidak ada kesimpulan yang tepat

95. Semua Martha adalah Adam. Semua Adam adalah 96. Semua Power Rangers dilengkapi dengan headset
Levine. Bluetooth. Sebagian Power Rangers berwarna Emas.
A. Ada Martha yang bukan Levine A. Sebagian Power Rangers tidak berwarna emas dan
B. Ada Levine yang bukan Martha tidak dilengkapi headset Bluetooth
C. Semua Levine pasti Adam B. Sebagian Power Rangers berwarna emas dan tidak
D. Semua Adam dan Levine pasti pandai bernyanyi dilengkapi headset Bluetooth
E. Ada Adam yang bukan Levine C. Semua Power Rangers berwarna emas dan
dilengkapi dengan headset Bluetooth
D. Sebagian Power Rangers tidak berwarna emas dan
dilengkapi dengan headset Bluetooth
E. Semua Power Rangers adalah pahlawan

97. Sebagian pemain basket tinggi badannya. Tidak ada 98. Larutan asam A harus dicampur dengan larutan basa B
pemain basket yang tidak jago matematika. agar bisa dijual. Tidak ada larutan basa B yang dapat
A. Semua pemain basket yang jago matematika tinggi diperoleh dengan mudah di tambang mineral.
badannya A. Larutan asam A adalah barang yang susah didapat
B. Sebagian pemain basket yang tinggi badannya di pasaran
jago matematika B. Larutan basa B merupakan zat yang beracun
C. Pemain basket tinggi badannya C. Tidak ada larutan asam A yang bebas dijual di
D. Sebagian pemain basket yang tidak tinggi pasaran
badannya mendapat remedial pada ulangan D. Hasil campuran larutan asam A dan larutan basa B
matematika merupakan produk yang langka di pasaran
E. Semua pemain basket yang tinggi badannya E. Larutan asam A mahal harganya
mengikuti remedial ulangan matematika

99. Jika kak Martha ganteng, maka kak Martha sudah 100. Jika aku mencintaimu, aku akan menikahimu. Aku
mandi. Kak Martha sudah mandi. menikahimu.
A. Kak Martha sudah pasti ganteng A. Sudah pasti aku mencintaimu
B. Kak Martha pasti rajin mandi biar ganteng B. Aku menikahimu karena paksaan kedua orang tua
C. Hanya orang ganteng yang rajin mandi C. Belum tentu aku mencintaimu
D. Kak Martha belum tentu ganteng D. Tidak ada yang memaksa aku untuk menikahimu
E. Tidak ada kesimpulan yang tepat E. Tidak ada kesimpulan yang tepat
101. Jika persamaan pangkat dua dengan satu variabel 102. Semua monster itu menakutkan. Sebagian dari makhluk
memiliki 2 solusi, maka persamaan pangkat tiga dengan yang menakutkan itu membuat kita rajin berdoa.
satu variabel memiliki 3 solusi. Kurva persamaan A. Kita pasti rajin berdoa
pangkat dua dengan satu variabel itu tepat menyinggung B. Tidak ada dari yang berdoa saat bertemu dengan
sumbu X. monster
A. Persamaan pangkat satu dengan satu variabel C. Sebagian monster membuat kita rajin berdoa
sudah pasti hanya memiliki satu solusi D. Hanya satu monster yang tidak menakutkan
B. Tidak ada solusi untuk persamaan pangkat tiga E. Tidak ada kesimpulan yang tepat
dengan satu variabel tersebut.
C. Persamaan pangkat tiga dengan satu variabel
tersebut belum tentu memiliki 3 solusi
D. Jawaban A, B, dan C semua benar
E. Tidak ada kesimpulan yang tepat

103. 4 kali 4 sama dengan 17. 17 merupakan bilangan genap. 104. Burung gagak adalah herbivora. Semua makhluk yang
A. 4 kali 4 sama dengan 16 herbivora pasti menyukai daging.
B. Bilangan genap adalah bilangan yang habis dibagi A. Burung gagak merupakan anomali dalam rantai
oleh bilangan prima makanan
C. Hasil dari 4 kali 4 merupakan bilangan genap B. Burung gagak tidak menyukai daging
D. 17 merupakan bilangan ganjil C. Burung gagak pasti tidak bisa terbang
E. Tidak ada kesimpulan yang tepat D. Burung gagak menyukai daging
E. Tidak ada kesimpulan yang tepat

Untuk soal nomor 91-100, masing-masing terdiri dari sebuah ilustrasi dan beberapa pertanyaan. Bacalah setiap ilustrasi
dengan saksama, kemudian jawablah pertanyaan yang berkaitan dengan ilustrasi tersebut.

Untuk soal nomor 105 - 108, perhatikan ilustrasi berikut.


Seorang pelatih sebuah klub basket di STAN akan memilih sebuah tim basket dengan komposisi sebagai berikut:
 Dalam satu tim berisi 5 pemain inti dan 3 pemain cadangan.
 Harus ada minimal satu playmaker dalam satu tim.
 Harus ada 2 guard dalam satu tim
 Center adalah pemain yang memiliki tinggi badan minimal 185cm.
 Harus ada kerja sama yang baik dari 5 pemain yang masuk ke lapangan nantinya
 Total pemain di klub basket STAN ada 10 orang yaitu: Martha, Rizky, Ade, Wijaya, Aditya, Suza, Anggi, Darma,
Adli, dan Waris.
 Martha, Ade, Darma, Adli, dan Waris adalah teman basket sejak mereka rata-rata berumur 4 tahun
 Aditya dan Suza pemain yang paling tidak disukai di klub karena sifatnya
 Semua pemain basket di klub memiliki fisik yang bagus.

105. Jika suatu waktu klub basket tersebut ikut turnamen dan 106. Jika Ade, Darma, Suza, dan Rizky bermain di posisi
ada dua orang yang tingginya di atas 185, yaitu Martha guard, Waris, Adli, Anggi, dan Wijaya bermain di posisi
dan Aditya, maka, pemain manakah yang pasti tidak playmaker, maka manakah dari bawah ini yang paling
akan diikutsertakan oleh pelatih di turnamen tersebut? sering/ lama ada di bangku cadangan?
A. Aditya A. Martha
B. Suza B. Ade
C. Martha C. Waris
D. Jawaban A dan B benar D. Aditya
E. Tidak ada jawaban yang tepat E. Wijaya

107. Jika saat pertandingan quarterfinal Adli dilanda cedera 108. Jika dihitung total point yang dimasukkan oleh pemain
engkel yang parah, maka manakah dari bawah ini masing-masing di bawah ini sepanjang turnamen,
pemain yang akan dipilih oleh pelatih untuk dengan peluang yang paling banyak mencetak point
menggantikannya bermain di semifinal-final? berturut-turut: playmaker, guard, dan center, maka
A. Rizky siapakah pemain yang paling mungkin mencetak point
B. Suza paling banyak sepanjang turnamen?
C. Martha A. Martha D. Waris
D. Aditya B. Ade E. Adli
E. Anggi C. Darma

Untuk soal nomor 109 - 112, perhatikan ilustrasi berikut.


Delapan buah bola masing-masing bertuliskan angka berlainan dari a, a+1, …, a+7. Bola-bola ditempatkan secara acak ke
dalam 8 kotak yang masing-masing ditandai dengan huruf-huruf S, T, U, V, W, X, Y, dan Z. Setelah ditempatkan, diketahui
bahwa:
i. W berisi bola dengan angka bernilai 4 lebih besar daripada angka bola dalam Z dan bernilai 3 lebih kecil jika
dibandingkan dengan angka bola dalam X.
ii. Sedangkan 5 berisi bola dengan angka bernilai lebih besar daripada angka bola di dalam T, dan lebih besar daripada
angka bola di dalam X.
iii. U berisi bola dengan angka yang merupakan nilai rata-rata dari angka bola dalam V dan X.

109. Jika nilai terendah dari angka-angka tersebut adalah 8, 110. Jika V lebih kecil dibandingkan W, manakah dari
berapakah angka bola dalam W? beberapa urutan ini yang paling mungkin merupakan
A. 15 urutan dari angka-angka tersebut, dimulai dari angka
B. 13 yang terbesar di sebelah kiri?
C. 12 A. X, S, T, W, V, U, Y, Z
D. 11 B. X, T, S, V, W, U, Z, Y
E. 10 C. X, S, U, W,V, T, Y, Z
D. Z, S, T, W, U, V, Y, X
E. X, U, S, T, W, V, Y, Z

[Type here]
111. Manakah dari pernyataan berikut yang benar? 112. Jika Y lebih besar 3 tingkat dari pada Z:
iv. W tidak berisi bola angka yang terbesar dari i. W lebih besar dari U
rangkaian angka tersebut ii. S lebih besar dari W
v. Z bukanlah angka yang terbesar dari rangkaian iii. Y lebih besar dari V
angka tersebut iv. Y lebih besar dari T
vi. X bukanlah angka yang terbesar dari rangkaian Dari pernyataan-pernyataan di atas yang benar adalah:
angka tersebut A. Hanya (i) dan (ii) yang benar
A. Hanya (i) yang benar B. Hanya (i) dan (iii) yang benar
B. Hanya (ii) yang benar C. Hanya (i), (ii), dan (iv) yang benar
C. Hanya (i) dan (ii) yang benar D. Hanya (ii), (iii), dan (iv) yang benar
D. Hanya (i) dan (iii) yang benar E. Tidak ada yang benar
E. (i), (ii), dan (iii) semuanya benar

Untuk soal nomor 113 - 114, perhatikan ilustrasi berikut.


Pada suatu pertunjukan seni di kampus STAN, kelas “Crocodile” yang masing-masing dari artist di kelas tersebut: Martha,
Gandi, Adam Levine, Ade, Waris, Anggi, Jay, Darma, dan Adli, yang memiliki bakat seni akan mementaskan kemampuan
mereka masing-masing dengan aturan sebagai berikut:
 kelas tersebut menampilkan musik (band) dan pentas drama secara bersamaan. Lalu ditutup dengan penampilan
seni tari saman.
 Martha, Ade, dan Jay adalah artist-artist yang multi-talent, mampu menampilkan berbagai bakat seni.
 Adli lahir dari keluarga musisi Jazz.
 Waris adalah penari kecak Bali semasa SMA-nya.
 Darma adalah seorang seniman pantomim yang sudah terkenal semenjak dia duduk di bangku SMA.
 Anggi adalah seorang pelukis yang senantiasa memamerkan karyanya di ajang international kids choice awards.
 Jay semasa SMP-nya pernah ikut bermain di industry film Bollywood, film Slumdog Millionare.
 Adam Levine adalah seorang vokalis band terkenal, Maroon 5.
 Bermain musik dalam band adalah kegiatan yang paling popular di kampus tersebut dan merupakan UKM wajib
bagi tiap mahasiswa.
 Tidak ada dari ke-9 anak yang disebutkan di atas yang tidak ikut ambil peran dalam acara pentas seni tersebut.

113. Dari soal tersebut, ada berapa macam jumlah seni yang 114. Jika pertunjukkan band dari kelas tersebut tersusun dari
ditampilkan oleh kelas “Crocodile” di acara pentas seni 5 personil, yaitu vocal, gitar, bass, keyboard, dan drum,
STAN tersebut? manakah di bawah ini yang tidak menjadi personil band
A. 4 kelas tersebut?
B. 5 A. Adam Levine
C. 6 B. Adli
D. 7 C. Ade
E. 8 D. Jay
E. Gandi

115. Jika pertunjukkan seni drama “Mahabarata” harus


dipimpin oleh setidaknya 5 orang artist kelas tersebut,
manakah dari di bawah ini yang pasti ikut bermain di sesi
seni tersebut?
A. Adam Levine
B. Waris
C. Darma
D. Anggi
E. Gandi

Untuk soal nomor 116 – 117 tentukan salah satu dari 4 pilihan yang merupakan perputaran gambar dalam soal!

116.

117.

Untuk soal nomor 118 dan 119 setiap bentuk gambar yang pertama akan dihubungkan dengan bentuk gambar yang kedua.
dengan cara yang sama bentuk ketiga dihubungkan dengan bentuk gambar yang keempat, manakah dari keempat pilihan
yang merupakan bentuk keempat

118.
119.

Untuk soal no 120, Tentukan pilihan yang tepat

120.

[Type here]
BAGIAN KEDUA
TES BAHASA INGGRIS
(NOMOR 121 s.d. 180)

121. He has blond hair,…….? 122. I remember ….. in this city when the traffic wasn’t too
A. hasn’t he heavy.
B. doesn’t he A. driving
C. didn’t he B. drive
D. has he C. to drive
D. drove

123. Were I rich, I ….. around the world. 124. You look so awful with your performance; you should
A. Will travel go to barber to have your hair ……
B. Would travel A. cut
C. travelled B. to cut
D. would be travel C. to be cut
D. cutting

125. Ben: “What did the cashier of the bookstore tell to you?” 126. A period of eight hours …. not enough to finish this
Brian: “…………. with a credit card. assignment.
A. not paying A. are
B. not pay B. is
C. let’s not pay C. to be
D. not to pay D. being

127. ….. her mistakes, the stewardess immediately 128. You should check your answer sheet before ……
apologized to the passengers. A. hand it in
A. realized B. to hand it in
B. realizing C. handing it in
C. she realized D. handed it in
D. having been realizes

129. I ….. down the street when it began to rain. 130. Could you please tell me ………..
A. am walking A. where is the toilet?
B. would walk B. where the toilet?
C. have walked C. where the toilet is?
D. had been walking D. where did the toilet?

131. Rather than ……. the homework, I’d prefer to watch TV. 132. In order to accommodate bigger air crafts, they need to
A. do ….. the runway.
B. to do A. lengthen
C. doing B. length
D. did C. prolong
D. longer

133. Almost all of the students fail ….. on the first try. 134. Reni : “Does John know that Merry had an accident
A. in passing the TOEFL test yesterday?”
B. to pass the TOEFL test Agatha: “I don’t think so. We ….. as soon as we were
C. to have pass the TOEFL test informed.”
D. pass the TOEFL test A. should have told him
B. had to tell him
C. must have told him
D. had better tell him

135. The concept of present value …. the time value of 136. This man is a …..citizen of this country.
money. A. natural
A. Is based on B. naturalized
B. Is based of C. natutalization
C. Is based at D. natured
D. Is based in

137. If my father had had a lot of books and encyclopedias, 138. Maria has a ….. dress.
he would have realized his own library. A. new small beautiful red cotton Italian
The real fact of the sentence above is….. B. small new beautiful red cotton Italian
A. my father didn’t have a lot of books and C. beautiful small new red Italian cotton
encyclopedias so he can’t realized his own library D. beautiful new small red Italian cotton
B. my father doesn’t have a lot of books and
encyclopedias so he can’t realized his own library
C. my father doesn’t have a lot of books and
encyclopedias so he couldn’t realized his own
library
D. my father didn’t have a lot of books and
encyclopedias so he couldn’t realized his own
library
139. If endangered species …. saved, rainforest must be 140. Language, of course, is ….. the more you know of the
protected. culture of the country you are dealing with, the less
A. are to be likely you are to get into difficulties.
B. can be A. full of difficulting, disaster might the only a syllable
C. be away, however
D. will be B. full of difficulties, disasters may be the only a
syllable away, but
C. full of difficulties disaster may be only a syllable,
but
D. full of difficulties, disaster maybe only a syllable
away, but

141. Would you mind ……. , please? 142. The president director agreed that the performance of
A. to answer the telephone the company has not been good these past few years.
B. answering the telephone In fact ……
C. answer the telephone A. Its organization restructured now
D. to the telephone answering B. Its organization is to restructure now
C. Its organization is restructuring now
D. Its organization is being restructured now

143. The foundation could only provide assistance ….. 144. All transaction effect at least two accounts. The passive
A. temporal form of this sentence is….
B. temporaries At least two accounts ….. by all transaction.
C. temporary A. were affected
D. temporarly B. are affected
C. have affected
D. affected

145. The old man asked her to move because he ….. in that 146. The man who was driving the truck would not admit that
chair. he had been at fault, and ….
A. used to sits A. neither was the other driver
B. was used to sit B. neither would the other driver
C. used to sitting C. neither had the other driver
D. was used to sitting D. the other driver neither

147. You ….. your seats today if you want to go to the game. 148. Ferry had his brother ….. his shoes for him.
A. had better to reserve A. to tie
B. had to better reserve B. tie
C. had better reserve C. tied
D. had to reserve better D. tying

149. The government is aiming at …. 150. One of the mysterious things in nature is …… to a
A. full employment particular place after doing journeys.
B. full employable A. The ability of certain creatures to find their way
C. full employee home, sometimes from great distances, for
D. full employer example birds have capability to return
B. The ability of certain creatures finding their way
home, sometimes from great distances, for
example birds are able to return
C. The ability of certain creatures to find their way
home, sometimes from great distances, for
example birds are able to return
D. The ability of certain creatures to find their way
home, sometimes from great distances, for
example birds are capable to return

ERROR RECOGNITION
Choose the one word or phrase which would not be appropriate in standard written English

151. A large percentage of the employees at the Kediri Government center are participating (A) in an experimental four-day
(B) work week (C) aimed at curbing gasoline consumption and pollution, two of the most urgent problems (D)
facing cities today.

152. According to (A) recent geological research, the climate (B) of the states along the Canadian border is (C) changing with
rapidity. (D)

153. Dr. Monica, the first woman elected (A) president of the collage, was (B) intelligent, capable, and awareness (C) of the
problems to be solved. (D)

154. A contract with (A) a supplier, either a manufacturer and (B) a wholesaler, may be the basis (C) for estimating (D) unit
cost .

155. Many jobs are usually available (A), primarily because of (B) the education and training for accounting (C) career have
not (D) kept pace with the demand for accounting services.

156. A painter (A) who lived most (B) of his life (C) in the Middle West, Grant Wood has called (D) America’s “Painter of The
Soil.”

[Type here]
157. Chromosomes as appear (A) long, tangled threads (B) when they first become (C) visible during (D) the early prophase
stage of mitosis.

158. In the Middle Ages, books called (A) bestiaries were prepared in an attempt (B) to describe animals, real or imagine (C),
that (D) exemplified human traits.

159. The relationship of (A) Latin American music to (B) Black music in the United States is clearly evident (C) in the
unaccented beats that are common to either (D).

160. Seven of planets (A) rotate in the same direction like (B) their orbital motions, while (C) Venus and Uranus rotate in the
opposite direction. (D)

READING COMPREHENSION
Read the passage carefully and select the one correct answer from the four choices

Reading 1 for question 161-165


The Butterfly
A man found the cocoon of a butterfly, on day a small opening appeared. He sat and watched the butterfly for
several hours as it struggled to force its body through that little hole. Then it seemed to stop making a progress.
It appeared as if it had gotten as far as it could and it could go no further. So the man decided to help the butterfly, he
took a pair of scissors and sniped off the remaining bit of the cocoon. The butterfly then emerged easily. But it had a swollen
body and small shriveled wings.
The man continued to watched the butterfly because he expected that, at any moment , the wings would enlarge
and expand to be able to support the body, which would contract in time. Neither happened. In fact, the butterfly spent the
rest of its l ife crawling around with a swollen body and shriveled wings. It never was able to fly. What the man in his kindness
and haste did not understand, that the restricting cocoon and the struggle required for the butterfly to get through the
tiny opening were God’s way of forcing fluid from the body of the butterfly into its wings, so that it would be ready for flight
once it achieved its freedom from the cocoon.
Sometimes struggles are exactly what we need in our life. If God allowed as to go through our life without
any obstacles, it would cripple us. We would not be as strong as what we could had been. We could never fly.

I asked for wisdom, and God gave me problems to solve.


I asked for love, and God gave me troubled people to help.
I received nothing I wanted. I received everything I needed

161. How is the condition of butterfly after the man help it? 162. What is the synonym of RESTRICTRING?
A. The body is big but the wings are small A. soft
B. The body is small but the wings are big B. hard
C. The butterfly can fly easily C. big
D. The butterfly grow quickly D. small

163. What kind of the text? 164. The generic structure of the text is …..
A. recount A. abstract-orientation-crisis-reaction-coda
B. anecdote B. identification-description
C. description C. orientation-complication-resolution-reorientation
D. narration D. thesis-argument-conclusion/recommendation

165. What can we learn from the text above?


A. Someone must do something to get what he/she
wants
B. People must help the others, because he/she may
need help too
C. God has a secret of something in the world, which
must be revealed by us
D. Butterfly can fly if someone help it

Reading 2 for questions 166 - 171


Joni Mitchell
Joni Mitchell doesn’t look or act like a superstar.Yet she is one of the most beautiful singers today. She plays a
guitar and sings with a soft, gentle style. Her voice has an unbelievably wide range – shecan sing very high notes and very
low notes. And she’s able to sing all types of music, from folk to jazz.
As a young girl Canadian-born Joni Mitchell taught herself to play the guitar. Although she played quite well, Ms.
Mitchell planned a career as a painter. After she graduated from high school, she attended art classes but soon left to try her
luck as a singer. She wrote her first song “Day After Day,” while riding a train to Toronto, Canada, where she hoped to find a
job. It was a ballad, a simple song that tells a story. She made up the tune by listening to the rhythmic clacking of the train
wheels.
Ms. Mitchell get ideas for songs from her own emotions. Her expresses her feeling about life, about love and about loneliness.
for more than 10 years she has been writing and recording hit songs. Yet she feels that she’s still growing with her music. She
says “I feel as if my best work is still in front of me.”

166. When did Joni Mitchell decide to become a singer? 167. This story is mainly about Joni Mitchell …..
A. When she was a little girl A. Childhood and schooling
B. When she was in high school B. Songs and singing
C. Shortly after high school C. Style and feelings
D. At the age of 30 D. High and low notes
168. Joni Mitchell wrote her first song when she was …. 169. In Toronto, Joni Mitchell hoped to find a job as a ….
A. In school A. songwriter
B. At home B. teacher
C. Working C. guitarist
D. On a trip D. singer

170. What is ballad? 171. Emotions are ….


A. A song for the guitar A. feelings
B. A song about a train B. thoughts
C. A story put to music C. songs
D. An easy tune to play D. sayings

Reading 3 for questions 172 - 177


Globalization
We will never live in a truly global world, where geography and local culture do not matter. Globalization is a process,
not a destination. For a start, people are sedentary. Moving about will always take timeand money. Most of what we consume
cannot be traded. Since many services have to be provided on the spot, much of the economy is set to remain local:
nurses, nannies, hairdressers, gardeners, shop assistants,fitness instructors, cleaners, and therapists cannot ply their
trade on the other side of the globe. These services are the fastest-growing area of rich-country economy.

172. What is the topic of the text above? 173. According to the text, which one is not true?
A. Globalization and culture A. There is a possibility of the existence of such a
B. Globalization destination truly global world
C. Local services remain unaffected by globalization B. Moving consumes time and money
D. Process of globalization C. There are some services which can not be traded
globally
D. Services are part of rich-country economy

174. What does the word “ply” mean? 175. Which one of these jobs doesn’t belong to a therapist
A. To reject category?
B. To misuse A. doctor
C. To neglect something B. psychiatrist
D. To conduct C. counselor
D. engineer

176. what is the synonym of the word “nannies”?. 177. What is the meaning of the word “sedentary”?
A. Baby-sitters A. mobile
B. housemaid B. sediment
C. teacher C. settled
D. laundry D. consuming

Reading 4 for questions 178 – 180

Britain has, traditionally, been a great commercial centre and has a highly developed economy. Accordingly,
its financial institutions are also well-developed and have a long history. The bank of England was founded in the seventeenth
century, and the industrial revolution encouraged the establishment of new private banks and finance houses and numerous
smaller country banks.
Banks became necessary as money come to play a larger part in society. Goods were exchanged for cash;
labourers were paid cash wages; services had to be paid for in cash; new businesses needed largesums money and were
prepared to pay interest. As money (rather than goods) became means of exchange, banks became important as secure
places to holds deposits, as sources of loans, and, with the development of the cheque system, as the medium by which
goods and services could be bought and sold without cash changing hands.
it is important to be clear about defining “ money”. Money isn’t just the coins and notes in your pocket; it is also the
amount held in bank accounts. From the late eighteenth century onwards it was realized that time and effort were wasted and
the change of robbery or loss heightened if every transaction was paid for in notes and coins, so, as the banks increased in
number, a system of payment by “cheque” was developed. Banks issued cheques to their customers. When an account-
holder made a large payment he wrote the amount on the cheque, and the recipient placed it in bank account. Money
passed from bank to bank, andfrom account to account, rather than from person to person. Each individual was thereby
relieved of the burden of having to carry large sums of money and the risk of loss was greatly reduced.

178. Money is the …. Held in bank accounts 179. The word “establishment” in last sentence of first
A. sum paragraph can not be replaced by….
B. amount A. commencement
C. total B. formation
D. savings C. standfast
D. maintenance

180. Which in the following, isn’t a plus point of a cheque?


A. Services could be bought and sold without cash
changing hands
B. Goods could be bought and sold easily
C. The risk of loss can be reduced
D. Each individual can not carry large sums of money

[Type here]
PEMBAHASAN 31. C
32. C
1. C. penanggalan 33. A
2. A. upah makelar
34. D
3. C. perjanjian internasional
35. B
4. D. lamban −3
2 −4 4 −1
−1 −3 −1
5. B. perempuan 𝑥 3 ∙𝑦 3 𝑥 2 .𝑦 ( −2) (1− )
36. D. ( 2 ) =( −1 −3 )= 𝑥 2 .𝑦 2 =
𝑦 3 ∙𝑥 2 𝑦 2 .𝑥 2
6. C. perulangan
3
7. A. mufakat 𝑥. 𝑦 = 𝑥𝑦√𝑦
2
5 5
8. D. agresif 8 2 3∙
3
√(0,008)5 ( )3 ( ) 3 2 5−4 2
1000 10
9. A. pengajaran 37. C. = 2 4
= 2 4
= ( ) =( )=
(0,2)4 ( ) ( ) 10 10
10 10
10. B. Warna
0,2
11. C. borok
12. A. bercerai-berai >< berhimpun
13. D. tidak terlalu menguntungkan >< sangat
menguntungkan 38. A

14. C. menyerang (ofensif) >< bertahan


15. C. pendahuluan (preambul)>< penutup (epilog)
16. A. perbedaan (disparitas)>< persamaan
17. C. HIDUNG terletak di atas BIBIR seperti halnya
ALIS terletak di atas MATA
18. D. GEMETAR disebabkan karena TAKUT seperti
halnya TERTAWA disebabkan karena hal yang
LUCU
19. B. SUKA merupakan kondisi yang berkebalikan
dengan DUKA seperti halnya TANGIS yang
berkebalikan dengan TAWA
20. C. PELUKIS menghasilkan karya LUKISAN
sebagaimana PENULIS menghasilkan karya
8𝑝
berupa BUKU 39. B. = 4; 8p = 8q + 12; 8p-8q= 12, maka p >
2𝑞+3

21. A. panggil : kerja q


22. A. Sebelum mati ada proses KEHIDUPAN 282 −142 1 (28+14)(28−14) 1
40. B. 𝑥 = × ;x= . =
28 4 28 4
seperti halnya sebelum PERSALINAN ada 42 .14 1 21
. = = 5,25
proses KEHAMILAN 28 4 4

23. C. PENUAAN ditandai dengan KERIPUT seperti


. 𝑦 = √42,5 ∶ 6,80 ; y =√6,25 = 2,5
halnya KEROPOS ditandai dengan adanya
LUBANG 3 5
41. C. matriks A+B = ( ) ; 𝑚𝑎𝑡𝑟𝑖𝑘𝑠(𝐴 +
24. D. Di JALAN RAYA biasanya terdapat LAMPU 3 5
9 25
LALU LINTAS seperti halnya KASUR biasanya 𝐵)2=( ) ; det(A+B)2= (9x25) – (9x25) = 0
9 25
terdapat GULING
25. C. dalam bacaan tidak dijelaskan kadar −1 −1
Matriks A-B = ( ) ; 𝑚𝑎𝑡𝑟𝑖𝑘𝑠 (𝐴 −
3 3
konsumsi asam lemak yang dapat mengganggu
1 1
kerja kemoterapi. 𝐵)2=( ) ; det(A-B)2 = (1x9) – (1x9) = 0
9 9
26. B. dapat dilihat pada kalimat terakhir paragraf
ketiga. 42. D. 2/9 = 0,2222
√𝑥−𝑦 √𝑥 5 𝑥−𝑦√𝑥 5
27. C. dapat dilihat pada kalimat kedua paragraf 43. A. . = ; = ; 8𝑥 − 8𝑦√𝑥 =
2√𝑥 √𝑥 8 2𝑥 8
keempat. 2𝑥 √𝑥
10𝑥; 2𝑥 = −8𝑦√𝑥; . = −8𝑦; 2√𝑥 = −8𝑦,
28. A √𝑥 √𝑥

29. B dengan demikian x pasti lebih besar dari y

30. C
54. B. (67,17 + 32,83) (67,17 – 32,83) = (100)
(34,34) = 3434
44. D. X=2, x= 3, x=4 ; >0 55. B. Perbandingan setara 1 kambing untuk 1
lapangan, maka waktunya tidak berubah, yaitu
tetap 3 minggu = 21 hari
𝑛1𝑥1+𝑛2𝑥2
56. A. 𝑋𝑔𝑏 = ; 175.000 =
2 3 4 𝑛1+𝑛2
13(160.000)+1(𝑥)
; 2.450.000 = 2.080.000 + 𝑥; 𝑥 =
13+1
Maka himpunan penyelesaiannya; 2<x<3 atau x
370.000
>4
57. D. karena harganya sama, maka total kenaikan
14+16
45. B. 103(𝑥
2 −3𝑥−4)
= 10(𝑥
2 −2𝑥−3)
; 3𝑥 2 − 9𝑥 − 12 = = = 15%
2

𝑥 2 − 2𝑥 − 3 58. D. Misalkan, 241= A + B ; 249 = A + C ; 258 = B


2
.2𝑥 − 7𝑥 − 9 = 0; ( 2x + 2) ( x – 9/2) +C
X = -1 atau x = 9/2 241 + 249 + 258 = A+B+A+C+B+C
46. C. Jika data semua dibagi 2, maka rata-rata 748 = 2A + 2B + 2C ; 748 = 2(A + B + C)
748 1 2
juga menjadi X0/2 , lalu ditambah dengan pola: Rata-rata dari (𝐴 + 𝐵 + 𝐶) = . = 124
2 3 3
(2+4+6+⋯+20) 110
. = = 11, maka rata-rata 59. A. dengan demikian diameter bola adalah X,
10 10
𝑋0 𝟏
menjadi + 11 maka jari – jari nya adalah 𝒙
2 𝟐
1
47. A. 22𝑥 − 2𝑥+1 > 8 ; (2𝑥 )2 − 2 . 2𝑥 − 8 > 0 L.permukaan = 4𝜋𝑟 2 = 4𝜋( 𝑟)2 = 𝜋𝑟 2
2
Misalkan 2x = p, maka 𝑝2 − 2𝑝 − 8 > 0 ; (𝑝 − 𝑛
(𝑎 + 𝑈𝑛)
60. D. Jumlah deret hitung: 𝑆𝑛 =
2
4)(𝑝 + 2) > 0
ada 9 suku dlm deret itu sehingga: 𝑆9 =
P >4 atau p>-2 maka: 2𝑥 > 22 𝑎𝑡𝑎𝑢 2𝑥 > 9 9
(23 + 135); (158) = 711
−2(𝑡𝑑𝑘 𝑚𝑒𝑚𝑒𝑛𝑢ℎ𝑖), 𝑚𝑎𝑘𝑎 𝑥 > 2 2 2
1 1 1 1
48. A. Pers1: A-10 = 2(B-10) ; A – 10 = 2B – 20; A 61. + + +⋯+
1 1+2 1+2+3 1+2+3+⋯+100

– 2B = -10 = +2
1 1
+3
1 1
+ 4(1+4) + ⋯ + 100
1
1 (1+2) (1+3) (1+100)
2 2 2 2
Pers2: A – 5 = 1,5(B – 5) ; A – 1,5B = -2,5 ; 2A
2 2 2 2 2
– 3B = -5 = + + + + ⋯+
1.2 2.3 3.4 4.5 100.101
1 1 1 1 1
Eliminasi pers 1 dan pers 2 sehingga didapat A = 2( + + + + ⋯+ )
1.2 2.3 3.4 4.5 100.101
= 20 dan B = 15 1 1 1 1 1 1 1
= 2 {( − ) + ( − ) + ( − ) + ⋯ + ( −
1 2 2 3 3 4 100
49. A. Rusuk kubus I : x
1
Rusuk kubus II: y, maka x – y = 4 ; x = y + 4 )}
101
3 3 3 3
Selisih volume: 𝑥 − 𝑦 = 784 ; (𝑦 + 4) − 𝑦 = = 2( −
1 1
)
1 101
2
784; 12𝑦 + 48𝑦 + 64 = 784 200
=
.𝑦 2 + 4𝑦 − 60 = 0 ; (𝑦 + 10)(𝑦 − 6); 𝑚𝑎𝑘𝑎 𝑦 = 101

−10 𝑎𝑡𝑎𝑢 𝑦 = 6 𝑛
(hint: Rumus deret aritmatika = 𝑆𝑛 = (𝑈1 + 𝑈𝑛 )
2
Tidak mungkin rusuk negative, maka y = 6,
)
sehingga x = 10
50. C. 𝑥 2 − 3𝑥 − 3 − 𝑎 = 0
Jawaban A.
𝑏
Jumlah akar-akar: 𝑥1 + 𝑥2 = − = 3
𝑎

.𝑥1 + 2 𝑥2 = 3 ; 𝑥1 + 𝑥2 + 𝑥2 = 2 ; 3 + 𝑥2 =
2; 𝑥2 = −1
62. 𝐸𝐺 2 = 𝐸𝐹 2 + 𝐹𝐺 2 = 42 + 42 = 32 (Phytagoras)
. 𝑥 2 − 3𝑥 − 3 − 𝑎 = 0; (−1)2 − 3(−1) − 3 − 𝑎 =
𝐸𝐺 = 4√2
0 ; 1 − 𝑎 = 0; 𝑎 = 1
1
(1+2+3+⋯+10)3 (55)3 (55)3
𝐸𝑂 = 𝐸𝐺
2
51. D. = = = 55
13 +23 +33 +⋯+10 3 (1+2+3+⋯+10) 2 (55)2
𝐸𝑂 = 2√2
52. B. 3 . 93 = 3𝑥 ; 37 = 3𝑥 ; 𝑥 = 7 2
110 𝐴𝑂2 = 𝐴𝐸 2 + 𝐸𝑂2 = 42 + (2√2) = 24
53. C. KPK dari 6 dan 9 adalah 18, maka =
18
(Phytagoras)
6,11111; 𝑚𝑎𝑘𝑎 𝑎𝑑𝑎 6 𝑏𝑖𝑙𝑎𝑛𝑔𝑎𝑛
𝐴𝑂 = 2√6
Jawaban D. Sehingga kita dapat mengambil hubungan dari
𝑏
1 1 𝑥1 +𝑥2 −𝑎 𝑏 5 5 10 analisis diatas
63. + = = 𝑐 =− =− = =
𝑥1 𝑥2 𝑥1 𝑥2 𝑎
𝑐 −13 13 26 1
𝑡2 = (2 − 𝑡1 )
4
Jawaban C.
Selesainya pekerjaan kita anggap sebagai 𝑆 = 1
64. Banyaknya susunan lineup = 𝐶13 𝐶410 𝐶28 𝐶44 =
𝑆 = 𝑆1 + 𝑆2 (ket: 𝑆1 = Sesi 1 pengerjaan ; 𝑆2 =
17.640
Sesi 2 pengerjaan)
Jawaban A.
𝑆 = (𝑉1 + 𝑉2 )𝑡1 + 𝑉1 𝑡2
65. Jumlah soal benar yang dapat membuatnya
𝑆 𝑆 𝑆 1
lulus tes TPA = {7,8,9,10} 𝑆 = ( + ) 𝑡1 + (2 − 𝑡1 )
3 4 3 4
1 1 1 9
n(lulus tes TPA)=4 1 = ( + ) 𝑡1 + ( − 𝑡1 ) (ingat, tadi kita telah
3 4 3 4
Semua kemungkinan jumlah soal benar TPA =
memisalkan 𝑆 = 1)
{0,1,2,3,4, … ,10} 7 3 1
1= 𝑡 + − 𝑡1
12 1 4 3
n(semua kemungkinan soal benar TPA)=11
3 7−4
Jumlah soal benar yang dapat membuatnya 1 − = 𝑡1 ( )
4 12
1 1
lulus tes TBI = {10,11, … ,15} = 𝑡1
4 4
n(lulus tes TBI)=6
𝑡1 = 1 jam
Semua kemungkinan jumlah soal benar TBI =
{0,1,2, … ,15} Jadi, pertengkaran tersebut dimulai pada pukul
n(semua kemungkinan soal benar TBI)=16 12.00 + 1 jam = 13.00
𝑃(𝑑𝑖𝑡𝑒𝑟𝑖𝑚𝑎 𝑑𝑖 𝑆𝑇𝐴𝑁) =
Jawaban B.
𝑃(𝑙𝑢𝑙𝑢𝑠 𝑡𝑒𝑠 𝑇𝑃𝐴). 𝑃(𝑙𝑢𝑙𝑢𝑠 𝑡𝑒𝑠 𝑇𝐵𝐼)
4 6 3
= . = 67. Coba perhatikan hubungan berikut:
11 16 22
1 1
+ =1
Jawaban A. 5𝑛 +1 5−𝑛 +1

1 1 1 1 1
66. Misalkan, Sehingga, + + ⋯+ + + +
55 +1 54 +1 51 +1 50 +1 5−1 +1
1
kecepatan Putri = 𝑉1 ⋯+
5−5 +1
kecepatan Rima = 𝑉2
Selesainya pekerjaan tersebut = 𝑆 1 1 1 1
=( + )+( + ) + ⋯+
55 +1 5−5 +1 54 +1 5−4 +1
1 1 1 1
( + )+ =5+
Putri dapat menyelesaikan pekerjaan tersebut 51 +1 5−1 +1 50 +1 1+1

𝑆
dalam 3 jam, 𝑉1 =
3 Jawaban D. (5,5)
Rima dapat menyelesaikan pekerjaan tersebut
dalam 4 jam, 𝑉2 =
𝑆 68. Misalkan:
4
̅̅̅
𝑥𝐴 = rata-rata nilai kelas A
Kerangka berpikir:
𝑥𝐵 = rata-rata nilai kelas B
̅̅̅
Pekerjaan tersebut dikerjakan dalam 2 sesi.
𝑥𝑛 = rata-rata gabungan kedua kelas tersebut
̅̅̅
Sesi 1: Pada pukul 12.00 siang mereka memulai
pengecatan bersama-sama
̅̅̅̅.n
xA a +x̅̅̅̅n
B. b
Sesi 2: Putri menyelesaikan pengecatan 𝑥𝑛 =
̅̅̅
na +nb

sendirian setelah mereka bertengkar 73.𝑛𝑎+88.(75−𝑛𝑎)


80 =
75
Kita dapat membuat ilustrasinya sebagai
80.75 = 73. 𝑛𝑎 + 88.75 − 88. 𝑛𝑎
berikut:
6000 = 6600 − 15. 𝑛𝑎
Total waktu pengerjaan pekerjaan tersebut
𝑛𝑎 = 40
1
adalah 2 jam 15 menit (2 jam) dikarenakan
4 Jawaban C.
selama 12.00 – 14.25 mereka sempat 69. 𝑃(𝐴) =
15
.
3
=
3
30 29 58
bertengkar selama 10 menit dan tidak terjadi
Jawaban B.
pengerjaan pengecatan selama 10 menit
70. Banyaknya kemungkinan = 𝑃12 . 𝑃55 = 240
tersebut.
Jawaban D.
𝑥1 +𝑥2 +𝑥3 +𝑥4 +𝑥5
71. 𝑥̅ = 76. Luas permukaan = 2(𝑝𝑙 + 𝑙𝑡 + 𝑝𝑡) = 2(16𝑥6 +
5
𝑥1 +𝑥2 +𝑥3 +𝑥4 +𝑥5 6𝑥12 + 16𝑥12) = 720
7=
5
Jawaban C.
𝑥1 + 𝑥2 + 𝑥3 + 𝑥4 + 𝑥5 = 35
77. Bilangan kuadrat 2-digit = {16,25,36,49,64,81}
𝑥3 = 9 ;
Bilangan kuadrat 3-digit
𝑥2 𝑎𝑡𝑎𝑢 𝑥4 = 9
100,121,144,169,196,225,256,289,324,
={ }
𝑥1 = 1 (agar selisihnya dengan data ke-5 bisa 729,784,841,900,961
maksimal) Dari susunan huruf TO dan NOT, yang
memungkinan hanya:
Kemungkinan 1 : TO = 16
NOT = 361 atau 961
𝑥1 + 𝑥2 + 𝑥3 + 𝑥4 + 𝑥5 = 35
Didapat O = 6 dan T = 1

1 + 9 + 9 + 𝑥4 + 𝑥5 = 35 Maka, TWO = 1W6  W yang memungkinkan


adalah 9 dengan TWO = 196
𝑥4 + 𝑥5 = 16 Didapat N = 3.
Berarti haruslah AGE itu tidak mengandung
Karena 𝑥4 ≥ 9 dan kita ambil nilai 𝑥4 = 9 maka digit-digit 1,3,6,9
didapat nilai 𝑥5 = 7 AGE yang mungkin = 784
A=7, G=8, E=4
Kontradiksi, haruslah 𝑥5 ≥ 𝑥4 ≥ 𝑥3 ≥ 𝑥2 ≥ 𝑥1
TWO + TO + TOO = 196 + 16 + 166 = 378 =

Kemungkinan 2: NAG
Jawaban B.
𝑥1 + 𝑥2 + 𝑥3 + 𝑥4 + 𝑥5 = 35 78. Total debit air yang masuk = 800 – 300 = 500
1 + 𝑥2 + 9 + 9 + 𝑥5 = 35 meter per menit
𝑥2 + 𝑥5 = 16 t = 3750/500 = 7,5 menit = 7 menit 30 detik
Agar 𝑥5 bisa maksimal, haruslah 𝑥2 = 2 karena Jawaban E.
𝑥2 > 𝑥1 , didapat 𝑥5 = 14. 1+4 2+3 5 5
79. 𝑑𝑒𝑡(𝐴 + 𝐵) = 𝑑𝑒𝑡 [ ] = 𝑑𝑒𝑡 [ ]=
3+2 4+1 5 5
Nilai simpangan adalah 13.
5.5 − 5.5 = 0
Jawaban C.
Jawaban C.

72. 𝑛(𝑆) = 𝑛(𝑥) + 𝑛(𝐴) + 𝑛(𝐵) − 𝑛(𝐴 ∩ 𝐵)


100% = 𝑛(𝑥) + 65% + 40% − 25%
80. 2𝑥+𝑦 . 2𝑦+𝑧 . 2𝑧+𝑥 = 2𝑥+𝑦+𝑦+𝑧+𝑧+𝑥 = 22(𝑥+𝑦+𝑧) =
𝑛(𝑥) = 20%
10.20.30 = 6.000
Jawaban D.
80.𝑛+𝑋
22(𝑥+𝑦+𝑧) = 6.000
73. 82 =
𝑛+1
2𝑥+𝑦+𝑧 = √6000 = 20√15
82(𝑛 + 1) = 80𝑛 + 𝑋
2𝑥+𝑦+𝑧 20√15
2𝑥 = = = √15
82𝑛 + 82 = 80𝑛 + 𝑋 2𝑦+𝑧 20

𝑋 = 2𝑛 + 82 Jawaban B.

Jawaban C. 81. 1 ⏟ 1 ⏟ 3 ⏟ 9 ⏟ 21 ⏟ 41 ⏟ 71 ⏟ 113


0𝑥1 1𝑥2 2𝑥3 3𝑥4 4𝑥5 5𝑥6 6𝑥7
74. Dari 00.00 sampai 12.00 :
Jawaban A.
Jam tangan Bayu telah terlambat sebanyak
82. Bilangan prima dikuadratkan lalu dikurangi 1.
12𝑥(−4 𝑚𝑒𝑛𝑖𝑡) = −48 𝑚𝑒𝑛𝑖𝑡 Jawaban C.
Jam tangan Agung telah lebih cepat sebanyak 83. Perhatikan {3,4,5} , {5,12,13} , {7,24, 𝑋}
12𝑥(3 𝑚𝑒𝑛𝑖𝑡) = 36 𝑚𝑒𝑛𝑖𝑡 merupakan kelompok bilangan phytagoras.
Selisih = 36 𝑚𝑒𝑛𝑖𝑡 − (−48 𝑚𝑒𝑛𝑖𝑡) = 84 𝑚𝑒𝑛𝑖𝑡 = Didapat X = 25
1 𝑗𝑎𝑚 24 𝑚𝑒𝑛𝑖𝑡 Jawaban E.
Jawaban B. 84. Lompat satu kali. Kelompok bilangan pertama =
75. Semua jawaban memenuhi permintaan soal, {1,2,3,4,5, … } . Kelompok bilangan kedua =
kecuali opsi B {1,3,5,7,9, … }
Jawaban B.
Jawaban A. Lihat lagi pada soal, premis 𝑄 bernilai benar,
85. Lompat satu kali. Kelompok bilangan pertama = maka cuma no. (1) dan (2) pada tabel di atas
{3,9,27,81, … } yang memenuhi.
Kelompok bilangan kedua = {8,16,24,32,40, … } Pada tabel no. 1 nilai premis 𝑃 benar,
Jawaban A. sedangkan di no. 2 nilai premis 𝑃 salah.
86. Deret itu terdiri dari kelompok huruf = Kesimpulannya, nilai premis 𝑃 bisa benar bisa
{𝐵𝐶𝐷, 𝐶𝐷𝐸, 𝐷𝐸𝐹, 𝐸𝐹𝐺, … } salah.
Lalu deret berikutnya = {𝑃, 𝑄, 𝑅, 𝑆, 𝑇, … } Jawaban D.
Jawaban D. 100. Dengan cara yang sama pada soal
87. Deret itu terdiri dari kelompok huruf = sebelumnya.
{𝐴, 𝐶, 𝐸, 𝐺, 𝐼 … } Jawaban C.
Lalu deret berikutnya = {𝐵, 𝐷, 𝐻, 𝑃} jika 101. Dengan cara yang sama pada 2 soal
dirubah menjadi angka, dikalikan 2. sebelumnya.
Jawaban A. Jawaban C.
88. 𝐴 ⏟
𝑏 𝐶 𝑑𝑒
⏟𝐹⏟𝑔 𝐻 𝑖𝑗
⏟ 𝐾 ⏟𝑙 𝑀 𝑛𝑜
⏟𝑃 102. Jawaban C.

Jawaban A. 103. Jawaban C.

89. Jika alphabet itu dirubah menjadi angka maka 104. Jawaban D.

akan didapat dua kelompok angka yaitu: 105. Jawaban D. Statement ke (viii) pada soal.

Kelompok 1: {𝐵, 𝐶, 𝐷, 𝐸, 𝐹, 𝐺, … } = {2,3,4,5,6,7, … } 106. Jawaban E. Statement ke (vii) pada soal.


107. Jawaban E. yang paling mungkin
Kelompok 2: {𝐵, 𝐶, 𝐸, 𝐺, 𝐾, 𝑀, … } =
menggantikan adalah orang yang sama
{2,3,5,7,11,13, … } = kelompok bil. Prima
posisinya, bisa Anggi atau Wijaya.
Jawaban A.
108. Jawaban D. 5 pemain utama yang mungkin
90. Jawaban D. sudah jelas
paling sering dipasang adalah Martha, Ade,
91. Jawaban E.
Darma, Adli, dan Waris. Berarti semenjak Adli
92. Jawaban B.
cedera di quarterfinal, maka yang mampu
93. Jawaban D.
mencetak point paling banyak adalah Waris.
94. Jawaban B.
109. Angka-angka pada bola tersebut adalah 8, 9,
95. Jawaban B.
10, 11, 12, 13, 14, dan 15.
96. Jawaban D.
Karena bola di W nilainya 4 lebih besar dari
97. Jawaban B.
bola di Z dan bola di W nilainya 3 lebih kecil
98. Jawaban D.
dari bola dalam X, haruslah angka dari bola
99. Perhatikan tabel berikut:
dalam W adalah 12.
No. 𝑃 𝑄 𝑃→𝑄
Bola dalam Z nilainya 8.
1 B B B Bola dalam X nilainya 15.
Jawaban C.
2 B S S 110. Karena U adalah rata-rata dari X dan V,
sedangkan X adalah kotak yang berisi bola
3 S B B
dengan angka tertinggi, haruslah nilai bola
yang ada dalam kotak U > V.
4 S S B
Karena X terbesar, maka pilihan jawaban D

Dengan premis tidak mungkin.

P = Kak Martha ganteng Karena W ada di no. 4, pilihan jawaban B dan

Q = Kak Martha sudah mandi E tidak mungkin.

Lihat pada soal diketahui nilai 𝑃 → 𝑄 bernilai Karena U ada di sebelah kiri V, maka jawaban

benar, berarti pada hanya pada no. (1), (2), dan A tidak mungkin.

(3) pada tabel di atas yang memenuhi. Jawaban C.


111. Jawaban B. Penjelasan ada pada 2 soal
sebelumnya.
112. -Pilihan (i) tidak mungkin karena W lebih 119. D. Ketiga lingkaran saling bertautan.
besar dari U, maka tidak akan ada nilai V 120. C
yang memenuhi.
-Pilihan (ii) benar karena sekarang nilai Y
adalah 11 dan W adalah 12 dan S diketahui
lebih besar dari Y, sehingga haruslah S > W
-Pilihan (iii) benar juga karena V hanya
mungkin mengisi angka 9 atau angka 10
-Pilihan (iv) benar juga karena T hanya
mungkin mengisi angka 9 atau angka 10
PEMBAHASAN BAHASA INGGRIS PREDIKSI I
Jawaban D.
113. Macam acara seni yang mungkin ditampilkan
121. B. Dalam Question tag, Has akan menjadi auxiliary
di acara pentas seni tersebut yaitu: Band,
bila diikuti dengan V3. Has disini adalah V1
drama, tari saman, tari kecak Bali, dan
sehingga harus diikuti auxiliary do/does
Pantomim.
122. A. Kata Remember bisa diikuti oleh V.ing atau To
Si Anggi yang memiliki bakat melukis tidak
+ V1. Ving dipakai apabila kejadian tersbut sudah
mungkin ditampilkan bakatnya di rangkaian
selesai terjadi, sementara To + V1 dipakai apabila
acara pementasan seni tersebut.
kejadiannya belum terjadi.
Jawaban B.
123. B. Conditional sentences type 2 : Would + V1
114. Jay tidak mungkin tampil sebagai pemain
124. A. Passive participle: kalimat pasif yang disingkat
band karena pementasan band dan drama
sehingga to be nya dihilangkan dan diikuti kata
dilaksanakan dalam waktu yang bersamaan.
kerja bentuk ketiga
Jawaban D.
125. A. cukup jelas.
115. Yang pasti ikut adalah si Anggi karena dari
126. B. kalimat tersebut belum mempunyai predikat.
soal belum ditemukan informasi bahwa dia
Subjeknya adalah a period of eight hours. Untuk
akan mengikuti salah satu bidang di rangkaian
durasi waktu selalu merupakan suatu subjek yang
pementasan seni tersebut.
singular sehingga memerlukan predikat berupa to
Jawaban D.
be Is
127. B. Active participle: harus diikuti dengan Ving atau
Having + V3
128. C. Setelah preposisi harus diikuti dengan gerund.
129. D. when + simple past + S + past continuous /
116. B
past perfect / past perfect continuous
130. C. karena kalimat tersebut sudah memiliki S+ P
sehingga kalimat selanjutnya harus diikuti oleh
Noun Clause
131. A. Would prefer + to inf + rather than + V1
132. A. cukup jelas
Kubus yang asli telah berputar sekali ke sisi kanan untuk 133. B. fail harus diikuti oleh to + V1
merubah posisi apa yang ada di atas ke sisi sebelah kanan 134. A. modal perfect should have + V3 = seharusnya
dan memuat sisi atas baru sudah
135. A. is based on
117. A 136. A. setelah article (a,an,the) harus diikuti oleh Noun
(Natural)
Kubus yang asli sudah berputar 180o (berlawanan atau
137. D. Condiitional Sentences Type 3 artinya harus
searah jarum jam)
dalam bentuk simple past dan maknanya harus

118. C. Lingkaran kecil pada gambar ketiga berubah berlawanan dengan Conditional.

menjadi lingkaran besar pada gambar keempat, 138. C. Noun Phrase (Sifat, Ukuran, Umur, Bentuk,

begitu pula sebaliknya. Warna, Asal, Bahan + Noun)


139. D. Conditional sentences type 1 171. A. Feelings
140. C. 172. C
141. B. Setelah mind harus diikuti dengan V.ing 173. A
(Gerund) 174. D
142. D. Present perfect (S+Have/Has +V3) 175. D
The fact is in Present Continous 176. A
(S+Is/Am/Are+Ving) 177. C
143. C. Temporary (adj) 178. B Cermati paragraf 3 kalimat 2.
144. B. Passive Voice (are affected) => kejadiaan 179. C Standfast: teguh pada pendirian
umum 180. D Berdasarkan bacaan, seharusnya each individual
145. D. To be + used to + V.ing can carry large sums of money
146. B. Negative statement => Neither + aux + S atau
S+ aux (-) + either
147. C. had better + V1
148. B. Causative Have/Had dan Objeknya person
sehingga harus diikuti V1
149. A. Full employment menyatakan pekerjaan apapun
yang dikerjakan setiap orang
150. C. to find (menyatakan tujuan) dan to be + able +
to inf
151. A. A large (singular) ……. Is participating
152. D. Rapidly (adv) = dengan cepat
153. C. Parallel structure
Intelligent, capable, aware (V1)
154. B. Eliptical Sentences , Either ….. or
155. B. Because + sentence
156. D. passive voice seharusnya is called
157. A. appear
158. C. real (adj) or imagine (verb) not a parallel
structure
159. D. another
160. B. as
161. A. Paragraf 1, kalimat terakhir “ The butterfly had a
swollen (big) body and small sriveled wings”
162. B. Paragraf 2, baris ke-10 Restricting = hard =
keras
163. D. The kind of the text is NARRATION
164. C. The generic structure of narration is orientation-
complication-resolution-reorientation
165. C. Paragraf terakhir
166. C. Shortly after high school Paragraph kedua “after
she graduated from high school”
167. B. Song and singing Paragraph pertama (singing).
Paragraph kedua (write a song) Pararaf ketiga (ideas
for songs)
168. D. On a trip Paragraph kedua “while riding a train to
Toronto”
169. D. Singer Kalimat ketiga paragraph kedua
170. C. A story put to music “it was a ballad, a simple song
that tells a story”paragraph kedua

Anda mungkin juga menyukai